A Guide to the GRE/Printable version


A Guide to the GRE

The current, editable version of this book is available in Wikibooks, the open-content textbooks collection, at
https://en.wikibooks.org/wiki/A_Guide_to_the_GRE

Permission is granted to copy, distribute, and/or modify this document under the terms of the Creative Commons Attribution-ShareAlike 3.0 License.

Introduction

Introduction to the GRE edit

The Graduate Record Examinations (GRE) is a standardized test used for graduate admissions in the United States and Canada, as well as for English-language studies worldwide. The test is created by Educational Testing Services (ETS), a company which also makes the Praxis and TOEFL exams.

Scoring edit

The GRE is scored on a scale of 130 to 170 for both its verbal and quantitative sections - each gets a separate score.

Other Important Facts About the GRE edit

Paper or Computer Options. The GRE may be taken on paper or using a computer. Test takers may register at www.ets.org.

Computer adaptivity. The computer based version of the test becomes harder or easier based on the test taker's performance, but only between sections. For example, doing well on the first verbal section will cause the second one to increase in difficulty, but not affect remaining questions on the first.

Calculator. Test takers may use a non-scientific calculator for the math sections. This is provided onscreen for the computer version of the test.

No penalty for guessing. Unlike some tests, here is no penalty for marking an incorrect response on the GRE.


Sections

Sections of the Test edit

The GRE has six sections

Section 1: Analytical Writing edit

30 minutes 1 essay question Score of 1 to 6 in half point increments (e.g., 4.5, 5, 5.5) 1 essay analyzing an issue

Section 2: Analytical Writing edit

30 minutes 1 essay question Score of 1 to 6 in half point increments (e.g., 4.5, 5, 5.5) 1 essay analyzing an argument (as opposed to “analyzing an issue" in the first section)

Section 3: Verbal Reasoning edit

35 minutes 25 questions Combined with second verbal section to create a scaled verbal score of 130-170 12-13 sentence completion questions, testing vocabulary 12-13 reading comprehension questions, based on passages

Section 4: Verbal Reasoning edit

35 minutes 25 questions Combined with first verbal section to create a scaled verbal score of 130-170 12-13 sentence completion questions, testing vocabulary 12-13 reading comprehension questions, based on passages This section is identical to the previous section (but see “computer adaptivity”)

Section 5: Quantitative Reasoning edit

35 minutes 25 questions Combined with second math section to create a scaled math score of 130-170 10 quantity comparison questions, asking to compare two values 10-11 standard math questions 4 questions based on reading a data chart

Section 6: Quantitative Reasoning edit

35 minutes Combined with first math section to create a scaled math score of 130-170 10 quantity comparison questions, asking to compare two values 10-11 standard math questions 4 questions based on reading a data chart This section is identical to the previous section (but see “computer adaptivity”)


Math

Introduction to GRE Math edit

The GRE tests only basic math universal to high school curricula. The exam generally does not test any math beyond high school geometry; there is no trigonometry or calculus on the test. Its math can be broken into essentially four categories.

Math Operations edit

The GRE frequently tests knowledge of these mathematical concepts such as the rules of radicals, exponents, absolute value, positives, negatives, and inverses.

Algebra edit

Algebra refers to the use of variables and equations to solve problems, such as by substituting a letter-value for an unknown. The GRE heavily tests algebra, particularly in problems involving rates or proportions.

Geometry edit

Geometry refers to the variety of rules used to calculate attributes of lines and shapes. The GRE tests a great deal of geometry, especially triangles, squares and circles.

Data Interpretation edit

Every GRE math section has a section of questions based on a chart, designed to test data interpretation skills. Many of the other questions test statistical concepts such as averages and standard deviation.


The Two Types of GRE Math Questions

The Two Types of GRE Math Questions edit

There are two types of GRE math questions: quantity comparison, and ordinary problem solving.

Quantity Comparison edit

A quantity comparison question will give two quantities and ask the test taker to compare them. For example:

1. Ashley's pet cockatoo eats   as much bird food as her pet parakeet. Ashley has no other pets.
Quantity A: the percentage of total bird food eaten by Ashley's cockatoo
Quantity B: 65%
(A) Quantity A is greater.
(B) Quantity B is greater.
(C) The two quantities are equal.
(D) The relationship cannot be determined from the information given.

(The correct answer is (A)).

Problem Solving edit

A problem solving question asks a "normal" multiple choice or open-ended question which requires mathematical reasoning to solve.

11. Vivie's pet cockatoo eats as much bird food as her pet parakeet. Vivie has no other pets. What fraction of the total bird food is eaten by Vivie's cockatoo?
(A) 
(B) 
(C) 
(D) 
(E) 

(The correct answer is (E))

The question prompt may be multiple choice or may be open-ended, asking test takers to bubble in the value. Additionally, some may have three or seven choices and more than one correct answer. This will be clearly indicated. Also, approximately four problem solving questions around questions 17-20 will be based on a chart.


Integers

Integers edit

Though not explicitly tested, the concepts of integers and whole numbers are important to solving GRE math problems.

Rules edit

An integer is a number without any fraction or decimal behind it.

1, 3, -5, and 268 are integers, while 1.77 and   are not.

A whole number is a non-negative integer.

0, 1, 2, and 3 are whole numbers; .62, -8, and   are not.

Practice edit

Determine whether each of the following is an integer, a whole number, or neither.

1. -2

2. 113.7

3.  

4. 593

Comments edit

The GRE isn't going to ask questions such as “which of the following is an integer?” but the concept is important for solving other questions. For instance, if 1 < x < 5, x could have an infinite number of values, from 1.00001 to 3.0065. But if x is an integer, its values are now limited to 2, 3, and 4.

Answers to Practice Questions edit

1. -2 is an integer – it does not end in a fraction or decimal. However, it is not a whole number. Whole numbers are positive.

2. 113.7 is not an integer because it ends in a decimal. Because it is not an integer, it is not a whole number either. Remember, whole numbers are positive integers.

3.   is not an integer – it is a fraction. Integers, remember, are numbers which do not end in fractions or decimals. Because it is not an integer, it is not a whole number either, since whole numbers are positive integers.

4. 593 is an integer because it does not end in a decimal. It is also a positive (non-negative) integer so it is also a whole number. Remember, all whole numbers are integers but not all integers are whole numbers, only if they are non-negative.


Absolute Value

Absolute Value edit

The concept of absolute value - meaning a number's distance from zero - is tested on nearly every GRE.

Rule edit

Absolute value makes a negative positive, but otherwise does nothing.

“| |” designates absolute value. For example, if | x + 3 | = 5, there are two possible values for x:

  • x + 3 = 5, meaning x is 2
  • x + 3 = -5, meaning x is -8

On an absolute value questions, split the value into two equations as seen above.

Practice edit

1. If | 3x - 4 | = 5, then what could be the value of x?

2. If | a | > a, then what is the greatest integer that a could be?

3. If 3|4k - 2| - 12 = -3, what is the value of k?

Comments edit

Absolute value tends to be tested in the quantity comparison section of the test, often with a variable modified by a constant within the absolute value. (e.g. | q + 7 | = 5) Solve these by writing out both of the potential values for the variable, and remember that either one could be the value. For example, in the prior equation, q could equal either -2 or -12, so it is unclear whether it is greater or less than -5.

Answers to Practice Questions edit

1. 3,  

If | 3x - 4 | = 5 then

3x - 4 = 5

or

3x - 4 = -5

3x - 4 = 5

Take the first equation and solve it. First, add 4 to both sides.

3x = 9

Now divide both sides by 3.

x = 3

x is equal to 3. But remember, this is just one solution - you still need to solve the other equation.

3x - 4 = -5

Now take the second equation and solve it. Add 4 to both sides.

3x = -1

Now divide both sides by 3.

x =  

x is equal to negative one third.

This means that x = 3 or  

2. -1

Absolute value makes a positive negative, but otherwise does nothing - if the absolute value of a number is greater than that number itself, the number must be negative. The greatest negative number is -1.

3. k =  5/4, 

If 3|4k - 2| - 12 = -3, then

3|4k - 2| = 9

Add 12 to both sides.

|4k - 2| = 3

Divide both sides by 3.

4k - 2 = 3

Split both possibilities.

4k - 2 = -3

Add 2 to to both sides

4k = 5 .

Divide both sides by 5.

k =  


Positives and Negatives

Positives & Negatives edit

Subtracting a negative is the same as adding a positive.

5 - -3 = 5 + 3 = 8

Adding a negative is the same as subtracting a positive.

2 + -5 = 2 – 5 = -3

Multiplying a negative and a positive equals a negative.

5(-3) = -15

Multiplying a negative and a negative equals a positive.

-7(-3) = 7(3) = 21

Thus, multiplying any odd number of negatives together equals a negative, while multiplying any even number of negatives together equals a positive.

Practice edit

1. What is the value of 8(-4)?

2. If -2 - y is positive, then is y negative or positive?

3. x^77 is positive. Is x negative or positive?

Answers to Practice Questions edit

1. -32

Multiplying a positive by a negative equals a negative.

2. y must be negative (and must be less than -2)

Subtracting a negative is the same as adding a positive. For example, y could equal -3. -2 - -3 = -2 + 3 = 1.

3. x must be positive.

A negative multiplied by a negative equals a positive (e.g., -1 ✕ -1 = 1), while a positive multiplied by a negative equals a negative (e.g., (-1 ✕ -1) ✕ -1 = 1 ✕ -1 = -1). Therefore, -1(-1)(-1) is negative, while -1(-1)(-1)(-1) is positive. Thus, any time an odd number of negatives are multiplied together, they equal a negative - meaning x must be positive, since 77 xs multiplied together are positive.


Odds and Evens

Odds & Evens edit

Adding & Subtracting edit

An odd plus an odd equals an even.

5 + 7 = 12 = even

An even plus an even equals an even.

6 + 4 = 10 = even

An odd plus an even equals an odd.

6 + 5 = 11 = odd


Multiplying & Dividing edit

An odd multiplied by an odd equals an odd.

5(5) = 25 = odd

An odd multiplied by an even equals an even.

5(2) = 10 = even

An even multiplied by an even equals an even.

4(8) = 32 = even

When in doubt, test this with two known numbers. If unsure, try 1 + 3 or (2)(5) and see what type of number it yields.

Practice edit

1. xy is even and xz is odd. Is y odd or even?

2. a + b + c is even. If a is odd, then could b and c be

both odd?

both even?

odd and even?

3. (f + g)h is an odd number. If f is even, is g + h odd or even?


Answers to Practice Questions edit

1. y must be even.

The product of two numbers is only odd if both numbers are odd. Since xz is odd, both x and z must be odd. However, xy is even. This means that between x and y, one must be even. Since x is odd, y must be even.

2. b and c must be odd and even, not necessarily in that order.

An odd plus an odd equals an even, as does an even plus an even. Only an odd plus an even equals an odd. Since a + b + c is even and a is odd, b + c must be odd. This means that, between b and c, one must be odd, and one must be even. If they were both odd or even, b + c would be even, meaning a + b + c would be even, but it isn't.

3. g + h must be even.

Only an odd and another odd multiply to make an odd. Therefore, since (f + g)h is odd, f + g must be odd. Since f is even, g must be odd, since only an even and an odd add up to an odd. h also must be odd, since it multiplies to make an odd. Therefore, since an odd plus an odd equals an even, g + h must be even.


Prime Numbers

Prime Numbers edit

A prime number is a number only divisible by itself and 1.

Prime numbers between 1 and 100 are as follows:

2, 3, 5, 7, 11, 13, 17, 19, 23, 29, 31, 37, 41, 43, 47, 53, 59, 61, 67, 71, 73, 79, 83, 89, 97

Some numbers like 51, 57 or 93 may look like prime numbers but are in fact divisible by an oddball prime number. These three are divisible by 17, 19 and 31, respectively.

Non-prime numbers can be divided into their prime factors.

A “factor” is a number by which a larger number can be divided. Non-prime numbers can be broken down into their prime factors using a factor tree.

24

         |     |
       2       12
              |     |
            2        6
                    |   |
                   2     3
 24 = 2(2)(2)(3) or 23(3)

This expression is known as the “prime factorization” of 24. When factoring numbers, it is typically easiest to divide by 2's and 3's when possible.

Practice edit

1. What is the prime factorization of 100?

2. How many prime numbers are greater than 50 but less than 60?

3. Express 372 as a product of prime numbers.

Answers to Practice Questions edit

1. 22(52) or (2)(2)(5)(5)

Prime factorization can be achieved by drawing a factor tree - picking any two numbers that a number divides into, and dividing downward.

100

          /      \
        4        25
      /    \     /    \
    2      2  5     5

The bottoms of the branches are the prime factors. It doesn't matter which two numbers are picked first. All factor trees lead to the same numbers.

100

          /      \
        5        20
      	      /    \

2 10 / \

	          2      5

2. Two - 53 and 59.

51 is divisible by 17, while 57 is divisible by 19.

3. 22(3)(31) or 2(2)(3)(31)

            372
         /       \
       2        186
                /     \
              2       93
                      /    \
                    3     31

As you can see, it is usually easier when factoring to take out 2's or 3's at first.


Least Common Multiple

Least Common Multiple edit

The least common multiple of two numbers is the number that contains all of each's prime factors in the appropriate quantities, but nothing more.

For instance, the least common multiple of 12 and 15 is 60.

12 15

         /     \			           /    \
       2        6			         3      5
               /    \
             2       3

The prime factorizations of 12 and 15 are 2(2)(3) and (3)(5). Their least common multiple is 2(2)(3)(5), or 60. The numbers share a prime factor, 3.

If numbers do not share any prime factors, the least common multiple is the product of the two numbers. For instance, the least common multiple of 7 and 12 is 84.

If numbers share all of the same factors, such as 16 and 32 (which are 2(2)(2)(2) and 2(2)(2)(2)(2), respectively), the least common multiple is the greater number.

Practice edit

1. What is the least common multiple of 20 and 15?

2. If the least common multiple of two numbers is 36, and one of the numbers is 9, what is the other number?

3. The least common multiple of 14 and 16 is how much greater than the least common multiple of 5 and 12?

Answers to Practice Questions edit

1. 60

20 15

         /      \			           /    \
       2       10			         3      5
               /    \
             2      5

The prime factorization of 20 is 2(2)(5), while the prime factorization of 15 is (3)(5). Their least common multiple will thus be 2(2)(3)(5) - the number which contains all of the prime factors of each in the appropriate quantities, but nothing more. The numbers share a common factor - 5 - thus, the 5 need not be repeated.

2. 4

36

         /      \
       4         9			9
      /   \      /  \ 		          /    \
    2     2   3    3	        3       3

The prime factorization of 36 is 2(2)(3)(3). The prime factorization of 9 is 3(3). Since 36 is the least common multiple of 9 and the other number, this other number must be 2(2), or 4, since the least common multiple of two numbers is the number which contains the prime factors of each in the appropriate quantities, but nothing more.

3. 14=7(2)

16=2(2)2(2).

LCM of 14 and 16 → 7(2)2(2)2 = 112.

5=5

12=3(2)2.

LCM of 5 and 12 → 2(2)3(5) = 60.

112-60=52.


Greatest Common Factor

Greatest Common Factor edit

The greatest common factor of two numbers is the product of the prime factors which they have in common in the same quantities.

For example, the greatest common factor of 60 and 48 is 12.

60 48

         /     \			           /    \
       2        30			         2       24
               /    \				     /    \
             2       15                                   2       12
                     /    \ 			          /      \
                   3       5			        2        6

/ \

                                                                         2      3

60 is equal to 2(2)(3)(5), while 48 is equal to 2(2)(2)(2)(3). Their shared prime factors are 2, 2 and 3, which equal 12.

Practice edit

1. What is the greatest common factor of 180 and 216?

2. If the greatest common factor of 84 and y is 6, and the greatest common factor of 125 and y is 5, what is the lowest possible value for y?

3. Which is larger - the greatest common factor of 120 and 192, or the greatest common factor of 128 and 192?


Answers to Practice Questions edit

1. 36

180 216

          /      \               /       \
        2        90          2       108
                 /    \                 /      \
               2     45            2        54
                     /    \                    /    \
                   3      15               3     18
                          /    \                    /    \

3 5 3 6 / \ 2 3

The prime factorization of 180 is 2(2)(3)(3)(5), while the prime factorization of 216 is 2(2)(2)(3)(3)(3). The greatest common factor is the product of the prime factors they have in common in the same quantities - 2(2)(3)(3), or 36.

2. 30

If the greatest common factor of 84 and y is 6, then y at a minimum must have prime factors of 2 and 3. If the greatest common factor of 125 and y is 5, then y must have 5 as a factor too. Thus, y must at least equal 2(3)(5), or 30.

3. The greatest common factor of 128 and 192

This question requires determining the prime factorization of each number.

       120				192				128
      /       \			          /        \                                      /       \
   2         60			        2          96	                     2         64

/ \ / \ / \

               2      30			     2      48                                     2    32

/ \ / \ / \ 2 15 2 24 8 4 / \ / \ / \ / \ 3 5 2 12 4 2 2 2 / \ / \ 2 6 2 2 / \ 2 3

The prime factorization of 192 is 2(2)(2)(2)(2)(2)(3). Its greatest common factor with 120 is thus 2(2)(2)(3), or 24, while its greatest common factor with 128 is 2(2)(2)(2)(2)(2), or 64.


Order of Operations

Order of Operations edit

The order of mathematical operations is parentheses, exponents, multiplication and division, and finally, addition and subtraction.

The acronym is “Please Excuse My Dear Aunt Sally.”

Within these categories, it doesn't matter which operation is performed first. Init doesn't matter whether 6 is divided by the 7 first or multiplied by the 8 first; the result will be the same. However, with parentheses, the interior parentheses must be worked out prior to working out any exterior parentheses. 2(3(4 + 5)) requires adding the 4 and 5 before multiplying the result by 3.

“Divide bars” operate as parentheses.

Inthe 6 and 7 are added first, unlike inor a similar expression.

Practice edit

1. What is the value of?

2. If= 45 then what does q equal?

3. If 5(62)(3 + 2) = 5 + e, what is the value of e?


Answers to Practice Questions edit

1. 6

Subtract the 2 from the 3 first, then multiply the 5 by the 1, then add the 4 to the 5. Multiply the 6 by the 9, then divide by 9 to arrive at 6.

2. 13

Start with the initial equation.

Work out the parentheses.

Multiply both sides by 7.

Expand the parentheses.

Subtract 24 from both sides.

Divide both sides by 3. q is 13.

3. 895

5(62)(3 + 2) = 5 + e Start with the initial equation.

5(36)(5) = 5 + e Work out the operations inside the parentheses.

900 = 5 + e Multiply the parenthetical amounts together.

895 = e Subtract 5 from both sides. e is 895.


Fractions

Fractions edit

To add or subtract fractions, first adjust them so that they have a common denominator. This can be achieved by “cross-multiplying”, or multiplying each by the other's denominator.

Fractions can be adjusted by multiplying the numerator (top) and denominator (bottom) by the same value.has the same value as

The value of + isn't but rather,

+	Take the original fraction.
	Cross multiply.
+ 	Work out the parentheses.
     	Add accordingly.

To multiply fractions, multiply numerators by numerators and denominators by denominators.

multiplied byis

To divide fractions, invert one of the fractions, then multiply them.

divided by= multiplied by=

It doesn't matter which fraction is inverted.

Always reduce fractions which aren't in their lowest terms

is equal toTo reduce a fraction, divide both the numerator and the denominator by a common factor.  For example, both 3 and 6 are divisible by 3; thus, equals

An “improper fraction” is a fraction such as in which the numerator exceeds the denominator.is the same asor 1.5.

Practice edit

Solve for the value of each of the

following.

1. +

2. ()
3. ÷

Answers to Practice Questions edit

1.

+			Take the initial equation.
+		“Cross multiply.”  Multiply each fraction by the 					other's 	denominator.
+		Work out the parentheses.
+=	Add the numerators - leave the denominators be.
= 	Reduce the final fraction by dividing both the 					numerator and denominator by 2, 3, or another 					common factor.
2.

To multiply fractions, multiply numerators by numerators and denominators by denominators - 2(1) over 9(3).

3.

To divide fractions, invert and multiply. This yields a result of 5(1) over 2(1).


Reciprocals and Inverses

Reciprocals & Inverses edit

The larger the denominator or divisor, the lower the value.

y =is an example of an inverse correlation. As y increases, x decreases.

Thus, if a is positive, is greater than

If s is positive, 1 ÷ s is greater than 1 ÷ (s + 1).

Practice edit

1. If g = 7 ÷ h, then which is greater: 7 ÷ (h + 1), or g?

2. If< .5, what must be true about y in relation to x?

3. If= 40, then what is the value of ?


Answers to Practice Questions edit

1. g is greater.

As the denominator increases, the value decreases. Thus, g, which is equal tois greater than

2. y must be greater than x.

If the numerator is less than half of the denominator, the fraction is less than .5; if the numerator is greater than half of the denominator, the fraction is greater than .5. For instance, equals which is less than or

3. If= 40, = 20

Doubling the denominator cuts the value in half. is half of


Percentages

Percentages edit

Percentage is a fraction of 100. 11% = .11 =

Percent simply means “per hundred.” However, be careful not to mix up the two key ways in which percentages are used.

Do not mix up changing to a percentage with changing by a percentage.

If a group of 100 members decreases to 20% of its membership, its membership goes down to 20. If the group decreases by 20%, it goes down to 80. Increasing to 110% is the same as increasing by 10%. This also applies to fractions and decimals but is an especially common error with percentages.

Practice edit

1 What is 30% of 80?

(Questions 2-3) If a bank account worth $10,000 increased in value by 25%...

2 ...how much did it increase?

$

3 ...how much is it worth now?

$

(Questions 4-5) If a bank account worth $10,000 decreased in value to 25% of its worth...

4 ...how much did it decrease?

$

5 ...how much is it worth now?

$


Ratios

Ratios edit

A ratio express the proportion of two commodities in relation to each other (as opposed to in relation to the whole, as a fraction or percentage does).

Rules edit

Add both numbers in a ratio to get the total amount with respect to each.

A ratio expresses quantity in relation to the other. This is not to be mixed up with parts of the whole. If the ratio of Democrats to Republicans at a conference is 6 to 4, then Republicans make up 40% of the conference (4 divided by 6 + 4). If guitar players make up 70% of the rock band seminar and drummers make up 30%, then the ratio of guitarists to drummers is 7 to 3.

Practice edit

1. Of the 50 lawyers at the meeting, 22 are bankruptcy lawyers; the remainder are divorce lawyers. What is the ratio of bankruptcy lawyers to divorce lawyers?

2. The ratio of stocks to bonds in the portfolio is 4 to 9. If there are 169 items in the portfolio, how many of them are bonds?

3. The ratio of ratio questions to non-ratio questions on the math quiz is 4 to 5. If there are a total of 36 questions, how many are ratio questions?

Comments edit

As alluded, the most common mistake area in ratios is forgetting that the ratio is an expression of one quantity in relation to each other and not the total amount. Whenever presented with a ratio, a test taker must add the two numbers to get the whole amount.

Answers to Practice Questions edit

1. 11 to 14

The ratio is 22 to (number of divorce lawyers) = 22 to (50 - 22) = 22 to 28 = 11 to 14

2. 117

The ratio of stocks to bonds is 4 to 9. Thus, the fraction of the portfolio which is bonds is =

Always remember to add both numbers in a ratio to get the total amount.

multiplied by 169 is 117.

3. 16

The ratio of ratio questions to non-ratio questions is 4 to 5. Thus, the fraction of the quiz which is math questions is = multiplied by 36 is 16.


Decimals

Decimals edit

Decimals are an extension of the base ten numeral system to fractions of ten (such as hundredths and thousandths). In this way they are a variation of fractions and percentages.

Rules edit

Decimals are fractions of 10, 100, 1000, and so on.

You will have access to a calculator on the GRE but it is nonetheless important that you understand the concept of decimals.

.1 is equal to one tenth; .01 is equal to one hundredth; .001 is equal to one thousandth. Decimals are added and subtracted in the same way ordinary numbers are.

When multiplying decimals, multiply the numbers without decimals; then, push the decimal place to the right once for each time each multiplier is pushed to the right of the decimal.

.05 multiplied by .05 equals .0025, not .25. 6 multiplied by .9 is 5.4, while .6 multiplied by .9 is .54. When in doubt, check these on the calculator.

Practice edit

1. What is .04 multiplied by .08?

2. y is equal to .4(x). If x equals 20, what does y equal?

3. What is the value of .009(.008)(.007)?

Answers to Practice Questions edit

1. .0032

4(8) is 32. .04 and .08 are both pushed 2 points to the right of the decimal; thus, their product is pushed 4 points to the right.

2. 8

.4(20) = (20) = 8

3. 0.000000504.

The multipliers are each pushed to the right of the decimal 3 times. Thus, we take their product without decimals, and push it to the right 9 times.

9(8)(7) is 504; which pushed to the right 9 times equals 0.000000504.


The Up and Down Rule

The "Up & Down" Rule edit

Increasing by a fraction or percentage requires decreasing by a smaller fraction or percentage to get back to the same quantity.

If a dress that sells for $100 increases in price by 25%, its price goes up to $125. To return the dress to its original price, its price would have to be reduced by 20%, not 25%. This is because the base number has increased. Increasing 60 by 20% yields a value of 72. 72, however, will have to be reduced by a lower percentage - about 16% - to get back to 60. With fractions, going down takes less than going up, since fractions are based on parts of the whole.

Practice edit

1. A grocery store sells one third of its shortening. What percentage of its remaining shortening will it have to restock to restore the shortening supply to its present value?

2. An accountant's fees increase by 15%, but the account promises a discount to loyal customers which will make prices equal to the rate before the increase. What fraction of the new rate will the accountant have to charge customers receiving the discount?

3. Office supply sales increase by 40% in April but fall back to their original value in May. What fraction of April's office supply sales are May's office supply sales?

Answers to Practice Questions edit

1. 50%

After selling of the shortening, remain. We are looking for the amount needed to return the shortening to the previous level. Let r equal the fraction that will be restocked.

+ r= 1 Take our initial equation.

r= Subtract from both sides.

r = .5 = 50% Divide both sides by r equals 50%.

2.

Let f equal the fraction that the account will charge of the new rate.

115(f) = 100

f == Divide both sides by 115. The fraction is

3.

edit


Rates and Distances

Rates & Distances edit

Distance, rate and time problems are commonly tested on the GRE. These problems are a common type of algebra problem encountered on the test, which test setting up equations and working with variables.

Rule edit

Distance equals rate multiplied by time.

If there are different rates for different periods of time, distance equals each rate multiplied by each unit of time.

For example, if a ship is traveling at 20 nautical miles per hour for 1.1 hours, the ship will travel a total of 22 nautical miles.

Practice edit

1. A train travels 360 miles over the course of 8 hours. How fast does the train travel?

2. A car travels at 45 miles per hour. How long will it take for the car to travel 225 miles?

3. If a truck travels 175 miles over the course of 3.5 hours, how fast is the truck traveling?

Comments edit

A common problem on the GRE is the so-called "brain-lapse mistake", where the person knows how to solve the problem but merely "spaces" mentally and puts the wrong number in the wrong part of the equation.

Rate and time problems, along with problems involving ratios and percentages are a common source of brian-lapse mistakes and thus test-takers who struggle with this particular problem should make sure to double-check rate and time problems on their GRE.

Answers to Practice Questions edit

1. 45 miles per hour

Distance equals rate multiplied by time, thus:

360 = r(8)

Take our initial equation.

45 = r

Divide both sides by 8.

2. 5 hours

Distance equals rate multiplied by time; thus

t(45) = 225

Take the initial equation.

t = 5

Divide both sides by 45.

3. 50 miles per hour

Distance equals rate multiplied by time, thus:

r(3.5) = 175

Take the initial equation.

r = 50

Divide both sides by 3.5.


Work and Time Problems

Works & Time Problems edit

On a work and time problem, calculate how much work each person does in one hour and solve using these fractions.

Crew A can clean up the lot in 7 hours. Crew B can do it in 5 hours. How long will it take both crews to clean up the lot together?

Crew A can do 1/7 of the work per hour while Crew B can do 1/5 of the work per hour. Combined they can do 1/7+1/5 = 5/35+7/35 = 12/35 per hour. This means it will take them 35/12 hours, or 2.91666...67, to finish.

Practice edit

1. Hal can paint a barn in 6 hours. Mary can do it in 9 hours. How long will it take them to paint the barn if they work together?

2. It takes Stephanie 4 hours to fix a tractor, while it takes Bruce 5 hours. How long will it take Stephanie and Bruce to fix the tractor if they work together?

3. Division I takes 11 hours to complete a report, while Division II takes 12 hours to complete one. How long will it take both divisions to complete a report if they work together?

Answers to Practice Questions edit

1. hours

Hal's rate: barns painted per hour.

Mary's rate: barns painted per hour.

Their combined rate: += += barns painted per hour.

Their time to paint one barn:== =hours.

2. hours

Stephanie's rate: tractors fixed per hour.

Bruce's rate: tractors fixed per hour.

Their combined rate: += += tractors fixed per hour.

Their time to fix one tractor:== =hours.

Answers to Practice Questions

3.hours

Division I's rate: reports per hour.

Division II's rate: reports per hour.

Their combined rate: += += reports per hour.

Their time to make 1 report:== =hours.


Exponents

Exponents edit

The GRE frequently tests exponents, which are numbers indicating how many times a value has been multiplied into a given system.

Rule edit

When multiplying exponents, add the exponent numbers; when raising exponents to a power, multiply the exponent numbers.

An exponent is a statement of how many times a number has been multiplied into a given system. 25 is equal to 52 or 5(5). Multiplying 52 by 52 equals 5(5)(5)(5) or 54. Raising an exponent to a power multiplies the exponent numbers, while multiplying exponents merely adds their numbers.

a3(a4) = a7
(b5)6 = b30

A negative exponent equals one divided by the number raised to that exponent; a fractional exponent indicates a radical of the corresponding degree.

Exponent of "0" is equal to 1 edit

A number with an exponent of “0” equals 1.

x0 = 1

Practice edit

1. What does y2(y5) equal?

2. If (m4)4 = mx what does x equal?

3. If   is equal to q−2 then what does q equal?

Answers to Practice Questions edit

1. y7

When multiplying, add the exponent numbers.

2. 16

When raising exponents to a power, multiply the exponent numbers.

3.  

Negative exponents indicate 1 divided by the number raised to this exponent.

q−2 is   so since q−2 =  , then q2 must equal 2 and q must equal  .


Scientific Notation

Scientific Notation edit

In scientific notation, move the number left of the units digit once for each power of 10, or right of the decimal point for each negative power of 10.

1.76642 x 1018 = 1,766,420,000,000,000,000

1.76642 x 10−18 = .0000000000000000176642

Scientific notation is a method of expressing very large or very small numbers. It consists of a single number multiplied by a power of 10.

Practice edit

1. What is 18,221 expressed in scientific notation?

2. What is 6.5438 x 1011 expressed in conventional notation?

3. Express .0000652 in scientific notation.

Answers to Practice Questions edit

1. 1.8221 x 104

Move the decimal place to the left of the first number, then multiply by a power of 10 for each place you slid it to the right, or by a negative power of 10 for each place you slid it to the left.

2. 654,380,000,000

Slide the decimal place to the right once for each power of ten. Add a zero if there is no number to slide.

3. 6.52 x 10-5

Move the decimal place to the left of the first number, then multiply by a power of 10 for each place you slid it to the right, or by a negative power of 10 for each place you slid it to the left.


Radicals

Radicals edit

Radicals can be multiplied and divided, and perfect squares can be extracted.

A radical is a number which equals another number if multiplied by itself a given number of times. The most common type of radical is the square root. Square roots can be multiplied and divided, as indicated above, but they cannot be added or subtracted.does not necessarily equal , but does equal

Mathematicians general frown on placing a radical in the denominator of a fraction. Adjust radicals in denominators by multiplying the numerator and denominator by the radical. For example,

Practice edit

In these problems, solve for x.

1.

2.

3.


Answers to Practice Questions edit

1.

Take the initial equation.

Multiply both sides by

Divide both sides by 3.

2. y = 3

Take the initial equation.

Square both sides.

Work out the parentheses.

Subtract 5 from both sides.

3.

Take the initial equation.

Multiply both sides by

Divide both sides by 7.


Mean, Median and Mode

Mean, Median & Mode edit

The GRE frequently tests the three different mathematical averages, known as the mean, median, and mode.

Rules edit

The “mean” is the sum of all elements divided by the number of elements in the set.

The “median” is the middle number in a series. If there are two middle numbers, the median is halfway between them.

The “mode” is the number that appears the most frequently.

“Average” is a term used for various data measures to determine what is “normal” in a series of numbers. For example, suppose in four quarters of a basketball game, the basketball team scores 14, 15, 14, and 13 points. The median number of points scored is 14 - the middle number. The mode is also 14.

As mentioned above, "mean" is calculated by adding all the numbers together and dividing by the number of elements in the set. For example the numbers 6, 10, and 20, would have a mean value of 12. (6 + 10 + 20 = 36, divided by 3 = 12)

“Mean” as used on the GRE refers to “arithmetic mean.” There is also a concept called “geometric mean” which the appropriate root of the product of all the numbers. It is not tested on the GRE.

Practice edit

1. Four friends each spend money at a shoe store. Andy spends $145, Becky spends $122, Carla spends $140, and Donald spends $182. What is the median expenditure on shoes?

2. H&T Holdings' stock portfolio had a value of $1.1 million in February, $1.5 million in March, and $1.6 million in April. What was the average (arithmetic mean) value of H&T's stock portfolio over these months?

3. Erin works 14 overtime hours during week 1, 16 overtime hours during week 2, and 17 overtime hours during weeks 3 and 4. Which is greater - the median overtime hours Erin worked, or the arithmetic mean of overtime hours that Erin worked?

Comments edit

The GRE likes to ask questions which test the nuances of the rules, such as what the median is in an even-numbered set. The test will also commonly scramble numbers and ask the test taker to pick out the median or mode - in which case sorting the numbers numerically can be useful.

A commonly asked question is what the "mode" is in a series where two numbers both show up an equal number of times. The answer from a statistical perspective is that both of these numbers are modes (this is known as a multimodular dataset). However, the answer from a GRE preparation perspective is that this is not tested on the GRE.

Answers to Practice Questions edit

1. $142.50

The median is the middle number, or halfway between two middle numbers.

The numbers are $122, $140, $145, and $182; thus, the median is $142.50 - halfway between $140 and $145.

2. $1.4 million

Mean is the sum of all values divided by how many values there were.

= $1.4 million

3. The median is greater.

The median is the middle number, or halfway between two middle numbers. The three numbers in this case are 14, 16, and 17; thus, the median is 16.

Mean is the sum of all values divided by how many values there were. The mean in this case is thus:


Working with Equations

Working with Equations edit

An equation can be altered in any manner so long both sides are equally affected.

An equation is a statement that two quantities are equal to each other. Anything can be done to an equation so long as it is done to both sides.

x = 2y It is true that x is equal to 2 times y.

100x = 200y Multiplying both sides by 100 yields another true statement.

x + 50 = 2y + 50 Adding 50 to both sides also yields another true statement.

If there is a variable on both sides of an equation, consolidate it on one side.

3x - 12 = x + 2

 - 2	   -2 		Subtract 2 from both sides

3x - 14 = x

  -x	  -x		Subtract x from both sides

2x - 14 = 0

 + 14    + 14		Add 14 to both sides

2x = 14

÷ 2      ÷ 2		Divide both sides by 2

x = 7

In an inequality (such as x < y), the same rules of equations apply, except that when multiplying both sides of an inequality by a negative number, the inequality sign must be flipped around. For example, if a > b, then -a < -b.

Practice edit

Solve for the variable in the following equations.

1. 6x - 4 = 1 + x

2. 2a = 3. 3h - 17 = h + 4

Answers to Practice Questions edit

1. x = 1

6x - 4 = 1 + x Take the initial equation.

6x = 5 + x Add 4 to both sides.

5x = 5 Subtract x from both sides.

x = 1 Divide both sides by 5.

2. a =

2a = Take the initial equation.

2a2 = 16 Multiply both sides by a.

a2 = 8 Divide both sides by 2.

a == Take the square root of both sides.

3. h = 10.5

3h - 17 = h + 4 Take the initial equation.

3h = h + 21 Add 17 to both sides.

2h = 21 Subtract h from both sides.

h = 10.5 Divide both sides by 2.


Setting up Equations

Setting up Equations edit

About a third of GRE math questions involve "algebra" - the process of substituting a letter for an unknown value and manipulating statements to solve a problem. One of the most important components of using algebra is knowing how to "set up" the initial equation; that is, to express the statement in terms of variables and mathematical operations.

Rule edit

Solve for unknown elements by substituting a letter for an unknown variable and adjusting the equation accordingly.

For example, suppose a cheetah can run 4 miles per hour faster than twice a giraffe's top speed. If the giraffe's top speed is 20 miles per hour, how fast can the cheetah run?

Let c equal the cheetah's top speed and g equal the giraffe's top speed.

c = 2g + 4

Take the initial equation.

c = 2(20) + 4

Substitute the known value for g.

c = 40 +4

Multiply out the parentheses.

c = 44

Add the numbers. c is 44.

Practice edit

Set up and solve the following word problems.

1. One slice of cake has 150 more calories than two slices of pie. If one slice of pie has 80 calories, how many calories does a slice of cake have?

2. Gary has worked at the auto shop 5 years longer than Pat, who has worked there 3 years less than Lindsay. If Lindsay has worked at the shop for 6 years, how many years has Gary worked there?

3. Jennifer has twice as many dachshunds as she has yorkies, and she has three fewer yorkies than poodles. If Jennifer has 8 dachshunds, how many poodles does she have?


Comments edit

Most (but not all) questions which involve a story or sentences will need to be set up as algebraic equations. These so-called "word problems" are very common on the GRE. Typically the mathematical operations themselves are not hard; rather, the test tends to trip up test takers on setting the problem up the right way.

Answers to Practice Questions edit

1. 310

Let c equal the calories in a slice of cake and p equal the number of calories in one slice of pie.

c = 2p + 150

Take the initial equation.

c = 2(80) + 150

Substitute a known value for p.

c = 160 + 150

Work out the parentheses.

c = 310

Add the values. A slice of cake has 310 calories.

2. 8 years

Let g equal the number of years Gary has worked, p equal the number of years Pat has worked, and l equal the number of years Lindsay has worked.

p = l - 3

Take one of the initial equations.

p = (6) - 3

Substitute the known value for l.

p = 3

Subtract the values. Pat has worked for 3 years.

g = p + 5

Take the other initial equation.

g = (3) + 5

Substitute the known value for p.

g = 8

Add the values. Gary has worked for 8 years.

3. 7

Let d equal the number of Jennifer's dachshunds, y equal the number of Jennifer's yorkies, and p equal the number of Jennifer's poodles.

2y = d

Take the initial equation. d is twice y.

2y = 8

Substitute the known value for d.

y = 4

Divide both sides by 4. Jennifer has 4 yorkies.

p = y + 3

Take the other initial equation.

p = (4) + 3

Substitute the known value for y.

p = 7

Add the values. Jennifer has 7 poodles.


Systems of Equations

Systems of Equations edit

Two equations with two variables can be solved by adjusting one equation so that one variable is expressed in terms of another, and substituting this value into the other equation.

In 3x + 3y = 15, x and y could conceivably have many values, from 1 and 4 to 7 and -2, and so on. Both variables can not be determined using this equation. However, if a second equation is added, it is possible.

3x + 3y = 15 These two are known as a “system of equations.”

x - y = 1 To solve, begin with the second equation.

x = y + 1 Phrase the second equation in terms of one variable. In this case, this is done by adding y to both sides.

3(y +1) + 3y = 15 Substitute this value into the other equation.

3y + 3 + 3y = 15 Expand the parentheses.

6y + 3 = 15 Combine the variables.

6y = 12 Subtract 3 from both sides.

y = 2 Divide both sides by 6. y is 2 (making x 3).

Practice edit

1. 5a - 3b = 21

2a + b = 15

If a and b satisfy the system of equations above, what is the value of a + b?

2. 2f - g = 2

    2g + 2f  = 20

If f and g satisfy the system of equations above, what is the value of f?

3. q + 4r = 21

    2r - q = 9

If q and r satisfy the system of equations above, what are the values of both q and r?

Answers to Practice Questions edit

1. 9

5a - 3b = 21

2a + b = 15 Take the second equation.

b = 15 - 2a Convert this equation to express one variable in terms of the other. In this case, subtract 2a from both sides.

5a - 3(15 - 2a) = 21 Substitute this value into the first equation.

5a - 45 + 6a = 21 Expand the parentheses.

11a - 45 = 21 Consolidate the variables.

11a = 66 Add 45 to both sides.

a = 6 Divide both sides by 11. a is 6, b is 3, and a + b = 9

2. 4

2g + 2f = 20 Take the second equation.

2g = 20 - 2f Convert this equation to express one variable in terms of the other. In this case, subtract 2f from both sides.

g = 10 - f Divide both sides by 2.

2f - (10 - f) Substitute this value into the first equation.

2f - 10 + f= 2 Expand the parentheses.

3f - 10 = 2 Consolidate the variables.

3f = 12 Add 10 to both sides.

f = 4 Divide both sides by 3. f is 4.

3. 5, 1

2r - q = 9 Take the second equation.

-q = 9 - 2r Convert this equation to express one variable in terms of the other. In this case, subtract 2r from both sides.

q = 2r - 9 Multiply both sides by -2

(2r - 9) + 4r = 21 Substitute this value into the first equation.

2r - 9 + 4r = 21 Expand the parentheses.

6r - 9 = 21 Combine the variables.

6r = 30 Add 9 to both sides. r is 5; q 1.


Averaging with Variables

Averaging with Variables edit

If an average consists of a word problem, place the known values and variables into the average formula accordingly.

In some cases, the proportions can be calculated even if the exact amounts are not known. Suppose the average trips to the grocery store per month is 5 in Scranton but 7 in Dunmore. If the combined average of the two places is 5.5, what is the proportion of their populations? If the populations were equal, the average would be 6 - Scranton must have a greater population.


Let s and d equal the populations of Scranton and Dunmore, respectively.

5s + 7d  = 5.5	(s + d)		Multiply both sides by (s + d).
5s + 7d  = 5.5	s + 5.5d	Expand the parentheses.

5s + 1.5d = 5.5s Subtract 5.5d from both sides.

1.5d = .5s Subtract 5.5s from both sides.

3d = s Divide both sides by .5.

Thus, 3d = s; in other words, Scranton has 3 times the population of Dunmore.

Practice

1. Debbie sold an average of 44 fur coats per month at her fur coat store in 2011. If Debbie's fur coat sales in November and December were three times as high as in the rest of the months, how many fur coats did she sell, on average, in November and December?

2. The average lifespan of the flamingos on Island A is 15 years while the average lifespan of flamingos on Island B 17 years. The combined average lifespan on the islands is 15.8 years. If 1,000 flamingos live on Island B, how many flamingos live on Island A?

3. The average height of the 12 office buildings in New Hartford is 32 feet. If the Union Bank building is excluded, the average height of the office buildings is 29 feet. How tall is the Union Bank building?


Answers to Practice Questions

1. 33

Let s equal Debbie's average fur coat sales in November and December and r equal her average fur coat sales for the other 10 months.

= 44 Take the initial equation.

= 44 Substitute the known value for s in terms of r.

= 44 Expand the parentheses.

= 44 Consolidate the variables.

= 528 Multiply both sides by 12.

 r = 33				Divide both sides by 16. The average sales per month 				in the 10 months outside of November and December 				is 33.

Since the regular rate is 33, the average rate for November and December is 99 - three times this amount.


 Answers to Practice Questions

2. 1500

Let a equal the flamingo population on island a.

= 15.8 Take the initial equation.

1,000(17) + a(15) = 15.8(1,000 + a) Multiply both sides by (1,000 + a).

17,000 + 15a = 15,800 + 15.8a Expand the parentheses.

1200 + 15a = 15.8a Subtract 15,800 from both sides.

1200 = .8a Subtract 15a from both sides.

1500 = a Divide both sides by .8. The flamingo population is 1500 on Island A.

Answers to Practice Questions

3. 65 feet

Let u equal the height of the union bank building.

= 32 Take the initial equation.

11(29) + u = 384 Multiply both sides by 12.

319 + u = 384 Work out the parentheses.

u = 65 Subtract 319 from both sides. The building is 65 feet tall.


Quadratic Equations

Quadratic Equations edit

A quadratic equation is an equation such as x2 - 3x = 2, where x is multiplied by both itself and a constant.

Rule edit

To solve an equation such as “x2 + 2x = -1”, adjust the equation so that a zero is on one side, then break the equation into factors such as (x + 1)(x + 1) = 0. The correct answer is the opposite of the numbers in the parentheses.

x2 - 3x = -3

Start with the initial equation.

x2 -3x + 3 = 0

Add 3 to both sides to set the equation equal to zero.

(x + ?)(x + ?) = 0

Break the equation into two “roots” or “factors.”

(x - 3)(x - 1) = 0

Find the two numbers which add up to the second number (the -3) and multiply to the third number (the 3). These will be the numbers in the parentheses, that multiply to x in the original equation.

x = 3, 1

Most quadratic equations have two answers because they have different numbers in each factor. Some, however, like (x + 1)(x + 1), only have one answer.

Difference of Two Squares edit

x2 - y2 factors out to (x - y)(x + y). This is known as the rule of “difference of two squares.”

Thus, x2 - 9 would thus factor out to (x - 3)(x + 3). This is a special rule that is often tested on the GRE.

Factoring edit

“Factoring” is one of the most difficult parts of solving a quadratic equation. x2 + 6x + 9 = 0 will factor into (x + 3)(x + 3), because if multiplied out, (x + 3)(x + 3) equals the prior equation. The numbers in the parentheses will add up to second number in the equation x2 + 6x + 9 = 0 and multiply up to the third number. Observe more examples:

Original equation Set it equal to zero Factor Solution

x2 + 4 = -4x x2 + 4x + 4 = 0 (x + 2)(x + 2) x = -2

x2 -4x = -3 x2 - 4x + 3 = 0 (x - 3)(x - 1) x = 3, 1

x2 - 1 = x x2 - x - 2 = 0 (x - 2)(x + 1) x = 2, -1

(find two numbers that add to the middle number and multiply to the third number)

Practice edit

1. If x2 + 5 = -6x, then what is the value of x?

2. x2 - 2x = 8. Solve for x.

3. If x2 + 4x + 2 = -1, then what does x equal?

Answers to Practice Questions edit

1. -5, -1

x2 + 5 = -6x Take the initial equation.

x2 + 6x + 5 = 0 Add 6x to both sides to set the equation equal to zero. Make sure that the equation is in the form x2 + bx + c = 0.

(x + ?)(x + ?) Break the equation into factors. What two numbers add up to the middle number (6) and multiply to the last number (5)?

(x + 5)(x + 1) = 0 Plug the correct numbers into the equation.

x = -5, -1 x is the opposite of these numbers.

2. 4, -2

x2 - 2x = 8 Take the initial equation.

x -2x - 8 = 0 Subtract 8 from both sides to set the equation equal to zero. Make sure that the equation is in the form x2 + bx + c = 0.

(x + ?)(x + ?) Break the equation into factors. What two numbers add up to the middle number (-2) and multiply to the last number (-8)?

(x - 4)(x + 2) = 0 Plug the correct numbers into the equation.

x = 4, -2 x is the opposite of these numbers.

3. -3, -1

x2 + 4x + 2 = -1 Take the initial equation.

x2 + 4x + 3 = 0 Add 1 to both sides to set the equation equal to zero. Make sure that the equation is in the form x2 + bx + c = 0.

(x + ?)(x + ?) Break the equation into factors. What two numbers add up to the middle number (4) and multiply to the last number (3)?

(x + 3)(x + 1) = 0 Plug the correct numbers into the equation.

x = -3, -1 x is the opposite of these numbers.

Some quadratic equations cannot be factored and must be solved using a special formula, the “quadratic formula.” This formula is not tested on the GRE.


Probabilities

Probabilities edit

The GRE frequently tests probability, which is the odds of a particular condition occurring.

Rules edit

The probability of an event happening MORE THAN ONCE is equal to:

(odds the 1st time)(odds the second 2nd)(etc.)

The probability of an event happening AT LEAST ONCE over many tries is:

1 - (odds of failure the first time)(odds of failure the second time)(etc.)

Probability is the likelihood of an event occurring. It is generally calculated as the number of desired outcomes desired by the total number of outcomes. Probability is sometimes expressed as a fraction, or as a decimal. When expressed as a decimal, a probability of “1” equals 100% chance, while a probability of .5 equals a 1 in 2 chance.

For example, when rolling a 6-sided die with faces numbered 1 to 6, the probability of rolling a 6 is 1 in 6, or 1.6666 repeating. (Assuming that the die is not imbalanced).

For the odds of rolling two 6s in a row on a 6-sided die, the formula is (odds the first time)(odds the second time).

The odds of rolling a six at least once over 2 tries on a 6-sided die. The formula is 1 - (combined odds of failure).

Practice edit

1. Brian is sending out job applications to law firms. His odds of being hired at Firm A are 1 in 3; his odds at Firm B are 1 in 5, and his odds at Firm C are 1 in 7. What are the odds that Brian is hired by at least one of the law firms?

2. Melanie is shooting baskets at the gym. She has a 1 in 3 chance of making a three-point shot. What are the odds of Melanie making 2 shots in a row?

3. Darren has a 90% chance of being late for class each day. What are the odds that he shows up on time three times in a row?

Comments edit

The distinction between the odds of something happening "every time" versus "at least once" is sometimes known as "straight" versus "cumulative" probability.

Probability can be one of the most complex areas of mathematics; however, GRE probability is usually relatively simple. The questions the GRE asks with regard to probability can almost always be solved with the above equations.

Answers to Practice Questions edit

1. 57 out of 105

Since this is the odds of something happening at least once over multiple tries, the formula is 1 - (odds of failure the first time)(odds of failure the second time)(etc.).

The odds of success for Brian's applications are  ,   and  . Thus, the odds of failure for Brian's applications are  ,  , and  .

Brian's odds of being hired by at least one of the firms are thus (1 - odds of failure), which works out to (1 -  ( )( ) or  

2. 1 out of 9

Since this is the odds of something happening more than once in a row, the formula is (odds the first time)(odds the second time)(etc.). The odds are:  ( )

=  

3. 1 in 1000

Since this is the odds of something happening more than once in a row, the formula is (odds the first time)(odds the second time)(etc.). The odds are:  ( )( )

=  


Arrangements

texvc

Arrangements edit

A permutation is an arrangement where the order is important. In this case, the number of possible arrangements of elements is (possibilities for spot 1)(possibilities for spot 2)... This can be represented by a formula as follows:

 .

  •   is notation expressing a number of permutations of n items, taking k items at a time.
  •  , read as 'x factorial', signifies the product of all the natural numbers from 1 to  . For example, 10! = 1(2)(3)(4)(5)(6)(7)(8)(9)(10).

A combination is an arrangement where the order does not matter. The combination formula is the same as the permutation formula, except everything is divided by  , i.e. the number of spots factorial:

 .

Suppose Kathryn is packing tie-dye shirts to take on vacation, and will pick 3 out of her 27 tie-dye shirts to take with her. How many assortments of shirts can she bring? Order is not important in this question - selecting shirt 3, shirt 5, and shirt 7 is the same as picking shirt 7, shirt 3, and shirt 5. Thus the number is:

 

Thus, there are 2,925 possible shirt arrangements.

If order was important above - suppose Kathryn was picking a specific shirt for the first, second, and third days - the formula would not be divided by 3!. It would simply be (spot 1)(spot 2)(spot 3), or (27)(26)(25), and would yield 17,550 possibilities.

Note that the number of possibilities descends (27, 26, 25) because the shirts can't be reused. If Kathryn was picking tie-dye shirts to wear on the next three Mondays, and had an opportunity to wash and reuse the same one, the formula would be (27)(27)(27).

Practice edit

1. A pizza parlor has 8 different toppings available. How many different pizzas with 3 toppings can be made?

2. Lee has 4 different colors of paint to use on his model airplane. He will paint the body one color, the tail rudder a different color, and the nose a different color yet. How many different ways could Lee paint his airplane?

3. A soup recipe calls for any 3 of a list of 6 herbs. How many different groups of 3 herbs can be put in the soup?

Answers to Practice Questions edit

1. 56

On any arrangement question, ask two questions - is order important, and, can the elements be reused?

Order is not important - pepperoni, sausage and olives is the same as olives, sausage and pepperoni. The elements can not be reused - a pizza can't have sausage, sausage, and sausage as its toppings. It either has sausage or it doesn't. Thus, the formula is:

== 56

2. 64

Order is important - a red nose and a blue rudder is different than a blue nose and a red rudder. The colors can be reused - each of four is available. The formula is thus:

4(4)(4) = 64

3. A soup recipe calls for any 3 of a list of 6 herbs. How many different groups of 3 herbs can be put in the soup?

Order is not important - thyme, parsley, and oregano is the same as oregano, parsley, and thyme in the soup. The elements cannot be reused - the soup must have 3 of the 6 herbs. The formula is thus:

== 20


Digits of Numbers

Digits of Numbers edit

If a question gives you the relationship between the various digits of a number, substitute variables for these digits and put the appropriate power of 10 in front of each one.

For example, the test might ask, “in a number, the hundreds digit is 2 greater than the tens digit, which is 2 greater than the units digit. If the number is equal to 195 multiplied by the units digit, what is the number?'

  • Let u equal the units digit, t the tens, h the hundreds, and x the number.
  • Restate the givens in terms of u, t, h, and x.
    • t = u + 2
    • h = t + 2 = u + 4
    • x = 195u
    • x = 100h + 10t + u
  • Find the value of u.
    • 195u = x = 100h + 10t + u
    • 194u = 100h + 10t
    • 194u = 100(u + 4) + 10(u + 2)
    • 194u = 100u + 10u + 420 = 110u + 420
    • 84u = 420
    • u = 420/84 = 5
  • Now you can find the values of the other numbers:
    • t = 5 + 2 = 7
    • h = 5 + 4 = 9
  • Check your work:
    • 975 = 195 * 5


Practice edit

1. The tens digit of a 3-digit number is twice the units digit and one less than the hundreds digit. If the number is equal to two less than 136 times the tens digit, what is the number?

2. If the digits of a 2-digit number are reversed, the resulting number is 45 greater than the original number. If the initial 2-digit number is less than 20, then what is its value?


Answers to Practice Questions edit

1. 542

Let t equal the tens digit.

100(t + 1) + 10t + t/2 = 136t - 2

Take the original equation.

100t + 100 + 10t + t/2 = 136t - 2

Expand the parentheses.

110.5t + 100 = 136t - 2

Combine variables and constants.

100 = 25.5t - 2

Subtract from both sides.

102 = 25.5t

Add 2 to both sides.

4 = t

Divide both sides by 25.5
t is equal to 4, and the number is thus equal to 542.

2. 16

Let t equal the tens digit and u equal the units digit.

10u + t - (10t + u) = 45 Take the equation.

10u + t - 10t - u = 45 Expand the parentheses.

9u - 9t = 45 Combine the variables.

u - t = 5 Divide both sides by 9.

u is 5 greater than t; thus, u and t could be 1 and 6, 2 and 7, 3 and 8, 4 and 9. However, since the initial value is less than 20, the digits must be 1 and 6, which make 16 and 64.


Lines and Angles

Lines & Angles edit

Opposite angles are equal. Adjacent angles on a line add up to 180º.

  • Angles n and p are equal.
  • Angles q and o are equal.
  • Angles n and o add up to 180º.
  • Angles o and p add up to 180º.
  • Angles p and q add up to 180º.
  • Angles q and n add up to 180º.

A line intersecting two parallel lines forms the same angles with each.

Parallel lines are those which are a constant distance apart. A line intersecting them is known as a “transversal.”

Assume both sets of lines are parallel; therefore:

  • Angles f and g are equal.
  • Angles h and i add up to 180º.
  • Angles f and h are equal.
  • Angles g and i add up to 180º.

Practice edit

1. In this figure, both pairs of lines are parallel, and w is 10º greater than u. What are the measures of angles t, u, v and w?

Answers to Practice Questions edit

1. t, u, v, and w have measures of 95º, 95º, 85º, and 95º, respectively.

Because lines make the same angles through any parallel lines they intersect, angle w is identical to angle v as well as the angle directly opposite t. Because opposite angles are equal, angles t and w are equal.

All three of these angles are equal to either of the angles adjacent to angle u along the line (either the line running from t to u or from v to u); thus u + v = 180º. Since w = u + 10º, we can solve for u.

u + v = 180º Take the initial equation.

u + (u + 10º) = 180º Substitute for v in terms of u.

2u + 10º = 180º Add the variables.

2u = 170º Subtract 10º from both sides.

u = 85º Divide both sides by 2. u is 85º, making the other three angles 95º.


Triangles

Triangles edit

The angles of a triangle add up to 180º.

In this triangle, c is equal to 60 because the angles of a triangle add up to 180º.

The greater the angle in a triangle, the greater the corresponding side.

Angle f is greater than angle g but less than angle h, because the corresponding sides are larger and smaller, respectively.

The Four Types of Triangles

Practice edit

1.

If FG is equal to GH but less than FH, then the measure of angle FGH must at least be greater than how many degrees?

2.

If i is 8 degrees greater than k and 4 degrees greater than j, what is the measure of i?

3.

If LM and MN both have lengths of 10, and angle LMN has a measure of 80º, what are the measures of the other two angles of the triangle, MLN and LNM?


Answers to Practice Questions edit

1. The angle measure must be greater than 60º.

The angles of a triangle add up to 180º. The greater the length of a side of a triangle, the greater its opposite angle.

If FH is greater than the other two sides, its angle measure must be greater. If all three angles were equal, they would equal 60º (180º divided by 3), but since this side is greater, its opposing angle must be greater than 60º, while the other two must be less than 60º.

2. 64º

The angles of a triangle add up to 180º. Thus, i + j + k = 180.

i + j + k = 180. Take the initial equation.

i + (i - 4) + (i - 8) = 180 Substitute the known values of the other two variables in terms of i.

i + i -4 + i - 8 = 180 Expand the parentheses.

3i - 12 = 180 Consolidate the constants and variables.

3i = 192 Add 12 to both sides.

i = 64 Divide both sides by 3. i is equal to 64.

3. 50º

Since LM and MN have identical lengths, the angles opposite these sides have identical lengths. The angles of a triangle add up to 180º; thus, since the third angle is equal to 80º, the other two angles must have a combined measure of 100º. Since they are equal, the measure of each is 50º.


Pythagorean Theorem

The Pythagorean Theorem edit

In a right triangle with sides a, b and c, a2 + b2 = c2, c being the longest side.

In this triangle, a2 + b2 = c2 and thus:

a =

b =

c =

There are a limited number of “Pythagorean Triples” or groups of integers which fit the formula.

The GRE tends to have questions which work out to integers; thus, keep an eye out for the “3-4-5” triangle and the “5-12-13” triangle.

The 3-4-5 Triangle

The squares of 3 and 4 add up to the square of 5, thus, the “3-4-5” triangle is common on the GRE. It may also be in the form of a “6-8-10” triangle or a “9-12-15” triangle, or another triangle similarly increased.

The 5-12-13 Triangle

In a similar manner the 3-4-5 triangle, this triangle has a series of integers which comport with the Pythagorean Theorem. This triple may also be increased to 10-24-26 and so on.

There are two special right triangles, the proportions of which can be inferred by their angle measurements.

A 45º-45º-90º triangle has sides in the ratio of 1 - 1 -

Thus if the shorter sides are 10, the longer side is

A 30º-60º-90º triangle has sides in the ratio of 1 -- 2.

Thus if the shortest side has a length of 5, the two other sides have lengths ofand 10.

Indeed, the proportions of every right triangle can be determined by the measurement of one of its angles based on this theorem. Mathematicians and engineers have developed tables of the proportions for these angles and have designated functions for them, such as sin and cos. This field of mathematics is known as “trigonometry” and is not tested on the GRE.

Practice edit

1.

What is the value of q?

2.

Solve for r.

3.

Determine the value of s.

Answers to Practice Questions edit

1.

In a right triangle with sides a, b and c, a2 + b2 = c2, where c is the longest side. Thus, q is equal to or

2.

In a right triangle with two equal sides, the longer side equals the shorter side multiplied by Thus, the shorter side equals the longer side multiplied by This means r equals 8 times this proportion, which equals or

3. 24

In a right triangle with sides a, b and c, a2 + b2 = c2, where c is the longest side. Thus, s equals which equals or 24. This is an example of the “Pythagorean Triple” of 5-12-13, in this case multiplied by 2.


Areas of Triangles

Areas of Triangles edit

The area of a triangle equals half of the base multiplied by the height.

In a right triangle, the base and the height will be the two smallest sides.

The area of this triangle equals (6)(8) divided by 2, or 24, because in a right triangle the base and the height are the two smaller sides

Otherwise, however, the height of the triangle will have to be known or deduced.

The height of an isosceles triangle can be deduced using the Pythagorean Theorem.

Take the original triangle. Split the base in half. The height will be perpendicular.

Under the Pythagorean Theorem:

The larger side squared equals the sum of the other sides squared.

Expand the exponents.

Subtract 25 from both sides.

Take the square root of both sides. The triangle has a height of 12 and thus an area of 30.

Practice edit

1.

Multiply the length time the width (5in x 7in = 12in).

2.

Take that answer and put the measurement at the end of the answer like (inches, centimeters, or millimeters). 3.

Then you have the area (your answer to the problem).

Answers to Practice Questions edit

1.

The base of an isosceles triangle can be split in half to form right triangles and calculate the height. In this case, the height equals or The area of the triangle is half of the base multiplied by this amount, or

2.

The area of a right triangle equals half of the product of its two smaller sides. The first of these sides equals t; the second must calculated using the Pythagorean Theorem as The area is thus

3.

This can be solved using the same procedure for isosceles triangles used in the first question. However, there is an even simpler way. An equilateral triangle forms 30º-60º-90º triangles when split down the middle. The sides of a 30º-60º-90º have the proportions 1 -- 2. Thus, the height of this triangle equals and its area equals or


Quadrilaterals

Quadrilaterals edit

The angles of a quadrilateral add up to 360º. There are 6 key types:

Square Rectangle
Four equal, 90º angles
Four equal sides
Rhombus Parallelogram
Opposing sides are parallel
Diagonal angles are equal
Adjacent angles add up to 180º
All four sides are equal

Trapezoid

  • One set of parallel sides

Kite

  • Two sets of adjacent equal sides


Squares

Squares edit

The area of a square equals its side squared.

Area = s2

The diagonal of a square equals its side multiplied by  

This comes from the Pythagorean theorem. The diagonal is the greatest distance between any two points on the square.

Practice edit

1. The diagonal of a square is equal to 5. What is the length of a side of the square?

2. Square A has 3 times the size of Square B. If Square B's side has a length of 10, what is the length of a side of Square A?


Answers to Practice Questions edit

1.

The diagonal of a square is equal tomultiplied by its side; thus, the side of a square is equal to its diagonal divided by, or multiplied by In the case of this particular square, this means that the diagonal is equal to

2.

The area of a square is its side squared. Thus if a2 = 3b2 then a must equalSince the length of one side of the latter square is 10, the first square's side must equal


Rectangles

Rectangles edit

The area of a rectangle equals length multiplied by width.

Area = 450

Rectangle questions usually involve algebra and a quadratic equation.

For example, if the length of a rectangle is twice its width, and the rectangle's area is 98, what is the width of the rectangle?

Let w equal width and l equal length.

w(l) = 98 Set up the formula.

w(2w) = 98 Substitute 2l for

  Expand the parentheses.

  Divide both sides by 2.

w = 7 Take the square root of both sides.

The width of the rectangle is 7 (and the length is 14).

Practice edit

1. A rectangle has an area of 132 and its length is 1 greater than its width. What are the dimensions of the rectangle?

2. A rectangle's area would increase by 90 if its length were extended by 18. What is the rectangle's width?


Answers to Practice Questions edit

1. 11 and 12

The formula for a rectangle's area is length multiplied by width. Thus, this problem can be solved with algebra and factoring. Let l equal the length and w equal the width.

  Take the initial equation.

  Substitute the length in terms of the width.

  Expand the parentheses.

  Subtract 132 from both sides.

  Break the expression into factors. What two numbers multiply to -132 and add to 1?

  Factor the equation. w equals 11 or -12. Since the width is not negative, it equals 11.

Using the width, the length can be easily determined by adding 1.

2. 5

Since area of a rectangle is length multiplied by width, the width equals the amount of extension increase - 90 - divided by the increase in length, which is 18. The width is thus 5.


Trapezoids

Trapezoids edit

Rules edit

The area of a trapezoid equals the average of the bases divided by the height.

The area equalsor 72.

When not given, the height of a trapezoid can often be deduced using the Pythagorean Theorem

In this figure, we split the trapezoid into a right triangle with a leg of 3 (10 - 5 -2) as seen to the right. The height of the triangle is 52 minus 32, which is equal to 4.

Practice edit

1. If the trapezoid below has an area of 22, what is the length of its top side?

2. What is the area of the trapezoid above?


Comments edit

Answers to Practice Questions edit

1. 4

The area of a trapezoid equals the average of its bases multiplied by its height. Since the height of this trapezoid is 4, the average of its bases must equal 22 divided by 4, or 5.5. At this point, the top side can be determine by solving the equation= 5.5, which works out to 4.

2. 192

This trapezoid can be split into a right triangle with sides of 20 and 16, the latter determined by subtracting 8 from 24. This means that the height of the trapezoid is the third side of the right triangle, which equals or 12. 12 multiplied by the average of the bases - 16 - equals 192.


Rhombuses

Rhombuses edit

Rules edit

The area of a rhombus or parallelogram is base multiplied by height.

The area of a rhombus is the product of its diagonals, which are perpendicular.

In this figure, the area is 8(6) or 48.

The area of the figure to the left is 10(12) or 120.

In a rhombus or parallelogram, opposite angles are equal, and adjacent angles add up to 180º.

This follows from the rules of parallel lines and opposing and adjacent angles.

Practice edit

1. In the rhombus to the right, the distance from F to H is 10. If the area of the rhombus is 85, what is the distance from G to I

2. A rhombus has a perimeter of 60 and a diagonal of 18. What is its area?

Comments edit

Answers to Practice Questions edit

1. 8.5

The area of a rhombus is equal to the product of its diagonals. Since this rhombus has an area of 85 and a diagonal of 10, the second diagonal is equal toor 8.5.

2. 432

The diagonals of a rhombus are perpendicular, and its four sides are equal. Thus, a rhombus with a perimeter of 60 has sides of 15; if this rhombus has a diagonal of 18, it can be broken into right triangles. The third side of these triangles is equal to or 12. Thus, the diagonal is equal to 24 and thus its area is 432.


Kites

Kites edit

Rules edit

The diagonals of a kite are perpendicular, and its area is the product of these diagonals.

A kite has two pairs of adjacent equal sides.

Its diagonals form right angles, which, if multiplied, yield the area of the kite. Because the diagonals are perpendicular, the perimeter of a kite can be determined using the Pythagorean Theorem.

Practice edit

1. In figure JKLM, sides JK and KL are equal, and sides LM and MJ are equal. The distance from K to M is twice the distance from J to L. If the area of JKLM is 50, what is the distance from K to M?

2. The lengths of both NO and NQ are 52, while the lengths of both OP and PQ are 25. If the distance from O to Q is 40, what is the distance from N to P?

Comments edit

Answers to Practice Questions edit

1. 10

The area of a kite equals the product of its diagonals. Since the area of this kite is 50 and one diagonal is half the length of the other, the lengths can be determined using algebra. Let d equal the smaller diagonal.

d(2d) = 50 Take the initial equation.

2d2 = 50 Expand the parentheses.

d2 = 25 Divide both sides by 2.

d = 5 Take the square root of both sides. The smaller diagonal is equal to 5, making the larger one 10.

2. 63

The kite can be split into four different right triangles. The missing sides when combined form the unknown diagonal, which pursuant to the Pythagorean Theorem can be calculated as +, or 48 + 15, which equals 63.


Circles

Circles edit

Rules edit

The area of a circle is π multiplied by the radius squared. The circumference of a circle is π multiplied by the diameter (twice the radius).

A circle is a set of points of equal distance from a given point. The number π has been developed by mathematicians to express the ratio of the circumference of a circle to its diameter. π ≈ 3.14.

An arc within a circle is equal to the fraction of the circumference that the arc's angle is a fraction of 360º, or the entire circle.

In the circle to the left, arc AB has a length of one fourth of the circumference of the circle

Practice edit

1. A circle has an area of 1. What is its radius?

2. Square GHIJ is inscribed within the circle. If the square has an area of 64, what is the area of the circle?

3.

D is the center of the circle, and points E and F are both on the circle. If EF and DF are both equal to 5, what is the length of arc EF?

Comments edit

Answers to Practice Questions edit

1.

The area of a circle is multiplied by the radius squared.

= 1 Take the initial equation.

= Divide both sides by

= = = Take the square root of both sides.

2.

If the area of the square is 64, then the sides of the square are 8. If the sides of the square equal 8, the diagonal - and thus the diameter of the circle equals

3.

The triangle is an equilateral triangle, meaning its angles are 60º. This means the arc has a length of one sixth of the circumference of the circle, or which works out to


Other Shapes

Other Shapes edit

Rules edit

A polygon's angles add up to 180º plus an additional 180º for every side over 3.

The angles of a pentagon add up to 540º.

The angles of a enneakaidecagon add up to 3060º.

(An enneakaidecagon is a figure with 19 sides). Polygons other than triangles and quadrilaterals are rare on the GRE; when they do appear, they are typically shapes which employ a combination of triangles and quadrilaterals.

Practice edit

1. In the figure to the left, angles a, d, h, and e are each equal to half of the measures of each of angles b, c, g, and f. What is the measure of angle a?

2. How many degrees do the angles of a 102-sided figure add up to?

3. The angles of a particular polygon add up to 2340º. How many sides does the polygon have?

Comments edit

Answers to Practice Questions edit

1. 90º

The angles of an octagon add up to 1080º. Since a, d, h, and e are equal and are each half of the b, c, g, or f, then it can be inferred that 4(a) + 4(2a) = 1080º. This means that 12a = 1080º and that a is equal to 90º.

2. 18,000º

A polygon has 180º plus an additional 180º for every side over 3. Thus, the angles of a 102-sided figure equal (102-2)180º or 18,000º.

3. 15

A polygon has 180º plus an additional 180º for every side over 3. Thus, the question can be solved using algebra. Let n equal the number of sides of the polygon.

(n -2)180º = 2340º Take the initial equation.

180ºn - 360º = 2340º Expand the parentheses.

180ºn = 2700º Add 360º to both sides.

n = 15 Divide both sides by 180º.

The polygon has 15 sides.


3-Dimensional Shapes

3-Dimensional Shapes edit

Rules edit

The volume of a non-tapering 3-dimensional object is the area of the base multiplied by the height.

Irregular objects and those which “taper” - or are not the same from the bottom to the top - are typically not tested on the GRE. However, for block-shaped objects, the formula is simply length(width)(height). However, there are two key tapering objects which do have simple formulas and could be tested - cones, and spheres.

The volume of a cone is one third of the area of the base multiplied by the height.

The area of the base will utilize the same formula for a circle.

The volume of a sphere is  . A “sphere” is a round three-dimensional object, such as a ball or a globe.

Practice edit

1. A cereal box (left) has a volume of 225. If the dimensions of the face of the cereal box are 18 and 5, what is the depth of the cereal box?

2. A cylinder (right) has a diameter of 22 and a height of 45. What is the volume of the cylinder?

3. A soccer ball has a diameter of 12. A tennis ball has a diameter of 2. The volume of the tennis ball is what percentage of the volume of the soccer ball?

Comments edit

Answers to Practice Questions edit

1. 2.5

The volume of a block-shaped object is equal to length(width)(height). The length and width are known to be 18 and 5, and the volume is 225. Thus, 18(5)(width) = 225. The depth equals 225 divided by 18(5) or 2.5.

2.

The volume of a cylinder equals the area of the base multiplied by the height. Because the diameter of the cylinder is 22, the radius is 11 and the area of the base isThis means that the volume isor

3.

The volume of a sphere is Thus, the volume of the soccer ball is or ; the volume of the tennis ball isorThe ratio of their volumes is or


Coordinates and Graphing

Coordinates & Graphing edit

Rules edit

In a standard coordinate plane, points are given locations of (x, y), where “x” is the horizontal coordinate and y is the vertical coordinate.

The x coordinate is also called the “abscissa”, while the y coordinate is called the “ordinate.”

The slope of a line graphed on the coordinate plane is equal to rise divided by run. If a line is expressed by an equation, the slope of the line is the number in front of the x when the equation is in the form y = ax + b.

For example, if the equation for a line is y = 2x + 1, the slope is 2 and the y intercept is 1. If the line is in the form y - +2x = 1, rearrange it so that it is in the above format - y = mx + b.

To determine when a line crosses the x or y axis, set x or y equal to zero.

For example, a line with the equation y = 6x - 5 crosses the y axis at (0, -5).

Practice edit

1. The equation for a line is 2y - 4x - 5 = 0. What is the slope of the line?

2. What is the slope of a line which passes through points (3, -2) and (6,3)?

Comments edit

What is the equation for the line above?

Answers to Practice Questions edit

1. 2

When a line is in the form y = mx + b, m is the slope of the line and b is the y intercept. If a line is not in this format, it should be converted into this format.

2y - 4x - 5 = 0 Take the initial equation.

2y - 4x = 5 Add 5 to both sides.

2y = 4x + 5 Add 4x to both sides.

y = 2x + Divide both sides by 2. The slope of the line is 2.

2.

The slope of this line is rise over run, or This equals or

3. y = 3x + 1

When a line is in the form y = mx + b, m is the slope of the line and b is the y intercept.

The line crosses the y axis at (0,1) and thus its intercept is 1. It passes through the point (1,4) after passing through (0,1) meaning it has a rise of 3 for every unit of run. Thus, the slope of the line is 3 and its equation is y = 3x + 1.


Standard Deviation

Standard Deviation edit

Standard deviation measures how much a given set of numbers varies. It does not in any way reflect the average value of data or the quantity of the elements.

Thus, the numbers 1, 700, 62,000 and 1,000,000 would have a greater standard deviation than the numbers, 4, 5, 6, and 7.

The GRE does not test the formula for standard deviation nor does it expect test takers to remember “normal” distribution. It does, however, expect test takers to understand the concept. The test will frequently ask trick questions hoping that test takers will confuse standard deviation with mean or median value or range, such as those below for you to practice on.

Practice edit

1. Data Set 1 has a range of 1-100. Data Set 2 has a range of 200-500. Is it possible to determine which set has a greater standard deviation?

2.

The chart above shows a normal distribution for hours spent watching television per week in Springfield, with a mean of 8 and a standard deviation of 6. What percentage of the population of Springfield watches fewer than 2 hours of television per week?

3.

Series I Series II
5, 6, 6, 7, 8, 10 1, 2, 3, 7, 15, 24



Which series has a greater standard deviation?

Comments edit

Answers to Practice Questions edit

1. No, the difference in standard deviation cannot be determined

Standard deviation is a tool to measure how varied the results are from a given mean. It has nothing to do with the value of the numbers or the quantity of numbers in the set, except to the extent that these influence average distance from the mean.

2. 16%

Since the mean is 8 and the standard deviation is 6, 84% of the data sample will fall above 8-2, pursuant to the chart. The cutoff is represented by the “m - d” on the chart.

3. Set II has a greater standard deviation.

Standard deviation measures how close or far apart a set of numbers are from the mean. Set II is clearly more spread apart than Set I, thus, it has more standard deviation.

Again, the GRE does not expect test takers to know the standard deviation formula but it expects them to understand the concept.


Bar Charts

Bar Charts edit

A bar chart uses vertical bars to compare quantities, which are measured using a vertical axis.

For example, the chart below compares computer repair calls from three office buildings over periods of three months. Every bar graph has a “ruler” or scale on the left or right-hand side (usually the left). Typically the questions involve identifying values from within the chart and manipulating them.

Practice edit

1. Approximately how many more repair calls were made to 230 Park Place in 2009 than at 1215 Main Street?

2. By approximately how much did repairs increase at 16 General Pershing Street increase from the first quarter to the second quarter?

Comments edit

Answers to Practice Questions edit

1. About 55

230 Park Place has just over 20 repair calls in the first quarter and between 15 and 20 in the second and third quarters, and just under 15 in the fourth quarter. This yields a total of 70. 1215 Main Street has about 5 in both of the first two quarters and just under 5 for the last two, giving it a total of about 15.

2. About 5

In the first quarter, the repairs at 16 General Pershing are just over 10, while in the second quarter, they are between 15 and 20. This is an increase of about 5.


Pie Charts

Pie Charts edit

Rules edit

“Pie charts” indicate fractions of a total. The larger the “slice” the greater the fraction.

The chart below, for example, shows the fraction of flights taken to Buffalo as opposed to Syracuse, Albany, and New York City.

Chart-reading is usually fairly easy. Most question trip-ups involve manipulating the data in the chart.

Practice edit

1. Approximately what percentage of the flights taken by LJ Air are bound for Albany, Buffalo, and Syracuse, respectively?

2. How many more flights would have to be taken to New York City for such flights to make up 60% of the total flights?

Comments edit

Answers to Practice Questions edit

1. Albany, Buffalo, and Syracuse, respectively, make up roughly 27%, 13%, and 11% of flight departures. The numbers above in the chart are not percentages but rather absolute numbers. There are a total of 93 flights, meaning the percentages are the flights to these three cities - 25, 12, and 10 - respectively.

2. There are a total of 93 flights of which 46 are currently bound for New York City. The amount of increase necessary to raise this proportion to 60% can be solved using algebra. Let i equal the necessary increase to raise it to 60%.

= .6 Take the initial equation.

 i + 46 = .6(i + 93)		Multiply both sides by (i + 93).
 i + 46 = .6i + 55.8		Expand the parentheses.
 i = .6i + 9.8			Subtract 46 from both sides.

.4i = 9.8 Subtract .6i from both sides.

 i = 24.5			Divide both sides by .4.

Approximately 25 flights are needed to increase the percentage to 60%.


Cell Charts

Cell Charts edit

Description edit

Cell charts indicate corresponding data of particular categories. To obtain totals or combinations, add the appropriate cells.

Often the cells will not give the total amount and the test taker will be asked to determine this. Pay attention to what data the cell is giving with regard to what data category.

Practice edit

Car Washes and Laundromats in Five Cities edit

Population Car Washes Laundromats
Arlington 365,000 50 41
Dallas 1,198,000 84 90
Fort Worth 741,000 35 44
Irving 216,000 29 38
Plano 260,000 40 41

1. Which cities have the greatest and least number of car washes per capita?

2. Dallas makes up what percentage of the car washes and laundromats amongst the five cities?

Comments edit

Answers to Practice Questions edit

1. Per capita is calculated by the number of items divided by the number of people. It can thus be calculated and compared by dividing columns 3 and 4 by column 2 (or a reduced version of it). Plano by far has the highest number of car washes per person, while Dallas has the lowest. Irving has the highest number of laundromats per capita, while Dallas again has the lowest.

2. Dallas has 84 of the 238 car washes, or about 35%, and it has 90 of the 254 car washes, again, about 35%.


Scatterplot Charts

Scatterplot Charts edit

Description edit

Scatterplot and bubble charts display individual data as points on a field for the purpose of comparing them.

“Bubble” charts use bubbles that get larger as the quantity gets bigger, while scatterplots typically do not. These chart types are rare on the GRE but are worth mentioning.

Scatterplot charts indicate how elements of a sample are distributed based on the measurement notches on either side. To isolate elements within a particular range on a scatterplot chart, follow the appropriate measuring notch.

Practice edit

1. Which type of merchandise has the greatest profit margin?

2. Which products are sold the most? The least?


Comments edit

Answers to Practice Questions edit

1. Based on the chart, fax machines have the greatest profit margin. The y axis depicts profit margin, and shows fax machines closest to the top. Note that the x axis doesn't mean anything.

2. The bubbles correspond to percentage of sales; thus, printers appear to be sold the most, followed by fax machines. Scanners and copiers are sold less, in roughly equal portions.


Introduction to GRE Vocabulary

Introduction to GRE Vocabulary edit

The first eight questions of each verbal section, as well as four to five questions further on in the section, test sentence completion, which heavily draws upon vocabulary. The key to answering these questions is memorizing the words.

Questions 1-2 have a single blank.

1. Though she generally found tales of romance to be rather _____ and tiresome, Mary enjoyed reading The Wuthering Heights, especially for its 19th century English.

(A) corroborated (B) constrained (C) banal (D) brackish (E) pernicious

Questions 3-5 have two blanks.

3. While prepared to delegate some authority, the mayor was not about to give away _____ control over the city's housing authority budget to the unpredictable committee, especially while facing a _____ reelection year.

(A) poignant (D) redolent (B) unfettered (E) trifling (C) hapless (F) baleful

Questions 6-8 have three blanks.

6. The rise of Ottoman Power in the Mediterranean _____ the ability of western European powers to _____ trade with the Far East in the traditional manner, which would prove to _____ efforts to seek other trade routes utilizing the Atlantic.

(A) prevaricated (D) parlay (G) subdue (B) abrogated (E) effect (H) gratify (C) inundated (F) evince (I) impel

Questions 13-16 or so have one blank to be filled with two words.

16. Mendelev was able to demonstrate the _____ of his theory when he predicted the chemical properties of the missing chemicals in his table of the elements.

(A) contempt (B) obscurity (C) veracity (D) estrangement (E) latency (F) rectitude (G) derision


Vocabulary List 1

GRE Vocabulary List 1 edit

Laconic adj.) using few words; abrupt

Sherry was still confused about where her husband was going to store the new fishing boat after a rather laconic conversation about it.

Caparison (n.)(1) a decorative cloth that covers the back of a horse; (2) some type of ornamentation or decoration(v.) to decorate with a caparison

The caparison adorning the display had the standard blue, white and pink colors.

Propinquity (adj.) being close to a person, having a close bond or kinship

Mary and Michelle had developed a remarkable propinquity since being neighbors when they lived in Bremerton.

Belie (v.) to give a mistaken impression of something

Senator Gumbart's statements would prove to belie his true intentions.

Reproof (n.) criticism for some mistake or fault

The board of directors' constant reproof of Jenkins over the blown deal was too much to bear, and led to Jenkins' resignation.

Diffuse (v.) to spread out; to go from high to low concentration or pressure

Tsar Nicholas' statement that Russian mobilization would only be against Austria did little to diffuse Russo-German tensions.

Flounder (n.) a type of fish; (v.) to struggle or fail clumsily

Though not expecting remarkable success, Pedro did not foresee just how much his business model would flounder.

Effrontery (n.) insolent or rudely presumptuous behavior

While not pleased with the comment about her dress, Jennifer had come to expect such effrontery at a Long Island dinner party.

Intrepid (adj.) seeking challenge; adventurous

While Bobby had always been intrepid and outgoing, his little brother had few interests other than staying home and playing his PlayStation.

Officious (adj.) asserting authority excessively and annoyingly, especially in regard to trivial matters

The motel employee was rather officious, reprimanding the children for silly things like running too fast by the pool.

Recondite (adj.) not commonly known

It was a recondite fact amongst her friends from work that Kate was a devout Southern Baptist.

Ramify (v.)(1) to spread out by forming branches; (2) to have consequences

The decision not to expand into the lucrative handbag market would have great ramifications for the company in later years.

Resolute (adj.) determined, steadfast, passionate

Jack was resolute about reaching the top of Mount Everest, and would not be discouraged by the mishap at the Khumbu Icefall.

Synthesis (n.) the combining or mixing of multiple things to produce a new thing

The completed K288 rocket was Otto's masterpiece, the final result of a synthesis of his years of studying rocketry.

Runic (adj.)(1) of or relating to letters of an alphabet historically used by many Germanic peoples (the “runic” alphabet)(2) having mysterious or hidden meaning

Claire did not know what to make of the runic phrase.

Retrograde (v.)(1) to move backward; (2)to decline or deteriorate into a worse condition

Profits at Bernard's Restaurant would prove to retrograde sharply as patrons' demand for exquisite Occitane cuisine deteriorated.

Repudiate (v.)(1) deny the validity or truth of; (2) to refuse association with

By 1914, the Italian-Austrian alliance only existed on paper, and would soon be repudiated after the beginning of the war.

Obviate (v.)(1) to remove the need for a particular thing; (2) to reduce a risk or problem

Because it helped obviate weight concerns, the Wankel engine was ideal to use in the Mazda RX-8.

Ardent (adj.) passionate; having enthusiasm

Billy's ardent feelings for a woman halfway across the world still lingered in his mind.

Enervate (v.) to drain of energy; (n.) the state of having been enervated

While the Battle of Verdun would prove to severely enervate the French army, it would also inflict heavy casualties upon the German attackers.

Coercive (adj.) unduly influential or overcoming of free will

While he felt bad about being so coercive, Jamie thought it was important to talk his mother into getting a vacuum cleaner.

Specious (adj.) (1) misleadingly attractive; (2) wrong despite appearing to be plausible

Hoffman refused to accept the specious conclusion that all marine mammals were descended from ambulocetus, and explored the possibility of a separate lineage for walruses and seals.

Upbraid (v.) to reprimand or scold

Truman wasted no time in upbraiding his son over the incident with the fire extinguisher.

Dulcet (adj.) sweet or pleasant

Patty longed for another weekend up at Matt's cottage north of Council Bluffs, especially desiring some more of his delicious homemade espresso and dulcet rhubarb muffins.

Cajole (v.) to talk a person into something

Walter wanted a big truck, and was not going to let his nagging wife cajole him into buying a silly Yaris or Prius.

Disparate (adj.) significantly different or distinct

McLaren was astounded by the disparate track times before and after the aerodynamic modification.

Equivocal (adj.) of an uncertain meaning, attitude, nature or disposition

To Gabby, the statement "I am definitely interested" seemed quite equivocal as to whether or not the developer wanted to buy out her pizza parlor.

Refractory (adj.)(1) tending to bounce back heat or light; (2) stubborn or incapable of persuasion or management

The librarian was expectedly refractory with regard to requests to take the reference books out of the library and to the science building.

Plangent (adj.) having a loud reverberating sound

The plangent sound of the factory machines brought back memories of years on the assembly line.

Semblance (n.) appearance or form

While hazy, the painting contained some semblance of a person, maybe intending to represent a memory or regret.

Waft (v.) to float over through the air or with such ease as if it were floating through the air

The smell of the pumpkin pie wafted over into the garage where Frank was fixing the bicycle.

Derogatory (adj.) something which is insulting or offensive

Leandra found it quite derogatory for her boss to tell her that she looked like someone who once worked at Dairy Queen.

Plasticity (n.) the state of resembling plastic in some way, often by being easily molded, adapted, or by being fake or superficial

While she loved not having to have to drive to work, she loathed the plasticity of city culture, and missed life back in Ohio.

Circumspect (adj.) of or relating to an all-encompassing viewpoint or determination

While the assassination of the archduke was certainly the catalyst, in circumspect, it is difficult to pin down a single cause or instigator of the Great War.

Implication (n.) a necessary or likely truth, inference or consequence which is not expressly stated

The implication of the diagnosis was that Quentin was not going to be able to go bass fishing for a while.

Flamboyant (adj.) that which stands out for excess energy, uniqueness, or the like

While expecting to see flamboyant costumes, the couple had no idea what was in store for them at the party.

Inimitable (adj.) impossible to copy or duplicate; that which cannot be imitated

Charlotte's performance of the aria was inimitable, even by comparable sopranos.

Advocate (v.) to speak for or on behalf of a person, organization or idea; (n.) one who advocates

Walter was always a staunch advocate of having more interesting baselines for the tuba section to play.

Ubiquitous (adj.) that which is present or can be found everywhere or universally

While less present on the West Coast, Republicans were ubiquitous in Ashley's native Oklahoma.

Garrulous (adj.) extremely talkative, often with regard to unimportant things

The drive to Syracuse was rather unpleasant, mostly thanks to Kevin's garrulous sister and her longwinded tales from work.

Indifferent (adj.) lacking any concern or interest in a matter

While Uncle Hal was mostly indifferent to foreign affairs, he had watched a great deal of CNN since September 11.

Libertine (n.) one who acts without restraint or defies convention

Three years in San Diego had turned a once shy biology student into something of a libertine.

Misanthrope (n.)one who dislikes people or society

A decade at Deloitte had made Gwynneth into a severe misanthrope, and ultimately she decided to move off to Yellowstone.

Macerate (v.) to softly break up, often via immersion in liquid

Randy began to macerate the dried apples a few hours before dinner.

Vital (adj.) that which is of the utmost importance; that which is essential

Securing the support of the Turks proved vital to the German cause, as it allowed for a blockade of the Black Sea.

Artless (adj.) (1) lacking skill or taste; (2) natural and simple, lacking any artificiality

After living in Paris for so many years, Elaine found herself unable to stand her artless parents back in Michigan.

Recumbent (adj.) lying down; in a comfortable or resting position

Fanny was nestled up and recumbent, and took a long nap which lasted for the rest of the afternoon.

Umbrage (n.) (1) a state of being offended or annoyed; (2) shade or a shadow

Yolanda's umbrage at not being selected for the danceoff finals was all too apparent at dinner.

Extol (v.) to enthusiastically praise

Many of the people of Naples would continue to extol the Angevin monarchs for years to come.

Equanimity (n.) the state of having a stable mind and character

Steve displayed remarkable equanimity and composure despite how incredibly angry Miranda was at him.

Homogeneous (adj.) composed of all the same type, style or consistency

The food at Eve's diner was all very good, but somewhat homogenous, always being some variation of a sandwich or breakfast platter.

Vindicate (v.) (1) to prove a person or idea to be justified or correct after the fact; (2) to obtain revenge or payback for some wrong

The junior V.P.'s prediction would be vindicated once the October sales report was available.

Savant (n.) one of exceptional ability or knowledge

While all the Koontz children were good at math, Jackie was particularly a savant.

Pertain (v.) to be relevant or informative with regard to a given issue

The interests of scientists in the bird species primarily pertained to their confusing geographic distribution.

Corporal (adj.) of or relating to the person’s body

Though corporal punishment was legal in Nevada, it had seldom been used in the last century.

Sinuous (adj.) bendable and flexible; subject to curvature

The thin, sinuous pieces of lead were used by Andy and his family as fishing sinkers.

Intransigent (adj.) refusing to change one’s views, agree or compromise

Juliet always found herself butting heads with her family's intransigent views sometime around Thanksgiving.

Consternation (n.) anxiety or dismay, often as the result of something unexpected

While she was flustered, Connie did not let the consternation bother her during her marathon run.

Usury (n.) the unlawful or unethical practice of lending money at excessive interest rates

The usury practiced by Ben against those who lost money to him playing poker was particular reprehensible.

Mettlesome (adj.) having a proud and unbroken spirit; courageous

While most retired women would've hesitated to canoe through the Boundary Waters so late in the year, Alison was particularly mettlesome, and set off to do just that.

Ascetic (adj.) strictly self-disciplined

The ascetic values of the German army were perhaps inherited from the days of the Prussians and Field Marshall von Blucher.

Venerate (v.) to hold in high respect; to honor

While perhaps seen as an afterthought at the time, the Bill of Rights would prove to be accepted and venerated by the American people.

Hermetic (adj.)(1) insulated from outside influences; (2) having no leaks

Brian became quite hermetic during exam season.

Soliloquy (n.) a lengthy speech given alone which expresses the speaker’s feelings, often used in dramatic productions

Melanie's rendition of the soliloquy impressed her theater class.

Puerile (adj.) childishly silly or unimportant

While Charles often worried about ants or termites in the house, his wife was seldom troubled by such, in her opinion, puerile concerns.

Proscribe (v.) to declare that something shall not be done; to forbid

While the exact date is not clear, a Norwegian law from 1274 mentions only "former" slaves, indicating that slavery had been proscribed in Norway.

Precursor (n.) that which came before another thing

Watt's machine was a precursor to the steam engines which would ultimately drive locomotives and factories.

Prolix (adj.) containing too many words; excessively lengthy

The dissertation was so prolix that the publishing company could not bind hard copies in the ordinary trim size.

Nostrum (n.) a medicine or remedy, often which lacks efficacy or is prepared by one without proper knowledge

The remedy of chewing on aspen branches to relieve headaches proved to be a disappointing nostrum.

Cachinnate (v.) to laugh in an unrestrained manner

Roger drowned out the conversation with his obnoxious cachinnations, often at his own statements.

Antipathy (n.) a strong dislike for a given thing

Alexander Hamilton's antipathy towards the French was well-known amongst his peers.

Enigma (n.) a puzzling set of circumstances; a mystery

The enigma of the Incan counting device known as a "quipu" puzzles scholars to this day.

Peremptory (adj.) (1) imperative without regard to the reasons why; (2) with arrogant self-assurance

The closing of the Maple Street and Vineland stores was seen as peremptory and unwarranted by many shareholders.

Prevaricate (v.) to avoid questions or speak evasively

Yvonne constantly prevaricated when asked about her plans for next year.

Pillory (n.)a wooden restrain used to punish individuals through public embarrassment in days past; (v.) to put an individual in a pillory

His blasphemy landed Ralph in a pillory from time to time.

Expatiate (v.)(1) to move around without restraint; (2) to communicate in lengthy detail

Lisa would be constantly expatiating about the movie she saw for the next few days.

Insular (adj.) isolated from or lacking contact from other things

Life in Brompton was remarkably insular, untouched by trends or the times.

Pretext (n.)a disingenuous reason given for a course of action

In reality, Kenny had little interest in wine; rather, it was a pretext for starting a conversation with Ashley.

Depredate (v.) to plunder or lay waste to something

The war would prove to depredate the Dylewskis' part of Poland.

Presumptuous (adj.) overstepping or exceeding of one’s boundaries or limits

Oscar thought it was rather presumptuous for his geometry students to tell him to change his curriculum because it was too hard.

Sagacious (adj.) demonstrating good judgment; wise or shrewd

Tony had always been a wild spender, despite attempts by his parents to teach him to be more sagacious with his money.

Bedizen (v.) to dress or decorate flamboyantly or gaudily

Meredith had grand plans as to how to bedizen the car for the parade.

Discourse (n.) a series of discussions or conversations

Both Stearns and Carlucci felt that lemurs received far too much attention in the discourse over unique species in Madagascar.

Irascible (adj.) easily made angry

Glen had become much more irascible over the years, and was especially irascible when he had been drinking.

Cadge (v.) to obtain something which one is not entitled to; to mooch

Paul was able to cadge a place to stay for the weekend from his friends in the area.

Epicure (n.) a person with highly refined tastes, especially with regard to food or wine

An epicure like Beth proved valuable in selecting what type of wine to get.

Recapitulate (v.) to summarize main points or ideas with regard to something

The body of the work was done, and Donato needed only to recapitulate the main ideas via a solid conclusion.

Sever (v.) to cut off or disconnect

Though the Finnish alliance with Nazi Germany was arguably only made to work together against the Russians, the United States nonetheless severed ties to Finland.

Inundate (v.) to overcome or overwhelm

Mr. Robbins was inundated with criticism from parents after having his high schoolers watch part of the racy film.

Membrane (n.) a thin and bendable sheet-like substance which forms a barrier, often in the biology of organisms or in industry

Desalinization of water for drinking is now often accomplished using a semi-permeable membrane.


Vocabulary List 2

GRE Vocabulary List 2 edit

Edify (v.) to build

The affordability and reliability of its products would slowly edify the company's reputation over the next decade.

Malevolent (adj.) desiring to do evil or malicious things

Local folklore told of a malevolent spirit that came out on moonless nights to wreak havoc.

Chaste (adj.)(1) not sexually promiscuous; (2) modest, simple or restrained

Michelle was by far the most chaste in her lifestyle among the housemates.

Malinger (v.) to fake or exaggerate illness to avoid some type of work

Most of her colleagues expected at least some malingering on the part of Susan when she called in sick.

Inquisitive (adj.) tending to ask questions or seek information; curious

Felicity was rather inquisitive about the past weekend's events.

Centrifuge (n.) a circular device which forces its contents toward the outside when spun, typically used to separate substances or simulate gravity

The "gravitron" ride at the fair was nothing more than a large centrifuge.

Dilatory (adj.) something conducive to being late; something which causes delay

Car trouble was always a dilatory issue for Frank in getting to work; after all, he drove an old Jaguar. Buttress (n.)(1) a protruding section of a wall designed to give it more support; (2) that which supports a given position or argument; (v.) to give support to something

Remi would buttress his argument by pointing to the fact that one of humans' blood factors was also present in great apes.

Supercilious (adj.) believing or conveying the impression that one is superior

Natasha was always supercilious in her demeanor around the office.

Antithetical (adj.) directly opposing or contravening

Continuing the war was antithetical to the will of the Russian people, and led to the toppling of the provisional government by the Bolsheviks.

Fidelity (n.) the state of always being true to something; faithfulness

John acted with remarkable fidelity while a prisoner of war.

Rationale (n.) set of reasons or bases for a particular belief

Scott's rationale for purchasing the car was that it would be better on the snow when he moved up to Canada.

Dynamo (n.) (1) an electrical generator; (2) a person with a great deal of energy

Bert was a dynamo after a few drinks.

Florid (adj.) of, relating to, or resembling a flower

Kelly awoke to florid arrangements in that white room after the accident.

Enumerate (v.) to assign numbers to something

The enumerated parts of the letter were not the only content which Richard found striking.

Adroit (adj.) clever or skillful in use of one's hands or mind

Wilson was adroit when it came to fixing air conditioners.

Sacrosanct (adj.) valued too highly to be subject to change or interference

Church in the Bankman household was far to sacrosanct to give way for anything else.

Impudent (adj.) rude; failing to give appropriate respect

Bryce was an impudent child, always talking back to teachers.

Insouciant (adj.) lacking care or concern

Marie was far too insouciant to care for her late grandmother's pet dog.

Probate (v.) to establish the validity of a will; (n.) the action of establishing a will

The children hoped that there would not be any disputes related to the upcoming probate of Herman's will.

Involute (adj.)(1) complex, convoluted or involved; (2) physically curling inward

There was a rather involute dilemma, however, as to whether to use the newly acquired land to expand the store or build a sister store.

Eulogy (n.) a positive formal speech or writing regarding a deceased person

Meredith read a beautiful eulogy at the funeral, referencing her times with Elise watching ducks on the pond in the park.

Rue (v.) to deeply regret or wish to have never happened

Anne would later rue her decision not to buy a snowblower.

Mendacity (n.) lack of truthfulness or candor; dishonesty

Only after a background check did Joseph's mendacity become completely apparent.

Obstreperous (adj.) noisy and difficult to manage

The children were far to obstreperous for Erica to deal with effectively.

Pragmatic (adj.) approaching situations and problems on a realistic and sensible basis

While usually she was very pragmatic, Vivie's decision to buy the new television was somewhat impulsive and whimsical.

Complaisance (adj.) tending to want to please or abide by the wishes of others

Trisha's complaisance made her bite her tongue instead of yelling at Ernie for not picking up his cigarette butts.

Forswear (v.) to swear falsely; to deny or renounce under oath

Megan went on to forswear that she did not know anything about where the funds had been transferred to.

Inveterate (adj.) having a longstanding habit or routine which is insusceptible to change or modification

Cecilia was rather inveterate when it came to her weekly routine.

Extirpate (v.) to wholly destroy or remove, often by pulling or cutting out

After the merger, the new management went on to extirpate any reference to old brand names.

Occlude (v.)(1) to close up or obstruct, often in the context of an opening or pathway; (2) to shut away or hide

Christopher occluded himself in the months prior to the bar exam.

Prodigy (n.) a person, often young, of outstanding talent or ability

Monica proved to be a prodigy at playing the fiddle, and went on to win many talent competitions in her native Nebraska.

Misogynist (n.) one who has a strong dislike for women

Cheryl argued that Freud was too much of a misogynist and should be removed from psychology curricula.

Tractable (adj.) easily dealt with, controlled or influenced

Wes was the most tractable after he'd had his morning coffee.

Maculate (v.) to mark with a spot or a stain; (adj.) marked with spots or stained

The maculate dress was just another reminder of the tragic accident.

Forbearance (adj.) restraint, tolerance, self-control

Becky's forbearance prevailed in her decision to save money by not taking the trip to Hawaii.

Eloquent (adj.) clear, simple and understandable

Christina's book featured an eloquent description of the Doge's palace in Venice.

Hierarchy (n.) a system or organization in which individuals belong to ranks above or below each other

Matt was still very low amongst the hierarchy in the bureau.

Penchant (n.) a strong or regular inclination toward liking something or doing something

Gwendolyn was happy that her husband had caught a redfish, as she had a penchant for eating them.

Luculent (adj.) clear and understandable

The preacher's sermon was luculent and beautiful.

Cavil (v.) to raise a groundless or trivial objection to something

Though the opposition continued to cavil with complaints about making the rich richer, the tax cut act overwhelmingly passed.

Doctrinaire (adj.) intending to impose an idea or doctrine

Both men were very doctrinaire about their globalization ideas.

Debacle (n.) a great disaster or upheaval; a catastrophic failure

He hadn't spoken with Chloe since the debacle at her birthday.

Esoteric (adj.) directed at or understood primarily by a small group with special knowledge

The cryptologists' esoteric ramblings went over most peoples' heads.

Tranquil (adj.) calm, relaxing

Rachael loved tranquil, natural piano music.

Aphorism (n.) a short, simple statement which relays a simple concept or lesson; an adage

Aunt Jeanne had so many aphorisms that it was almost annoying to hear her say one.

Aggravate (v.) to annoy or bother

Viennese imperialism would continue to aggravate the Italians well into the 20th century.

Reversion (n.) a return to a previously existing condition

Reversion of the property to the cooperative was part of the agreement.

Spurious (adj.) not as purported; false despite seeming to be true

Uma's spurious claims about the stegosaurus were met with great skepticism.

Retard (v.) to slow or delay the progress or advancement of; (n.) a person of compromised mental ability

The Marne offensive would retard and ultimately hault the German advance on Paris in 1914.

Efficacy (adj.) ability to create a certain result; effectiveness

The efficacy of the sleeping pills was so great that Caitlin did not even remember anything past lying down.

Arduous (adj.) requiring a lot of effort, difficult

Crossing the hills in a Conestoga wagon proved to be an arduous task.

Logistical (n.) of or relating to the organization or transportation of persons or things

The offensive in Galicia proved to be a logistical nightmare for the Russian army.

Apprise (v.) to inform or give notice

The treasurer went on to apprise the others of the situation.

Disabuse (v.) to set right or clarify something; to eliminate a misconception or error

Tony went on to disabuse rumors that he would be retiring.

Churlish (adj.) lacking manners, civility or composure; rude

The boys from Holy Cross Academy were surprisingly churlish given their education.

Qualms (n.) (1) fears or concerns; (2) feelings of sickness or faintness

The weather report did little to ease Edna's qualms about damage to her property in the impending hurricane.

Loquacious (adj.) talkative

Knowing how loquacious his children were, Christopher restricted their use of the telephone so as not to run up his bill.

Commensurate (adj.) set to an appropriately relative level; proportionate

The salary for the position was commensurate with experience.

Unscathed (adj.) without suffering injury or detriment

Since it was captured early on in the war, New Orleans escaped relatively unscathed.

Convention (n.) (1) that which is normal or standard in a given circumstance; (2) a meeting, often of widespread members of an organization

Beethoven's string quartets both exemplified and moved away from convention.

Excoriate (v.) (1) to wear off the outer surface of something; (2) to criticize something heavily

Years of use had excoriated the gardening tools.

Quiescence (adj.) the state of being still, at rest, or at peace

Hubert loved the quiescence when he was home on his ranch.

Laud (v.) to praise or commend, often publicly

Teddy was lauded for his Eagle Scout project at the annual dinner.

Exorbitant (adj.) exceeding any appropriate limit or boundary

After her appointment, Ursula found the corporate jet far too exorbitant and immediately sold it.

Innervate (v.) to stimulate to some action

Marcel's crash would innervate the banning of road racing in Europe.

Overweening (adj.) exhibiting excessive self-confidence

Barry had a very overweening attitude everytime he went to the gym.

Propitious (adj.) favorably inclined

Sister Margaret was very propitious when it came to helping others.

Preempt (v.) to engage in a course of action to prevent a particular event from happening

The goal of refusing to hire smokers was to preempt lost working hours due to smoke breaks.

Militate (v.) to be an important or sufficient means of preventing something

The flood control measures failed to militate valley flooding as expected.

Mollify (v.) (1) to appease or reduce a person’s anger; (2) to reduce or mitigate an impending threat

While Susanna was quite angry, he hoped he could mollify her by talking about it.

Supine (adj.) lying face up

Both of them lay supine, watching the stars.

Piquant (adj.) pleasantly stimulating

The piquant smell of Marty's cooking excited his children.

Effluvia (n.) some emanation or outflow which is often unpleasant or disliked

The effluvia from the plant raised some degree of concern for local conservation groups.

Marginal (adj.) of or relating to the edge of some thing or concept

The success was, at most, marginal, and the team could have done better.

Propitiate (v.) to win favor by doing something; to appease

The success of the research project proved to propitiate sources of funding, and paved the way for newer projects.

Ignominious (adj.) deserving of or causing public disapproval or disgrace

Larry's ignominious behavior caused him so much shame that he ultimately moved out of Lawndale.

Lackluster (adj.) unimpressive or boring

Wootley was threatened with termination after his lackluster performance in September.

Nefarious (adj.) notoriously sinister or treacherous, often with regard to a criminal

Though not as nefarious as some, he was just as dangerous and maniacal.

Recidivism (n.) habitual relapse into crime

Willy's recidivism after release was an embarrassment to his family and friends.

Pellucid (adj.) clear and easy to understand

It was a most pellucid account of the Third Battle of Ypres as told by an English soldier.

Epitome (n.) a perfect example or symbol of something

Wal-Mart was perhaps the epitome of large generic American chain stores.

Placate (v.) to cause to be less angry or hostile

The telegraph to the Kaiser was intended to placate war tensions.

Sidereal (adj.) of or relating to the stars

The sidereal wallpaper in the dining room glowed when the lights were low.

Martial (adj.) of or relating to war or the appropriations therefore

The French martialed as many resources as possible to move troops to the front, including commandeering Parisian taxis.

Pristine (adj.) pure, in perfect condition

Thinking Michigan to be an industrial wasteland, Kent was surprised by its pristine wilderness.

Corroborate (v.) to verify or support with additional independent evidence

Her project was to identify which evidence corroborated the stories in the Book of Acts.

Tyro (n.) beginner or novice

Collin was a tyro when it came to poker.

Minatory (adj.) that which expresses a threat

The letter to the Duke was somewhat minatory when it mentioned what would be done in the event of noncompliance.

Extraneous (adj.) unnecessary, irrelevant or superfluous

Maurice's coat proved to be completely extraneous, as the weather was sunny during the entire trip.


Vocabulary List 3

GRE Vocabulary List 3 edit

Aplomb (n.) composure and self-confidence

Her father's aplomb never wavered throughout the course of the meeting with the banker.

Trifling (adj.) lacking importance

Though some at the computer store objected that the room temperature was slightly too hot, Becky was not to be bothered with such trifling concerns.

Peripheral (adj.) of or relating to the sides, edges, or perimeter of something

The class was primarily about the history of Christianity since its inception, and the early days as discussed for example in the Book of Acts were only peripheral to the curriculum.

Turgid (adj.) swollen, misshapen or congested

The physician was unable to make a diagnosis with regard to the turgid, painful lesions on the patient's leg, without knowing more.

Guile (n.) craft; cunning

Franz's remarkable guile again proved to help him when he was able to figure out how to build a computer program to fill in the data forms for him, saving time and effort.

Diaphanous (adj.) something which is delicate, vague, lacking substance, or ethereal

Her diaphanous memories of her childhood were often difficult to sort out or comprehend.

Invective (n.) heavily critical or insulting language

Mr. Barrison would typically shout a biting insult or invective at the other person when he didn't get his way.

Asseverate (v.) to assert the truth of something

Fanny proceeded to asseverate that she had seen the fight outside the bar, and that Keb was not the aggressor.

Avarice (n.) extreme greediness

Wesley's avarice was apparent as he happily conned the old woman into buying the vacuum cleaner.

Extemporaneous (adj.) done in a manner which is impromptu, spur of the moment, or otherwise lacking preparation or forethought

Unable to find the proper anti-virus software, Clarence came up with the rather extemporaneous remedy of wiping the entire hard drive and reinstalling all of his files.

Empirical (adj.) based on observable and testable methods

Stacy was a firm believer in astrology despite its lack of confirmation through any empirical methodology.

Chauvinist (n.) one who believes in the supremacy of some group or thing with which that person is associated; an arrogant person, often in the context male sexism against women; (adj.) that which is a belief held by a chauvinist

While she had expected her boss to hold some chauvinist views, she was still blatantly offended at the comment.

Platitude (n.) a remark that has been overused to the point of lacking meaning

The negotiations had drug on to the point that "compromise" was becoming a platitude.

Sophistry (n.) the use of flawed or fallacious arguments, often deceptively

Kenneth was annoyed at how easily the other jurors fell for the lawyer's sophistry.

Autonomy (n.) the state of being self-governing or self-directing

Although under the authority of the marketing department, Andrew's division exercised a considerable degree of autonomy when it came to designing advertisements.

Shrill (adj.) piercingly loud or harsh, often with regard to a scream; (n.) a shrill scream; (v.) to emit a shrill

They knew from the shrill noises that something was amiss back at the campsite.

Candor (n.) honesty; straightforwardness

Stephanie's lack of candor during the investigation contributed to the harsher sentence which would later be imposed upon her by the board.

Lackadaisical (adj.) lacking the appropriate effort or enthusiasm

Though good at his job, Bruce was a lackadaisical parent

Bane (n.) a source of harm or trouble; that which is hated

Malpractice lawyers have long been the bane of the medical profession.

Boor (n.) an inconsiderate or unrefined person

Though he was very knowledgeable on the subject, he was also something of a boor.

Disperse (v.) to break up the concentration of something; to take that which is concentrated in a small area and spread it out over a large area

The Koontz family was dispersed all over central Illinois.

Stint (v.) to restrain or be sparing with regard to (n.) a short period of work

After a brief stint as a tax preparer, Hunter took a job at a publishing company.

Arbitrary (adj.) lacking any clear reason or rationale; capricious

Joseph found the decision to discipline him but not Otto somewhat arbitrary.

Turbid (adj.) cloudy, opaque, or thick with an absorbed substance, usually in the context of liquids

The turbid concoction in the tank was only one of a series of chemicals processed at the refinery.

Impediment (n.) something which contravenes a goal; an obstacle or problem

The weather proved to not be as much of an impediment, given that Marta drove a Subaru.

Insidious (adj.) sinister or treacherous; harmful

Robert's insidious intentions were revealed when the others went through the files on his computer.

Sanguine (adj.) exhibiting a positive attitude

Though others were nervous about the weather, Melanie was rather sanguine.

Supplication (n.) invocation or prayer

The initial supplication was a short and simple statement by the reverend.

Pusillanimous (adj.) having no courage; timid

Wendy was rather pusillanimous on that day, and it took a great deal of persuasion to get her to come along.

Temperate (adj.) (1) of or relating to mild temperatures; (2) showing restraint and moderation

Umberto was much more temperate than his anger-prone wife.

Repine (v.) to feel or communicate discontent or stress

They would constantly repine to each other in the break room about the woes of working for BH&H.

Divest (v.) to remove or take away

The financial subcommittee's members were ultimately divested of their power after a series of lamentable decisions on their part.

Presumption (n.) something which is taken to be true though never stated, often not illogically

In Martin's mind, there was a presumption of poor driving if somebody drove into a stationary object.

Savor (v.) to enjoy completely, typically with regard to taste or smell; (n.) a desirable flavor or aroma

Yoshi had every intention to savor his first victory, knowing that it wouldn't always be so easy.

Precipice (n.) a steep and dangerous cliff or rock face

Mentally, Wanda was at the edge of a precipice, about to fall and tumble down.

Rectify (v.) to make right, fix, correct

Haley worried that what happened in the past was a mistake that she would never be able to rectify.

Ingratiating (adj.) seeking approval or favor

His ingratiating requests only annoyed them, and did little to carry favor.

Deplore (v.) to strongly disapprove

The monarchy would always claim to deplore violence and brutality, even after the February incident.

Dissolution (n.) an act which causes something to come apart or cease to exist

Things had been less stressful since the dissolution of the two's partnership.

Clemency (adj.) the state of being mild, merciful, or forgiving

Though she was angry at him, she displayed a remarkable degree of clemency after his sincere apology.

Superficial (adj.) existing only at the surface; lacking depth

To Jeb, she was the only cheerleader in high school who wasn't totally superficial.

Impunity (n.) immunity from punishment or consequences

President Bush made very clear that officer Koon's acts were not shrouded by impunity, and that the justice system would have its turn with him.

Tamp (v.) to ram or pack down a substance firmly

With the handle of his knife, Custer proceeded to tamp the outside of the pie crust all the way around.

Toady (n.) one who is far too attentive and obedient (v.) to be a toady

Every junior associate at Milbank was a complete toady - a diligent little retriever for that person's superior.

Voluble (v.) to fluent, flowing, coming easily (especially with regard to language)

A native speaker of German, Susanne found the English language rather voluble.

Coquettish (n.) of or relating to a flirtatious woman

Pamela wasn't the only coquettish person in the bar, but she was by far the most noticeable.

Precept (n.) a general rule designed to govern individuals’ behavior

Though largely rejected in the field, Freud's precepts remain well-known today in popular culture.

Wean (v.) (1) to become accustomed to a food other than a creature’s mother’s milk; (2) to become accustomed to something new, often which is less desirable

Patty was unsure exactly when was the proper time to wean a colt.

Interdict (v.) to prohibit or forbid; (n.) a prohibition or forbiddance, often from the government

The mayor issued an order to interdict any activities in the city park after midnight, given the recent gang activity there.

Inveigh (v.) to speak or write about a topic with great contempt

He would go on to inveigh at length about the horrors of working in an emergency room in a sketchy part of town.

Epithet (n.) an insulting or offensive word or phrase

While Marcus wasn't sure which epithet the man had screamed at him, he was quite certain it didn't matter.

Defalcate (v.) to wrongly use things entrusted to oneself, to embezzle

Unbeknownst to Brian, every time he would defalcate property of his employer, it would catch up to him.

Connoisseur (n.) one with a great deal of knowledge, interest, and often pickiness in something

Jenny was a self-proclaimed connoisseur of fettuccine alfredo.

Deprecate (v.) to belittle or downplay something

He made no effort to deprecate how much he loved Gilmore Girls in front of his male friends.

Subtext (n.) an underlying theme, often in literature

Some believed that Native Son's true subtext was that the tragedy of the story was society's own creation.

Culpable (adj.) something which can be blamed or which is the cause of something

The faulty sprinkler system was partly culpable for the destructiveness of the fire.

Bucolic (adj.) of or relating to a country or rural lifestyle; pastoral

La Sonnambula was memorable for its beautiful singing and bucolic setting.

Estimable (adj.) capable of being determined in approximate amounts; capable of being estimated

Though the exact harm was only very crudely estimable, Barnes knew that it was in the millions.

Guise (n.) a manner of appearance or presentation, typically concealing a person or thing’s true nature

The entity was in fact a collection agency operating under the guise of a law firm.

Undue (adj.) to a degree excessive with regard to that which is necessary or appropriate

Everyone knew it would be an undue strain on the store's resources to move all of the merchandise off the ground floor in the event of a flood.

Enormity (n.) (1) that which is very large; (2) that which has which has improperly gone beyond its bounds

The sailing apparatus on the Cutty Sark, a clipper ship, was an absolute enormity.

Secrete (v.) (1) to produce or emit, often with regard to a gland or body party; (2) to hide away

Aphids secrete a sucrous substance eaten by the ants who tend to them.

Diatribe (n.) a bitter and negative criticism or attack upon something or someone

The article was yet another diatribe appearing in The Wall Street Journal about the mishaps of politicians.

Astrology (n.) a field of study in which people try to predict future events based on the alignment of planets, stars, and other celestial objects

Karen loved the astrology section of the newspaper.

Facetious (adj.) joking; not meant to be taken seriously

Phil's claim to be a member of the Spanish royal family was facetious, though some may not have taken it that way.

Tentative (adj.) for the time being; subject to change

The tentative recording schedule involved laying the drums and bass down first.

Clique (n.) an inclusive social group

In high school, the two had been part of the same clique.

Sycophant (n.) one who is excessively obedient or flattering of another, often for purposes of taking advantage of the person

Edgar found it interesting how, in his old age, everyone he knew began to turn into something of a sycophant.

Tacit (adj.) understood or implied though not explicitly stated

Vivie and Taylor had a tacit agreement with regard to walking the dogs every day.

Facilitate (v.) to do something which allows something to happen; to accommodate

Veronica's efforts were necessary to facilitate expansion into the internet advertising field.

Warrant (v.) to provide an adequate basis for undertaking an action or accepting a fact; (n.) something which provides an adequate basis, such as a document used by law enforcement to validate a search

Though it raised some suspicion, the fact that the man was carrying a crowbar itself did not warrant arresting him.

Disingenuous (adj.) lacking honesty or candor; that which is fake or a sham

Felicity's stated reason for hiring her cousin - that he was the best person for the job - was universally seen as disingenuous.

Ambiguous (adj.) open to more than one potential meaning

Poe's stories were ambiguous as to whether they were merely entertaining fiction or allegorical with regard to larger concepts in life.

Redolent (adj.) (1) strongly reminiscent or suggestive of another thing; (2) having a strong smell

The hair tie was quite redolent, reminding him of Nancy.

Salacious (adj.) stating, suggesting or implying that which is sexually inappropriate

Janet became enraged at the salacious comments, and demanded an apology.

Doggerel (adj.) irregular, disproportionate, or badly put together

They were not looking forward to pouring through another fifty pages of the doggerel that always made up Ivan's reports.

Covert (adj.) that which is secret, hidden, disguised

Though he could never prove it, he always suspected that Lindsay had a covert motive.

Delineate (v.) (1) to mark or depict something by drawing a line; (2) to describe something

They went on to delineate the categories of permissible employee expenditures.

Approbation (n.) an act of approval or praise

The firefighters received regular approbation from the city of Harahan.

Errant (adj.) having gone astray

Errant golf balls were a common source of lawsuits against the golf course.

Verbose (adj.) using excessive or unnecessary words or language

The verbose writing style was typical of standardized tests. Splenetic (adj.) ill-tempered and unpleasant

Terry did not enjoy being forced to babysit her bratty, splenetic cousin for the day.

Puissance (n.) great strength, power or influence

Rex had a great deal of puissance at the country club.

Inherent (adj.) that which is an invariable attribute or trait of something

The ability to understand "quick-sale" value was inherent in the job description of any appraiser.

Anoint (v.) to smear or rub with some substance, often as part of a religious ceremony

The priest went on to anoint the others.

Fallow (n.) (1) a plot of land not planted during a given growing season; (2) a shade of yellow

The Grefsruds' land had not been fallow for over a decade.

Prodigal (adj.) excessively wasteful or indulgent

Dave was rather prodigal and neglectful when it came to the family business his father had let him run.

Panegyric (n.) a public speech or published text which speaks highly of something

The president's latest panegyric was with regard to civil rights leaders in the mid-1900s.

Implode (v.) to collapse or destruct inward

Stein predicted that the entire economy would implode if taxes were continually increased.

Reconciliation (n.) a reestablishment of cordial relations or understanding

While the reconciliation took a long time, eventually the two were able to be friends again.


Vocabulary List 4

GRE Vocabulary List 4 edit

Extenuate (v.) to lessen the seriousness of something; to mitigate or excuse

While touching, his sob story failed to extenuate the punishment imposed on him.

Luminous (adj.) (1) producing or conveying light; (2) clarifying or resolving some uncertainty

The Rosetta stone proved to be a luminous discovery with regard to understanding the language of ancient Egyptians.

Headlong (adv.) (1) headfirst; (2) impatiently and without thinking

Chuck was known for his poorly thought-out, headlong acts.

Aberrant (n.) different from or departing from what is normal

Though she had noticed her sister's aberrant behavior with regard to the pet raccoon, she thought little of it at first.

Pedantic (adj.) (1) resembling a pedant; (2) overly concerned with formal rules and trivial points of learning; showy with regard to knowledge and language

Though initially impressed by her Ivy League education, after an hour of conversation, Kristen found Saru to be nothing more than an endless sea of inflated, pedantic ramblings.

Transitive (adj.) of or relating to motion or change from one thing to another, especially when this involves multiple steps

Ludwig von Beethoven is often considered to be a transitive figure in European music, as evidenced by the changes through the course of his string quartets. Unprecedented (adj.) that which has never occurred before

Edmund Hilary's feat was, at the time, unprecedented.

Inalienable (adj.) Not subject to being taken away; permanent

Though the anti-execution advocates agreed that certain liberties could be forfeited, they maintained that life should be an inalienable right.

Beatify (v.) (1) to for a religious official to declare to have achieved the blessedness of heaven (2) to make a person very happy

His gift of a new dough mixer proved to beatify her greatly.

Rubric (n.) a direction as to how something should be conducted, often in the context of a church service

The administrator provided each intern with a description of the rubric by which their performance would be evaluated.

Cozen (v.) to trick, fool, or deceive

While he had fooled Mrs. Korbel, Jeff was not about to cozen Mrs. Patsel with his story about the lawnmower and his homework.

Lugubrious (adj.) appearing to be sad or depressing in some way

Given the lugubrious look on her face, Erin was certain that her roommate had been fired from her job.

Exacerbate (v.) to worsen an existing problem

They found that arguing with the woman at the traffic ticket office only served to exacerbate their problems.

Debase (v.) to reduce in value or worth

The construction of the tar factory proved to debase the rest of the properties in the neighborhood, much to Mrs. LaPorte's dismay.

Multifarious (adj.) having many parts, types, styles or aspects

The people who hung out and raced near highway 315 were a diverse, multifarious gang of hooligans.

Duplicity (adj.) the state of having two parts, sides, variations, or the like

Kenny failed to see the duplicity of Romeo & Juliet as story of both love and tragic lust.

Metamorphosis (n.) the process of changing from one thing into another

The metamorphosis from Weiz Corp. to W&W Holdings took more than ten years.

Proxy (n.) (1) a person authorized to act on behalf of another; (2) something which acts as a substitute for something else

Since Lauren would not be around to vote, she designated Gabby as a proxy.

Goad (v.) to provoke or taunt to prompt some desired action; (n.) a pointed stick for herding cattle

Seeking to unite Germany, Bismark managed to goad the French into war in the late 1800s.

Dispassionate (adj.) lacking strong feeling

Carlos was rather dispassionate when it came to the choice of where to stop for dinner.

Intersperse (v.) to scatter or mix such that one thing is varyingly distributed within another

Chinese cooks intersperse olive oil with canola oil and use this blend for both cooking and frying.

Bombast (n.) remarks or writing which are inflated or exaggerated

Jared's latest bombast was his claim to have been valedictorian in high school.

Serenity (n.) the state of being calm or tranquil

The serenity of the forest was a pleasant relief from life as an accountant in the big city.

Untoward (adj.) that which is unexpected and inappropriate or inconvenient

Andre found the comment about his hat to be rather untoward, and did not appreciate it.

Fortuitous (adj.) merely the result of happenstance or good luck

It was rather fortuitous that the old Dodge pickup truck made it to Escanaba and back without breaking down.

Remonstrate (v.) to say in protest or opposition

In political science class, Dr. Jackson would typically spew out his conservative views, and the students would remonstrate their own opinions back to him.

Rarefy (v.) to lessen in density or solidity

Jenkins' lack of complete knowledge of the situation served to rarefy his culpability in the matter.

Illusory (adj.) the state of being an illusion; not real

Fanny's sense of security in the matter would prove to be illusory.

Salient (adj.) predominant, noticeable or important; (n.) a part of a landmass or structure which comes to a point

Most of the voters saw the economy as a particularly salient concern.

Conversion (n.) the act of changing something into something else; the act of converting

Cassius Clay changed his named to Muhammad Ali upon his conversion to the religion of Islam.

Dirge (n.) a sad or mournful melody, often played at a funeral

Though typically brighter, when Largo was played by the pit orchestra that night it sounded more like a dirge.

Assuage (v.) to ease the severity or intensity of

Wendel's promises of better sales next year failed to assuage the fears of the stockholders.

Perfidious (adj.) not capable of being trusted; dishonest or deceitful

Yolanda thought that the man she had hired was perhaps the only lawyer in the state who was not a perfidious parasite.

Ordinance (n.) an authoritative decree or rule, often by a municipality

Mr. Palmer was alleged to have violated the city's loitering ordinance.

Catalyst (n.) that which enables a reaction or chain of events to occur

The assassination of Archduke Francis Ferdinand, as well as the events which happened thereafter, proved to be the catalyst for World War I.

Solvent (n.) something causes something else to be absorbed or eaten away; (adj.) having sufficient assets to pay all of one’s liabilities

Though deep in debt, Kelly's business remained solvent.

Bellicose (adj.) predisposed towards starting conflicts

Mr. Hopkins, who had a history of being bellicose, was immediately believed to have been the instigator of the fight.

Fastidious (adj.) having a great deal of concern with regard to order, precision and detail

Luigi was known to be the most fastidious taxidermist in all of British Columbia.

Paucity (n.) a lack of something

The great paucity of qualified tax professionals in the area was the source of substantial disappointment for Marge and her husband.

Simper (n.) a flirtatious smile; (v.) to display a simper

He had seen that simper on her face before the last time they went shopping at Macy's.

Typify (v.) to be a fair representative example of something

Being just under five feet tall, Brenda certainly did not typify basketball players.

Reconnoiter (v.) to make an observation of a region, often with reference to a military objective

Stacy went to reconnoiter the dessert table with regard to any remaining cupcakes.

Incubation (n.) a stage of isolated development

Before being launched nationwide, the business model was put through an incubation phase at a single location in Ohio.

Impugn (v.) to dispute or call into question

Her speech went on to impugn the claims of BP about the oil spill cleanup effort.

Vehement (adj.) passionate or intense

Heidi was a vehement supporter of ending the Cuban embargo.

Gauche (adj.) Lacking in ease or grace; socially awkward

His speech was rather gauche, full of stuttering and jittering.

Succor (n.) assistance or aid; (v.) to provide succor

The purpose of the Berlin Airlift was to succor the portions of Germany which had been roadblocked by the Soviets.

Rapacious (adj.) aggressively greedy or self-indulgent

Corporate litigators were known to be rapacious, and he was no exception.

Jejune (adj.) overly simple, naive or uninteresting

Her jejune outlook towards the world would prove to get her in trouble.

Fledgling (n.) (1) a young bird; (2) that which is new or just getting started

Once a fledgling computer repair center, Turtle Computers now dominated the local market.

Perigee (n.) (1) the orbital point at which a satellite is closest to the object it orbits; (2) the lowest point of something

For hardware retail, January and February was the perigee of the year.

Pulchritudinous (adj.) having great physical beauty, invariably with regard to other persons

The pulchritudinous bride stood out amongst the other bridesmaids.

Undulate (v.) to move in a wavy patterned motion; (adj.)the state of being wavy

The undulate pattern on her scarf was noticeable, even in the large crowd.

Distill (v.) to reduce into something more concentrated

This particular brand of vodka is distilled five times over.

Imbroglio (n.)a confusing or socially problematic situation

Brian then joined the imbroglio by arguing with Samantha about the best way to get to Covington.

Defamatory (adj.) having the effect of injuring a person’s reputation

The defamatory statements, while hurtful, did not cause any lasting harm.

Detriment (n.) a problem or source of trouble

The property was moved out of state, to the detriment of creditors who would have seized it.

Superfluous (adj.) unnecessary; extraneous

It was so hot that they began to remove many superfluous articles of clothing.

Construe (v.) to clarify or elaborate on the meaning of something; to interpret

She had a tendency to construe the meaning of poetry in vary unusual ways.

Petrous (adj.) (1) hard as stone; (2) of or relating to rock

The petrous foundation below the house prevented it from suffering serious damage during the earthquake.

Garner (v.) to gather or collect, often in the context of necessary evidence or permission

Julia's request would ultimately garner approval from the water board, but it would take several months.

Inopportune (adj.) occurring at an inconvenient time, place or situation

Baxter would always telephone at the most inopportune time.

Apogee (n.) (1) the point at which an object, such as the Earth, is furthest from the object which it orbits; (2) a high point or climax

Verdi and Wagner arguably represented the apogee of 19th century opera music in their respective nations.

Burgeon (v.) to grow or expand, often rapidly

Arlington continued to burgeon as more highways were built.

Nexus (n.) a connection by which multiple things linked together

The nexus between the pollution and the poor health of the populace was of great concern to Andy and Mary.

Infer (v.) to deduce something to be true which is not explicitly stated

They could infer, based on the writing style, that whoever wrote the note was not particularly skilled in the English language.

Propagate (v.) to cause to be reproduced and spread

Joseph Smith went on to propagate his ideas amongst the masses, and he attracted many followers.

Stanza (n.) a blocked group of lines in a poem

The final stanza of a sonnet is shorter than the first three.

Lambast (v.) to strongly criticize

Hillary went on to lambast uncle Hal for not doing enough about the leaky roof.

Byzantine (adj.) (1) of or relating the ancient city of Byzantium or its area of influence; (2) (typically not capitalized) that which is devious and secretive

Her byzantine agenda was known only to a few in the office.

Sate (v.) to completely satisfy

Though able to sate Belinda's appetite, the spaghetti left Martha still hungry for more.

Raffish (adj.) vulgar, racy, unconventional (especially in a sexual manner)

Aloysius was tired of the raffish programs she always saw playing on television.

Pundit (n.)an expert or notable figure in a field who regularly gives opinions to the public

Whether or not a pundit, Mr. Barrows had a point about the current federal budget.

Monotonous (adj.) having little variation in tone or style; boring

While Cally loved watching European movies with subtitles, her boyfriend found the reading involved in watching them monotonous.

Vigor (n.) a person’s strength, healthiness or enthusiasm

Edna still demonstrated great vigor for a woman of her age.

Castigate (v.) to subject something to harsh criticism or punishment

The penalty was both to castigate Donald for his behavior but also to deter others who might do the same.

Voracious (adj.) having a very strong or insatiable appetite

The lacrosse players were particularly voracious, and ate all of the stromboli.

Baseness (adj.) lack of worthiness in some regard due to absence of some higher understanding

Wesley chuckled at Jimmy's baseness when it came to exotic cars.

Lethargy (n.) a lack of energy or motivation

Patty had been meaning to write a book about lethargy, but had never quite had the motivation.

Viscous (adj.) having a thick composition such that free flowing is slow, typically in the context of liquids

Meredith didn't want to get any of that viscous engine oil on her nice blue dress.

Curmudgeon (n.) a grumpy and unpleasant person, usually an elderly male

Brian was well on his way toward becoming a curmudgeon.

Sinecure (n.) a position of great title and financial benefit which involves little work or liability

"Hiring management supervisor" was essentially a sinecure for relatives of the company's owners.

Recede (v.) to move back or fall away from a previous position

The ocean would ultimately recede, allowing the sunken boat to be retrieved.

Abstruse (adj.) obscure, difficult to understand, or known by very few, often with relation to knowledge

The abstruse nature of 17th century architecture was a little known subject in Davenport.

Tourniquet (n.) an item used to stop blood flow when tightly tied around an appendage

A former boy scout, he was trained in treating rattlesnake bites with a tourniquet.

Harangue (n.) aggressive speech

While "Cross of Gold" was Brian's most famous oration, it was not his only harangue.

Mitigate (v.) to lessen the severity or immorality of a particular action

Stephanie hoped that her youth and inexperience would mitigate her punishment from the disciplinary committee.

Penury (n.) an extreme state of poverty or lack of resources

The penury of the residents of the public housing development was all too apparent.

Imperious (adj.) asserting of authority, often excessively

The store security guard was rather imperious when it came to asking to check peoples' pockets.

Anomaly (n.) that which ordinarily does not happen

Chet insisted that his accomplishment in the chess tournament was an anomaly.


Vocabulary List 5

Vocabularly List 5 edit

Leviathan (n.) (1) a sea monster mentioned in the holy texts various Abrahamic religions; (2) a large ocean-dwelling creature; (3) an ominous and powerful entity, such as the government

Though experienced in litigation, Teddy did not feel he was yet ready to go up against the leviathan that was the EPA.

Parry (v.) (1) to block a blow or strike with one’s own counterblow or counterstrike, typically in the context of weapon combat; (2) to turn aside, dodge or evade; (n.) an act of parrying

Quentin's campaign found it easy to parry off the disparaging remarks and rumors concerning his college years.

Verdant (adj.) of or relating to countryside or lush vegetation

The couple had a most verdant back yard, replete with many flowers and shrubs.

Multifaceted (adj.) Having many sides or aspects

The panel conducted a multifaceted review of the incident.

Denounce (v.) to criticize or speak negatively about something, often publicly

Abby was not the only person to denounce the new state law.

Posterity (n.) all future generations

The posterity of the Hapsburg line would never regain the Austrian throne.

Indolent (adj.) lacking desire to be active; lazy

While most of the workers in the office were quite dedicated, Marcy was rather indolent.

Pugnacious (adj.) having a strong tendency toward argument or conflict

The establishment had long since fired the bouncer for his pugnacious tendencies. Nonplused (adj.) surprised or confused to the extent of inability to form a reaction

Like the deer in the headlights, Remi was nonplused upon learning of his mother's death.

Adversity (n.) those things which are hostile to or thwarting of goals or desires; opposition, challenge, difficulty

Caitlin had little problem with setbacks or adversity, and did well in the competition.

Malign (adj.)that which is evil or malicious in nature

The malign tendencies of the Sigma Nu fraternity were well known across campus.

Blandishment (n.) that which has the tendency to cause or encourage a course of action; an enticement or allurement

The promise of pecuniary gain was the primary blandishment for Bridget and Rachel when they decided to invest in silver certificates.

Precipitate (n.)that which is condensed out of the air, such as rain; (adj.)having been done suddenly without much thought or consideration; (v.) to bring about, often abruptly or without warning

The Kansas-Nebraska Act served to precipitate the Civil War. Craven (adj.) having absolutely no courage

Though he was brave and stood his ground, his craven little brother ran away when the Soviets began firing.

Attribute (n.) a characteristic, trait or skill; (v.) to give credit for authorship or production, often in the context of literature or music

Scholars would ultimately attribute the minuet not to Bach, but to one of Bach's contemporaries.

Antediluvian (adj.) (1) of or relating to the time before the great flood described in various texts of Abrahamic religions; (2)(colloquially) old, archaic, or outmoded

Rich still retained many antediluvian ideas about women which annoyed Mary.

Precis (n.) a summary or outline of a speech or literary work

James wrote an excellent precis of The Jungle.

Disparage (v.) to speak about something with contempt or as being lower

Peter hoped that the remarks would disparage the opposition more than they actually did.

Convoluted (adj.) that which is detailed, complicated or difficult to follow

The standardized test was full of long, convoluted reading passages.

Turpitude (adj.) malice, wickedness, or depravity

Eichmann was put on trial in Israel for his alleged acts of moral turpitude.

Entreat (v.) to plead or make an offer

Captain Rafalko returned to Houston to entreat the shareholders of Neptune, who he felt he could sell on the new merger agreement.

Denigrate (v.) to attack or defy the status, reputation or importance of something

Wesley found it annoying how others would denigrate his career as a financial analyst.

Confabulation (n.) a casual conversation

The confabulation on the porch between Mrs. McLaughlin and Mrs. Dylewski lasted more than four hours.

Glib (adj.) smooth but insincere, often in the context of spoken words

The would-be-buyer soon saw through Marty's glib descriptions of the used car.

Mettle (n.) an ability to deal well with new and unforeseen situations or problems

Andrew knew that the others would help him if he could prove his mettle.

Sententious (adj.) tending continually offer moral statements or aphorisms

Jennifer was not looking forward to the prospect of spending another Thanksgiving being lectured by her sententious great aunt and uncle.

Idyll (n.) a peaceful or picturesque scene

Nate told her stories which left poetic idylls in her head of life growing up in the country.

Accretion (n.) (1) growth by steady accumulation; (2) that which is the result of accretion

The slow accretion of wealth in K&H holdings was mostly due to the investment efforts of Hughes and his subordinates.

Augury (n.) a determination of what is to happen in the future based on signs or omens

The future business' partners initial friendship in school proved to be an augury, and years later, they would ultimately end up working together.

Palliate (v.) to alleviate or make less severe

A sip of the brandy helped to palliate the pain in Joel's foot.

Cogent (adj.) requiring or strongly supporting some result; compelling

The fact that he tried to walk out of the convenience store with the can in his pocket proved to be cogent evidence of his intent to steal it.

Plummet (v.) to fall or drop, often at a high speed

The dislodged piece of the airplane went on to plummet toward the sea far below.

Condemn (v.) to speak negatively about something or blame something; to declare something unworthy

Fran made it a point not only to condemn the mayor, but the city council as well.

Vitriol (n.) (1) harsh or unfriendly criticism; (2) sulfuric acid

The lengthy vitriol of the new advertising campaign by Courtney and a few others ultimately led to its cancellation.

Blithe (adj.) lacking concern or attention

Though Colleen was paying attention, her assistant was quite blithe.

Perfunctory (adj.)done with as little effort as possible

They made at most a perfunctory attempt to find a place to stop for dinner in Bristol, preferring to continue driving into Virginia and find a place there.

Blight (n.) disease or decay; (v.) to infect with a blight

The durum wheat crop in North Dakota would be ravaged by severe blight that year, leading to a jump in the price of spaghetti.

Synergy (n.)an interaction of the energy or ability of multiple persons or things

The branch manager's efforts to promote synergy were only really an annoyance to those in the office.

Audacious (adj.) (1) extremely or recklessly daring or bold; (2) rude or insolent

Roger's audacious quest to memorize Pi to a thousand places attracted the support of many of his friends.

Moratorium (n.) a temporary prohibition with regard to some activity

The administration placed a moratorium on drilling after the great oil spill.

Evince (v.) to clearly point to a conclusion

As evinced by the clothing found in Mr. Stone's closet, he was clearly the same person described in the report.

Limn (v.) to depict or describe via painting or literature

In her paintings, Laura love to limn the lives of her ancestors in Europe and the early colonies.

Bilge (n.) (1) the lowermost part of a ship where water often collects; (2) ideas or remarks which are nonsense, worthless, or a waste of time

Mr. Zaffny's argument proved to be a bunch of hot air and bilge.

Frisson (n.) a momentary excitement or thrill

After the brief frisson that followed the good news regarding the football team was over, the doldrums of everyday life ensued.

Stratagem (n.) (1) a plan or scheme, often utilized strategically or to outfox an opponent; (2) craft or skill at designing strategic plans

Sally's cunning little stratagem was to wait until after the holiday season to shop for kitchen gadgets.

Restitution (n.) return of or compensation for something of which a person has been deprived

Embarrassed to have been caught stealing, Matt made full restitution to J.C. Penny.

Aversion (n.) intense dislike

Meredith had an aversion to onions, and thus was in no mood to eat the Philly Cheesesteak sandwich.

Taciturn (adj.) tending to say few words

Bruce had been more talkative as a child, but in recent years had become more taciturn.

Veracity (adj.) truthfulness; the state of being true

Historians dispute the veracity of the existence of Socrates, with some believing him to be merely the literary invention of Plato.

Hapless (adj.) having no good luck; unfortunate

The hapless travelers found that their luggage was over a thousand miles away in Bremerton.

Equipoise (n.) a state of being evenly balanced; (v.) to evenly balance

The bargaining power of each side was far from equipoise, given that Raritan Bank had the power to seize the couple's home.

Jibe (v.) to come to the same conclusion; agree

Though neither was completely satisfied, the parties would ultimately jibe with regard to what to do about the flooding problem on both of their properties.

Slake (v.) to quench or satisfy one’s desire

The magazines would prove to slake Camellia's desire for civilization the camping trip for awhile.

Fawn (n.) a deer less than one year old (v.) to produce young

The fawn followed its mother around the field as the crowd watched.

Partisan (adj.) of or relating to a party, especially a political party or the party’s cause

The senator was known for never going against partisan interests.

Placebo (n.) a pharmacologically inert substance

Though she insisted it made her feel better, the drug given to Elizabeth was but a mere placebo.

Travesty (n.) a false, distorted, or mocking representation of something (v.) to represent something in the form of a travesty

Martin's "still life" painting could only have been described as a travesty.

Fervid, fervent (adj.) having a great degree of intensity; passionate

None of the employees were particularly fervent about working at the sporting goods store.

Tumid (adj.) (1) swollen, especially with relation to body parts; (2) ostentatious and pompous

They were used to Mr. LaNasa's tumid and outrageous claims.

Effluent (n.) that which is emanating or flowing out; outflow

The people downriver regularly complained about the sewage effluent from the plant.

Contumacious (adj.) stubbornly refusing to obey authority, often in the context of a criminal defendant

She was rather contumacious with regard to her supervisors at the store.

Edict (n.) a command or directive

The church's position on the matter came from a papal edict issued some thirty years prior.

Denouement (n.) the final outcome of some complex turn of events, particularly in literature

The novels' denouement would not come until the sixteenth volume, when the plot was finally resolved.

Asperity (adj.) the state of being, rough, harsh, severe, or uneven

Claudia's study of orthopedics was full of a great deal of asperity.

Paradigm (n.) an intellectual framework or worldview with regard to an issue

The paradigm of most of the people working for the newspaper was somewhat different from Jake's.

Nadir (n.) (1) the low point of something; (2) the point on a planet or other celestial body furthest from the observer

The late 1800s is often considered the nadir of harmony in American Society.

Conspicuous (adj.) that which is easily noticed, obvious, or apparent

The two had a large conspicuous gym bag with them, and Wendy could only wonder what they had in it.

Volatile (adj.) heavily reactive, unstable or dangerous

The most volatile substances at the lab were off-limits to most people for safety reasons.

Transliterate (v.) to write or print a letter or word from another language using the closest corresponding linguistic implements

Charles typically forewent the vowel modifiers in in his transliterations of French.

Recrudescence (n.) the return of some unpleasant condition or situation after it previously had ceased

The recrudescence of Virginia's symptoms prompted a great deal of concern amongst family members.

Endemic (adj.) confined to a particular region or group

Though notably present in West Africa, sickle-cell anemia is certainly not endemic to this area.

Tawdry (adj.) flashy and showy but cheap and of poor quality; (n.) items such as jewelry which are tawdry

The personal injury lawyer's tawdry advertising annoyed members of the community.

Labile (adj.) (1) of or relating to mood swings or spontaneous emotions; (2) susceptible to change or modification

They both knew that Eliott had rather labile opinions on the subject.

Discrete (adj.) distinct and recognizable

The marketing firm followed many discrete tactics and strategies.

Misnomer (n.) an inaccurate or misleading name or designation

The"war veteran", in Gwen's opinion, was a misnomerwhen applied to Fuchs.

Eradicate (v.) to wipe out or cause to cease to exist

While it took a few months, the couple managed to eradicate all cockroaches from the building.

Umlaut (n.) a marking of two dots placed over a word commonly used in the German language; (v.) to mark with an umlaut

Skilled in German, Kenny knew that the umlaut changed the pronunciation of the word.

Digress (v.) to lose track, to go off on a tangent, to stray

Jeanne would constantly digress and find herself telling random stories.

Ostensible (adj.) that which is purported, stated, or appears to be true but need not be

It was certainly ostensible that the "John" of Revelation was the same John mentioned in other parts of the New Testament.

Provident (adj.) tending to timely prepare for the future

Phyllis was very organized and provident, always making sure the necessary arrangements were made.

Timorous (adj.) afraid, fearful, lacking confidence

Allyson had been very timorous about singing in public when she was younger.

Jocular (adj.) playful, humorous or joking

Mr. Dames was rather jocular when it came to his profession of banking, despite the serious nature of it.

Conflate (v.) to combine or confuse two things, often when they should be kept separate and distinct

Walter knew all too well his friend Jerry's tendency to conflate opinion with fact.

Abrogate (v.) to make something cease to exist, to abolish or repeal

The new administration issued an order which abrogated the old rule with regard to stem-cell research funding.

Condone (v.) to allow or approve something

Cindi would condone spanking and small physical violence, but nothing more.

Aver (v.) to claim or allege (especially in a complaint that begins a case in court)

The letter went on to aver that Mary Ann's will had been stolen and destroyed.

Impute (v.) to cause to be possessed or assigned to another person or entity

Liability for the employee's malfeasance was imputed to the employer, and ultimately to the entire corporation.

Symmetry (n.) the state of being equally balanced or reflective

The flounder is an example of a fish which lacks symmetry.

Sundry (n.) an item not important enough to be mentioned specifically; a miscellaneous item

A sundry like cornstarch would probably be on some random aisle near the baking products, she figured.

Mendicant (n.) one who begs for money

Yet another mendicant asked Claire for some spare change on her way to the bus stop.


Vocabulary List 6

GRE Vocabularly List 6 edit

Heterodox (adj.) lacking conformity with accepted standards or practices

Though discussed in some medical texts, homeopathy is still seen as a rather heterodox form of treatment.

Parsimonious (adj.) tending to not spend money; frugal

Though generally parsimonious, Scott spent top dollar on getting the right mattress for his bedroom.

Verisimilitude (n.) the state of appearing to be true or correct

The verisimilitude of Evan's statements would not go unquestioned, and they would ultimately be proved to be false.

Saturnine (adj.) having gloomy and dark mannerisms

The saturnine mortician tended to creep out potential customers.

Corrigible (adj.) something which is capable of being made right

Though the harm to the shipment of goods was severe, it was corrigible.

Sanction (n.) a penalty for disobeying a rule or principle; (v.) to levy with a sanction

The U.N. declared that a sanction be issued against each disobedient rogue state.

Solder (n.) (1) a metal which melts at a low temperature which is used to repair other metallic things; (2) something which causes two things to unite; (v.) to repair with solder

Andy's least favorite part of electrical repair was having to solder wires. Subtle (adj.) difficult to notice or observe without giving great attention

Roger's request was rather subtle and perhaps overlooked by whoever read the letter.

Idiosyncrasy (n.) a nuance or behavior which is unique to a particular individual or group, often which has no apparent reason or rationale

Not wanting to eat oranges was not Mr. Hepburn's only idiosyncrasy.

Erudite (adj.) having broad scholastic knowledge

Though they were both intelligent, Maria, with her lengthy education, was far more erudite.

Desiccate (v.) to dry out, drain out, or dehydrate

The hot weather would desiccate every stream in the region.

Sodden (adj.) (1) soaked or saturated with a particular liquid; (2) intoxicated by the consumption of alcohol

The mattress was sodden after sitting in the rainstorm for hours.

Furtive (adj.) stealthy or sly

With his devious smile and furtive tactics, Mr. D'Angelo wondered what the boy had planned for his daughter that night.

Eclectic (adj.) drawing influences from many different sources

South Philadelphia had a very eclectic past.

Peregrination (n.) travels or wanderings about

The couple went on to write a book about their years of peregrination.

Shambolic (adj.) lacking organization, order or management; in shambles

The planning commission's organization was utterly shambolic and in chaos.

Exigent (adj.) demanding prompt attention

While normally Clarence would've just waited until Monday, exigent reasons compelled him to come into the office on Sunday.

Desultory (adj.) lacking a purpose or plan, having no motivation

Chris and Matt's management of the wielding shop was rather desultory and uncoordinated.

Saturate (v.) to fill or soak until incapable of being filled or soaked further

The company went on to saturate all of South Florida with advertisements and promotions in order to best get its name out.

Iniquity (n.) that which is immoral or unfair

The underrepresentation of women in the professional fields struck Anne as a most severe iniquity.

Pariah (n.) a social outcast

Uma had been a pariah since the incident at the wedding.

Stimulant (n.) a substance that raises the rate of bodily physiological activity

Caffeine is perhaps the most commonly ingested stimulant.

Disconsolate (adj.) unhappy, depressed, or downtrodden

Brandy was rather disconsolate after the football team lost to Barnesville.

Chagrin (n.) a state of discontent or distress caused by one’s shortcomings

Much to Nate's chagrin, the store did not sell the brand of peanut butter he liked.

Fitful (adj.) irregular or unpredictable

Though it was supposed to come every hour, the Freret Street bus was rather fitful and unreliable.

Diffident (adj.) hesitant to speak through lack of self-confidence

Kelsey was quite intelligent, but was very diffident and had a difficult time speaking in front of people.

Sullen (adj.) having an unpleasant or depressing temper or personality

Though usually sullen, on that day, Antonella was the happiest person in Blakely.

Obfuscate (v.) to cause to become unclear, convoluted or incomprehensible

Jackie quickly found out the location to which Billy had absconded, despite his efforts to obfuscate his recent activities.

Descry (v.) to locate in one’s sight; to find out or learn

With a trip to the library, she was able to descry that "Hibernia" was in fact the Latin name for Ireland.

Magnanimous (adj.) showing great generosity or kindness, often toward those who would not expect it

The two found the judge to be rather magnanimous in his sympathy and understanding towards them.

Pertinent (adj.) relative or informative on a given issue

The police officer did not find the eyewitness' account to be particularly pertinent.

Admonish (v.) to advise, warn or reprimand

The chaperon admonished the two not to do that again.

Recreant (adj.) lacking courage; cowardly; (n.) one who is recreant

Theresa was rather recreant about fixing things around the house by herself.

Remittance (n.) (1) money sent in payment for goods or services or as a gift; (2) the act of sending a remittance

The employees were all instructed that the company would send them remittances to cover their expenses on the trip.

Exculpate (v.) to show a lack of guilt

Camera footage from the gas station proved to exculpate the two, and the charges against them were dismissed.

Coalesce (v.) to join into a single thing, to merge or agglomerate

Sarah sat back and let all the ideas she had just read about psychology coalesce in her mind.

Capitulate (v.) to give up, surrender or cease resistance

After Lee's surrender at Appomattox Court House, the Confederates would finally capitulate .

Punctilious (adj.) extremely attentive to detail and propriety

Though he himself had overlooked it, his punctilious assistant noticed a mistake in the numbers on the form.

Imperturbable (adj.) difficult or impossible to upset or disturb

They both tried to annoy her to the point of quitting her job, but Melinda was imperturbable.

Docile (adj.) gentle or submissive

The manatee is a docile creature.

Egress (v.) to exist

The band formed by Chris, Marty, and Nate, would egress for several years before ultimately dissolving.

Decorous (adj.) something which is appropriate and in good taste; something which exhibits decor

The room was full of decorous furnishings, all in a beautiful red and white color scheme.

Insensible (adj.) (1) lacking reason or logic; (2) numb; unable to feel

In Pat Buchanan's opinion, the decision to invade Iraq was insensible.

Bifurcate (v.) to split something into two

The freight team manager decided to bifurcate the tasks, designating one team for each.

Irresolute (adj.) uncertain or hesitant

Erica was irresolute about whether or not she wanted to marry Todd.

Succinct (adj.) clear and comprehensible, typically with regard to written language

The committee loved Henry's beautiful, succinct summary of the plan.

Iconoclast (n.) one who attacks or criticizes longstanding or beloved ideas or institutions

Kenny had by now gained a reputation as a local iconoclast, his latest book attacking religion and other traditional ideas.

Lachrymose (adj.) tending to induce crying; sad

Though a good movie, it was terribly lachrymose, and Catherine could not bear to watch it anymore.

Banal (adj.) lacking originality to the point of being obvious and boring

Meredith was not willing to sit through yet another banal B-movie.

Fathom (n.) a unit of measurement used by operators of watercraft equal to six feet, used to express the depth of a body of water; (v.) to comprehend or imagine

Walter could not fathom the damage that would be caused if the levees in Devils Lake broke.

Prevarication (n.) a statement which deliberately misrepresents the truth; a lie or exaggeration

The landlord's statement that it was a safe neighborhood turned out to be more than a small prevarication.

Squander (v.) to waste or recklessly allow to be lost

Though expected to squander his parents' wealth, Billy in fact went on to achieve great success in the business world.

Dearth (n.) a shortage, scarcity, or amount which is inadequate

There was a dearth of clean water in the city after the hurricane, and they found themselves drinking a great deal of soda.

Prudent (adj.) exercising good and reasonable judgment and consideration

Dawn thought that the prudent thing to do was to explain to Austin that what he had done was wrong, but not to punish him any further.

Apostate (adj.) having abandoned or renounced religious faith or some other loyalty

Yvonne had been apostate with regard to Christianity since her late teens.

Aggrandize (v.) to make something become greater or appear greater

Kelly loved to aggrandize even the smallest of her achievements.

Ersatz (n.) a lower quality substitute or imitation

According to Jonathan, Treet was just an ersatz of Spam.

Dogged (adj.) persistent and determined

Mike was the most dogged worker in the entire department, often staying late to get projects done.

Conspire (v.) to secretly agree to do something, often which is immoral or illegal; to form a conspiracy

Roman hoped he could get Theron to conspire with him with regard to the April Fool's Day prank.

Influx (n.) arrival or entry of a significant amount of something

The influx of Czechs into Vienna in the late 1800s had a dramatic effect on the culture of the city.

Inscrutable (adj.) unable to be understood or interpreted

Though initially inscrutable, after Anderson's work, the meaning of the cave writings began to be somewhat understood.

Virulent (adj.) highly dangerous or infective, usually with regard to a disease

A most virulent strain of influenza swept the world in the early 1900s.

Solicitous (adj.) demonstrating eagerness or strong interest

At the job interview, Miriam came off as very solicitous, but also as very naive.

Boisterous (adj.) having rowdily high spirits

Hockey crowds were ordinarily boisterous, and the crowd present that day was no exception.

Distrait (adj.) distracted or diverted in terms of attention

Regan was distrait for most of the lecture because she was worried about her little brother being stuck at the border station in Michigan.

Vitality (adj.) the state of being strong, health and active

Though twelve years old, the couple's pet poodle still had a great deal of vitality.

Subjective (adj.) based on or affected by an individual’s personal opinions, beliefs and nuances

While the essay grading was entirely subjective, the multiple choice sections were not, and thus assured that there would be some level of objectivity.

Intemperate (adj.) lacking moderation; indulgent or impulsive; lacking temperance

Milt was rather intemperate, and would often yell and scream when he didn't get his way.

Expiate (v.) to make amends or restitution for something

Over a year later, Steve showed up to expiate with regard to his behavior at the family picnic.

Wend (v.) to slowly or indirectly travel in a given direction

The two would ultimately wend their way up through Arkansas to Cape Girardeau.

Corporeal (adj.) having a physical form or existence

Though Janice could look after all of her deceased mother's corporeal property, she wondered what other rights she had, such as a potential stake in her brother's business.

Cunning (n.) a personality trait of cleverness or ingenuity; (adj.) that which displays cunning

Daniel's cunning was all too apparent when he built a program to automatically double-check page number references in documents.

Vilify (v.) to create the impression that a particular person or thing is the source of a problem; to make another a villain in a situation

The American media went on to vilify the Iranian head of state with regard to his comments on Israel.

Anachronism (n.) something which exists out of its proper place in time

Michelle loved literature that contained the occasional anachronism, such as a medieval-style fairy tale which would mention cellphones or Twitter.

Striated (adj.) to marked with lines or linear indents, grooves or stretches

The side of the cantaloupe was striated; no one was sure why.

Probity (n.) the state of having good morals and ethics

Father Gregory's probity was well-known in the neighborhood as well as throughout the city.

Pejorative (adj.) conveying contempt or disapproval; (n.) a word or remark which is pejorative

Even Tony would sometimes describe himself using pejorative words for his profession.

Unfounded (adj.) lacking any sound basis in fact

Though interesting, Cliff's claim that the Book of Acts was written by a woman was for the most part unfounded.

Tautology (n.) something which is repetitive, circular or self-establishing

Eddie loved to say the occasional tautology, such as "you're either in or your out."

Perennial (adj.) enduring or occurring year after year; often in the context of a plant; (n.) a plant which is perennial

Backyard flooding was a perennial problem for the Kennedys.

Converge (v.) to unite, meet, or come together

The old streetcar lines went on to converge at Canal Street.

Innocuous (adj.) incapable of causing anything undesirable; harmless

Sheila worried that the vaccine was not entirely innocuous and that it perhaps would cause harm to her little boy.

Vestigial (adj.) (1) that which is a very small remnant of something which was once much greater; (2) that which while once useful and serving a purpose is now not used, especially in the context of organs

The vestigial parts of whales and dolphins such as their pelvic bones strengthen the theory that they are descended from land creatures.

Anthology (n.) a collection of pieces of art, literature, or music, often but not necessarily all-encompassing

His mother loved to put on her Beatles anthology and crochet.

Inured (adj.) (1) having taken effect; (2) having become used to something, often which is undesirable

By now, Kwame was inured to the poor conditions of the streets in Central City.

Bestial (adj.) of or relating to beasts; beastly

Despite his hairy and bestial appearance, Rick was a very nice and well-mannered person.

Impecunious (adj.) poor; lacking money

Freddie agreed to help Samantha with her taxes, despite how very impecunious she was, out of the goodness of his heart.

Ephemeral (adj.) lasting only a very short amount of time

The ephemeral "Roman Republic" created by Napoleon had a flag similar to the French flag, albeit with a black stripe instead of blue.

Quintessential (adj.) the most perfect example of something

Steven was the quintessential hipster.

Opprobrium (n.) public disgrace that results from shameful conduct

The excessive use of force in the course of riot control measures led to a great opprobrium for the police department


Vocabulary List 7

GRE Vocabulary List 7 edit

Wretched (adj.) that which is extremely malicious, contemptible or distressing

Jack had long hoped for a swift end to the wretched war.

Plumb (n.) a suspended weight used to determine whether something is vertically straight; (v.) to critically examine

They went on to plumb the stated explanations for not giving any notice before selling the loan into default.

Regale (v.) to entertain, amuse, or pleasure an individual with conversation, food or drink

The couple proceeded to regale at the little corner spot in Hoboken.

Assay (n.) an examination of the characteristics or qualities of something

Mert's assay of the jewelry revealed that it was very fine.

Proponent (n.) one who advocates an idea or course of action

Though a proponent of the "asteroid-impact theory", Kellogg admitted that there were still unresolved questions.

Exscind (v.) to cut or tear something and remove it

The physician proceeded to exscind the tumor.

Vex (v.) to cause distress or worry

The flood warnings would vex the residents of Riverside.

Philanthropy (n.) the contribution of money or effort toward the good of others or society

Warren Buffet's philanthropy was lesser known that his great wealth.

Gossamer (n.) the material out of which spider webs are made

Fairy tales told stories of gossamer laced toadstool shoes.

Tact (n.) adeptness at dealing with others with regard to sensitive or difficult issues

While intelligent with regard to accounting, they were both somewhat lacking when it came to tact.

Deposition (n.) (1) the act of removing some government or public figure; (2) a formal statement or declaration made under oath outside of formal proceedings

The monarch abdicated to avoid his deposition by the angry masses.

Inchoate (adj.) not fully formed; existing only mentally

His evil intentions were, at that point, merely inchoate.

Cosmopolitan (adj.) having familiarity and tolerance with regard to countries and cultures; (n.) a cosmopolitan person

She loved the vibrant, cosmopolitan nature of the big city.

Recalcitrant (adj.) having an uncooperative or difficult attitude, especially with regard to authority; (n.) a recalcitrant person

The store manager was rather recalcitrant when questioned by the detectives.

Arbitrate (v.) to decide a dispute, usually on behalf of others

They agreed to let Austin arbitrate their property division in the course of their divorce.

Porous (adj.) (1) having many small holes or pores; (2) not solid, secure, or effective at retention

The porous substance turned out to be real harvested sponge from the ocean.

Peccadillo (n.) a minor or unimportant wrongdoing

Talking on a cellphone while driving was a mere peccadillo compared to the DUI offense.

Distension (n.) the state of being swollen or extended, often unnaturally

The figure in the painting had a peculiar abdominal distension.

Substantiate (v.) to offer evidence supporting the truth of a matter

Weaver went on to substantiate his claims about global warming with evidence from contemporary research.

Infuse (v.) to fill or insert with something

As a hemophiliac, he had to infuse his blood with Factor VIII on a regular basis.

Meretricious (adj.) that which is appealing by appearance or name but has no significant value

At this point, the company's stock was relegated to being merely meretricious pieces of paper.

Articulate (v.) to put into words clearly and concisely; (adj.) clarity and conciseness with regard to putting things into words

Unable to articulate what he meant, he tried to illustrate using an example.

Impetus (n.) energy or force with which something moves or happens

The moon's gravitational pull is the impetus of the tides.

Ingenious (adj.) demonstrative of great intelligence; clever

Though it seemed ingenious at the time, Clarence's idea of mass mailings was in fact of little use or effect.

Collude (v.) to join, agree, or work together toward some immoral purpose

Keb was oblivious to their intentions to collude with regard to the shady deal.

Lunacy (n.) the state of being crazy or a lunatic; insanity

Her lunacy was apparent in her wardrobe and fashion sense.

Effusive (adj.) outpouring without restraint

The effusive leak in the pipe was a source of substantial trouble for the next week.

Countenance (n.) the state of being calm and composed

She displayed remarkable countenance for someone whose home had just been destroyed.

Stanch (v.) to stop or mitigate the flowing of something

Lisa hoped that the bandage would stanch her sister's bleeding enough to allow her to continue the hike.

Salutary (adj.) creating some positive or beneficial effect or outcome

His salutary efforts to create a public park in the region finally paid off.

Encapsulate (v.) to enclose something inside of something else

Sherry's mother would encapsulate each pill she gave her daughter in something sweet so that it could be eaten easily.

Archaic (adj.) old and fallen into disuse; exemplary of a more primitive time

To many, it was a somewhat archaic law.

Depreciate (v.) to lose value

While he knew his farm equipment would depreciate, he did not know just how much.

Satiate (v.) to fully satisfy

The monetary payments were enough to satiate their needs for the time being.

Apocryphal (adj.) having mysterious or doubtful authenticity or origin

Penny didn't know what to make of the apocryphal texts.

Impose (v.) to cause to be accepted or put in place, often with regard to something undesirable

The government proceeded to impose a total ban on the drug in the 1980s.

Exonerate (v.) to clear from liability or wrongdoing

The DNA evidence would ultimately exonerate him.

Itinerate (v.) to travel about

Michael made plans to itinerate around Midtown for the morning.

Prune (v.) (1) to trim vegetation by carefully cutting dead growth and overgrowth away; (2) to remove unnecessary components; (n.) a plum preserved by drying

She took the shears and made her way down to the hedge to prune the shrubs.

Foment (v.) to stir up or rouse, to instigate

The hockey team's loss was expected to foment a riot.

Engender (v.) to cause to exist; to bring into being

She thought that by being difficult to please, she would engender some level of respect and positive treatment.

Capricious (adj.) lacking any clear reason or standard; arbitrary

Given that every other speeder was given a warning, Barry found the officer's decision somewhat capricious.

Ostracize (v.) to cast out, exclude, or banish

Carl wasn't about to ostracize her over the issue, but he made clear his disappointment.

Aspersion (n.) (1) a false or exaggerated charge against a person meant to harm that person’s reputation; (2) a sprinkling a water at a religious or other type of ceremony

The judge maintained that the sexual harassment scandal was merely an aspersion.

Reconnaissance (n.) preliminary surveying, often in the context of military operations

The initial reconnaissance showed that the oxbow lake was about a hundred feet across.

Vacuous (adj.) (1) of or relating to a vacuum; (2) lacking thought, intelligence or personality

Jeffrey was rather vacuous and a difficult person with whom to carry on a conversion.

Apotheosis (n.) a perfect sample; that which has divine or godly status

The basketball team's performance on Saturday was the apotheosis of good teamwork.

Desuetude (n.) the state of not being used; disuse

The room was full of dust and other signs of desuetude.

Unfettered (adj.) having no restraints or restrictions

When her father was gone, her mother had unfettered discretion with regard to how to discipline her.

Viable (adj.) capable of being used or implemented successfully

Most of the senior officers did not believe Tuttle's plan was viable.

Seminal (adj.) strongly influencing later developments

Paul was a seminal figure in the spread of Christianity.

Cabal (n.) (1) the methods and interests of some secret plot, often to overthrow the government; (2) a group partaking in a cabal

Collin wasn't sure who was the mastermind behind their cabal.

Discretion (n.) an ability to choose, latitude, or deference

Polly used her discretion in selecting the appropriate castle for her goldfish tank.

Corrosion (n.) that which is contaminated, weakened, or eaten away

The effects of corrosion on the metal over time were all too apparent.

Conflagration (n.) (1) a large and destructive fire; (2) a conflict or struggle

The ensuing conflagration would engulf all of eastern Nebraska.

Rescind (v.) to revoke, cancel, or repeal, often with regard to a law or edict

The council finally decided to rescind their order about fireworks in town.

Nugatory (adj.) having no use, importance or worth

Tom's nugatory credentials failed to impress anyone. Morose (adj.) ill-tempered or gloomy

Renee was rather morose after her father's death.

Harrow (v.) to annoy or bother

Clarise knew that the boys on the bus would harrow her the entire ride home.

Chicanery (n.) an act of deceiving or fooling a person; trickery

Benjamin's chicanery was, at this point, expected.

Felicitous (adj.) (1) suited to the circumstances; (2) enjoyable and pleasing

Her big hat was rather felicitous given the occasion.

Natty (adj.) trendy and fashionable

Nate was unfamiliar with the natty apparel that people from the city wore.

Impassive (adj.) having or showing showing little feeling or emotion

He was usually impassive, but on that day, he was especially stoic.

Miser (n.) one who hoards money but spends as little as possible

Dickens' "Scrooge" character is a famous example of a miser.

Personable (adj.) easy to get along with; friendly

Elizabeth was not quite as personable as her sister Maria.

Authoritarian (adj.) of or relating to a preference for stronger authority and obedience thereto

George's mother had a very authoritarian mentality.

Anesthesia (n.) the state of being unable to feel or sense something, typically pain

Christopher was so intoxicated his condition could almost be described as anesthesia.

Compliant (adj.) satisfying the requirements of something; obedient

The bank had been compliant with federal regulations ever since it opened.

Anathema (n.) (1) a disliked or unpleasing person or thing; (2) (archaic) a person cursed by ecclesiastical authority

Onions were an anathema to Meredith, who despised their harsh taste.

Obloquy (n.) a harsh and severe statement of criticism

She felt bad after such a vicious obloquy was leveled against her.

Veritable (adj.) the state of truly being something (often used to qualify a metaphor)

After the heavy rain, Clara street was a veritable pond.

Pique (n.) a feeling of resentment or irritation; (v.) to stimulate, to create interest

Trisha sensed some level of pique with regard to Connie and Allyson's feelings toward her.

Prone (adj.) (1) likely to experience something; (2) lying flat on one’s stomach

The park ranger wasn't sure why he was so prone to being stung by wasps.

Fetter (n.) a chain used to restrain a prisoner, typically placed around the ankles; (v.) to restrain, such as with a fetter

The Supreme Court serves to fetter the powers of Congress and the President.

Judicious (adj.) of or relating to the exercise of wise judgment

William was known as an especially judicious arbitrator of disputes.

Polarize (v.) to break into directly opposed groups or categories; bifurcate

It was yet another political issue which would polarize the nation.

Trivial (adj.) having little or no value or importance

While Ms. Lambert thought that retaining customers was top priority, her associates thought it was trivial.

Stygian (adj.) (1) very dark; (2) of or relating to the underworld river Styx in Greek Mythology

Salome was a most stygian work by Richard Strauss.

Seine (n.) a fishing net which drapes down vertically in the water; (v.) to use a seine

He and his brother would go seine for minnows near the creek.

Vitiate (v.) to ruin or destroy the validity, quality or efficiency or something

When Jacquelyn was caught plagiarizing, it served to vitiate her academic credentials for years to come.

Inhibit (v.) to stop or prevent

The mouth guard was designed to inhibit certain types of injuries.

Meritocracy (n.) a society or organization where individuals are assigned duties and tasks based on ability

Michelle regretted how far the world was from being a meritocracy.

Tortuous (adj.) having many twists and turns; excessively complicated and lengthy

The tortuous Mississippi river was difficult for large ships to navigate.

Verbatim (adv.) using exactly the same words as another source

The quote was taken verbatim from one of Shakespeare's works.

Deviance (n.) the state of having gone astray from the usual norm

Her tattoos and piercings were a sign of her deviance.

Exemplar (n.) an ideal sample or specimen

Tony was not an exemplar of the typical Democrat by any means.

Expostulate (v.) to try and convince an individual otherwise; to argue against; to express strong disapproval

Mr. Ninness took his turn to expostulate with regard to the traditional understanding of feudalism.

Insurgency (n.) a rebellion or insurrection against a government

Rumsfeld commented at length about the insurgency abroad.

Noisome (adj.) that which has an unpleasant smell or is otherwise undesirable

Andy wasn't sure exactly what the noisome substance was, but he knew he did not like its odor.

Tout (v.) to promote or play up the positive characteristics of something

The RV salesman proceeded to tout the superior characteristics of the Winnebago.


Vocabulary List 8

GRE Vocabulary List 8 edit

Meticulous (adj.) paying great attention to detail and precision

He was very meticulous when it came to keeping track of his weather observations.

Edacious (adj.) having a very strong appetite

It would not be long, he knew, before their edacious St. Bernard devoured the entire bowl of dog food.

Daunt (v.) to frighten or bother

Worries of not having enough money to pay next month's rent continued to daunt him.

Extant (adj.) existing or real

Within the extant universe, nothing has been shown to be able to travel through time.

Recluse (n.) one who lives a solitary life and avoids others

Mark was something of a recluse, rarely socializing with others.

Sedulous (adj.) exhibiting diligence and persistence

Maria was the most sedulous of any of those designated to work on the project.

Cacophony (n.) an unpleasant mixture of dissonant sounds

The horrid cacophony outside must have been a street fight or similar type of quarrel.

Pervasive (adj.) widespread and continuous, often with regard to something not desired

Homelessness became a pervasive problem in San Francisco after the dot-com bust.

Munificent (adj.) more generous or lenient than is customary or necessary

He was considered to be one of the more munificent of the Ottoman Sultans.

Vexation (n.) the state of being distressed or worried

The current vexation had to do with the loan payments due at the end of the month which the couple was unable to pay.

Tirade (n.)a lengthy and ranting attack or criticism

Shaun's nasty tirade was met with an equally bitter response from his opponent.

Brevity (n.) having a short duration; that which his brief

The brevity of Kennedy's presidency did not mean his legacy would soon be forgotten.

Arcane (adj.) obscure, difficult to understand, or known by very few, often with relation to knowledge

The family had many strange traditions with rather arcane origins.

Erstwhile (adj.) as things were at a prior time; previously

Melinda regretted that some of these erstwhile traditions had not continued.

Homily (n.) (1) a religious speech or sermon (2) a tedious lecture on morals or ethics

Brian was not looking forward to another homily from his mother.

Feckless (adj.) worthless, weak or ineffective

Her feckless dog began to wimper and subsequently ran away.

Subpoena (n.)a document compelling one to appear in court; (v.) to compel an individual via subpoena

Brenton made clear his intent to subpoena all eleven witnesses.

Dogma (n.) an established tenet or belief, such as in religion (typically used by one who does not agree with the belief)

Lorelai would constantly vent about the crazy dogma her mother believed in.

Salubrious (adj.) healthy or in good condition

Monica looked quite alive and salubrious despite the stroke.

Transfiguration (n.) a change in form or appearance, often which is exalting or glorifying

She described a vision wherein there was a strange transfiguration.

Calumniate (v.) to make harmful and malicious statements about a person or thing; to make a calumny

They would sit back in the coffee shop and calumniate about people passing by outside on the street.

Phlegmatic (adj.) demonstrative of calmness and composure

Juliette's phlegmatic behavior was surprising given the tress she was under.

Chimera (1) a fire-breathing monster with a lion’s head, a goat’s body, and a serpent’s tail, described in Greek Mythology; (2) a myth, illusion, or falsehood

The chimera of Carver's invention of peanut butter is still taught in schools today, despite its falsity.

Sporadic (adj.)happening or occurring at a few irregular times or locations

Their eating patterns were quite sporadic, lacking any consistency.

Emollient (adj.) (1) having the effect of making something less harsh; (2) something which has an emollient effect

Her sob story was intended to be an emollient.

Obstinate (adj.) (1) exhibiting stubborn refusal to change an opinion or belief; (2) difficult or frustrating to deal with

While she was obstinate at first, Rachel grudgingly moved her car out of the way.

Maunder (v.) (1) to speak in a rambling or unstructured manner; (2) to wander

The man on the street continued to maunder, and neither of the two were exactly sure just what he had said.

Latent (adj.) not yet fully developed; hidden or dormant

He had many latent characteristics which would not exhibit until much later in life.

Ineffable (adj.) too extraordinary to be expressed by words

Seeing the entire bay from the top of the bridge was an ineffable sight.

Usurp (v.) to take by force, fraud or collusion, often in the context of political office

The Bolsheviks proceeded to usurp power after the Tsar's abdication.

Calibrate (v.) to determine the diameter or caliber or something; to measure something precisely

It was standard procedure to calibrate various measures on the automobile before working on it.

Parody (n.) a copy or imitation of something else for comedic purposes; (v.) to make a parody

The song was a "stylistic parody" of music of the genre.

Gainsay (v.) to take the opposing position, to dispute

He took great pleasure in the gainsay of what his opponent had to offer.

Baleful (adj.) eerily threatening or foreboding

The baleful parking fines were enough to persuade Allison to pay for parking at a nearby lot.

Episodic (adj.) of or relating to episodes; proceeding in or consisting of different stages episodes

His was an example of an episodic novel, even with distinct books within it.

Edifice (n.) a building or structure (typically large)

The edifice was in fact an old warehouse from the neighborhood.

Bolster (v.) to strengthen or support

The eyewitness' testimony tended to bolster the defense's theory of the case.

Dissemble (v.) to take on a false appearance

Hugo often made efforts to dissemble or disguise himself, embarrassed of his job.

Insipid (adj.) lacking any spirit, vigor or vitality

Matty was a lazy and insipid child, preferring to watch TV all day.

Deride (v.) to ridicule someone or something; to pejoratively laugh

Penny was used to how the people at school would constantly deride and insult her.

Compunction (n.) a feeling of guiltiness or remorse

They both felt remarkable compunction after what happened to the neighbor's dog.

Apposite (adj.) highly relevant or appropriate

The wedding was filled with lovely, apposite music, such as Mendelssohn's famous wedding march.

Encomium (n.) overly enthusiastic praise

The latest encomium venerated the book as a great stride in modern literature.

Discordant (adj.) having sounds which do not harmonize together

The chirping birds outside were rather discordant on that given morning.

Caustic (adj.) (1) that which will corrode materials or eat them away; (2) sharply sarcastic or mocking

While he shrugged it off, the caustic criticism truly did hurt his feelings deep down inside.

Contemn (v.) to treat a person with contempt; to display dislike for an individual

As a militant vegan, Sara would continually contemn “animal killers.”

Torrid (adj.) (1) very hot and dry; (2) having powerful emotions stemming from sexual attraction

The torrid weather in the region did not bode well for Mrs. Jackson.

Peripatetic (adj.) tending to travel from place to place, often staying for short periods; (n.) one who is peripatetic

Belinda had been more peripatetic in recent years, living all around New England.

Petulant (adj.) childishly selfish, bratty, or ill-tempered

He wished the neighbors would do more to discipline their rude, petulant children.

Poignant (adj.) tending to cause sadness or unpleasant feelings

It was rather poignant to hear that she was leaving and not returning.

Fetid (adj.) having an unpleasant smell

One whiff of the barn reminded Neil of the fetid odors on his uncle's farm growing up.

Peroration (n.) a conclusion to a speech

Billy ended with a peroration that analogized the current situation to the founding days of the nation.

Ingenuous (adj.) innocent and unsuspecting

Rachael had no idea that such an ingenuous little poodle could be capable of such a vicious bite.

Lassitude (n.) a lack of energy or motivation

Paul's lassitude was apparent during the hunting trip.

Echelon (n.) any one in a sequence of ranked groups

He would ultimately be promoted into a higher echelon than the others.

Coda (n.) an ending, conclusion or finalization, such as in a musical work

She recognized the coda, and nudged her husband to wake him, since the concert was ending.

Proliferate (v.) to increase or reproduce rapidly

Though there were only a few Polish immigrant families in Luzerne county at first, the population of them soon began to proliferate.

Venal (adj.) susceptible to bribery or corruption, often in the context of public officials

Rick was a rather venal individual when it came to his position as representative, and would readily take money from lobbyists.

Opaque (n.) an opaque thing or substance; (adj.) not able to be seen through; not transparent

The glass on the window was opaque.

Invigorate (v.) to increase the strength or energy of something; to give vigor

Andy knew that an energy drink would invigorate him.

Fractious (adj.) having a tendency to cause trouble

Eddie's fractious behavior ultimately got him kicked off campus.

Amalgamate (v.) to merge into one; combine

Ultimately the two jewelry stores would amalgamate into one single brand, which would dominate the local jewelry market for years to come.

Convalescence (n.) the recovery of one’s health after some sickness or injury

Jerry had a remarkably quick convalescence after his fall.

Lucid (adj.) clear and understandable

Meredith gave by far the most lucid explanation of a sonic boom.

Lubricious (adj.) (1) smooth or slippery; (2) offensively sexual

He did not realize how lubricious the oil-slicked sidewalk was until it was too late.

Abscission (n.) (1) an act of cutting something off; (2) the natural shedding of a plant’s leaves or fruits

Marta proceeded to trip the hedge by making a large abscission of its lower leaves and branches.

Normative (adj.) of or relating to what is typical and standard; normal

The two made all the normative arrangements for their wedding, with nothing too extravagant.

Zeal (n.) an eager and passionate desire towards some goal

Her zeal in pursuit of the employee of the month award was all too apparent.

Decibel (n.) a unit measuring the intensity or volume of sound

Whether described by the decibel or the ringing in his ears afterward, Kyle knew that the concert was loud.

Querulous (adj.) making whiny, ill-tempered, or annoying complaints

The querulous branch manager was always nagging on people to follow regulations more closely.

Mellifluous (adj.) pleasant to listen to, typically with regard to spoken words

Though she didn't understand what he was saying, she found his speech mellifluous.

Effigy (n.) a tangible representation of a person such as a doll or stick figure created for some expression of contempt

The police officer would be all but burned in effigy after the town found out what he had done.

Jettison (v.) to rid oneself of something by forcefully ejecting it, often from an airplane or other vehicle

Zelda would jettison her traditional beliefs about marriage in her early 20s.

Prate (v.) to speak at tedious length about something, often unwisely

Kelly would prate for lengthy periods of time about how much she hated her job.

Discomfit (v.) to thwart or frustrate the plans of someone or something

The launch of the new product would serve to discomfit competitors' hopes of dominating the market.

Obdurate (adj.) stubbornly reluctant toward changing an opinion or belief

Her father and uncle were tied for being the most obdurate people in the world.

Jocose (adj.) playful, humorous or joking

Though jocose most of the time, Heather was quite serious that day.

Cower (v.) to back away or flee from something out of fear or fright

Despite claims that he would cower like a cockroach, Jose stood his ground.

Virago (n.) (1) a controlling or ill-tempered woman; (2) a female warrior

Becky was as much of a Virago as Wesley's last girlfriend.

Plod (n.) a slow walk with heavy steps; (v.) to partake in a plod

Their plod along the trail was rather unpleasant given how muddy it was there.

Detraction (n.) that which belittles or reduces the magnitude of something else

To Todd, the plot of the musical was really only a detraction from the lovely orchestration.

Perjury (n.) dishonesty under oath

Though he knew it was wrong, he did not know it to be perjury.

Attenuate (v.) to make thinner or weaker; (n.) that which is thinner or weaker

Each minute waiting would increasingly attenuate his patience.

Fracas (n.) a noisy dispute or fight

The loud fracas at the restaurant was provoked by an insulting comment between different tables.

Frieze (n.) (1) a pile of uncut loops; (2) an ornamentation on a building; (3) coarse wool

The frieze on top of city hall displayed a farmer and an ox with a plow.

Propensity (adj.) a natural tendency to do something

While all the children got into trouble, Pete had the greatest propensity for mischief.

Limpid (adj.) clear and not cloudy or murky

The limpid contents of the beaker were nonetheless not presumed to be water.

Fatuous (adj.) silly or pointless

The plot of the operetta could best be described as fatuous.

Arrant (adj.) not giving the proper respect, excessively bold, rude, insolent

He wasn't the first arrant young boy to have quarreled with the police officer.

Disseminate (v.) to spread or disperse

Lenin's followers would then disseminate his ideas throughout the party.


Vocabulary List 9

Vocabulary List 9 edit

Filigree (n.) metalwork jewelry or ornaments, typically made of threads of silver of gold

The typical filigree sold at the store was very ornate.

Veto (n.) an absolute right to overrule others’ decision, often in the context of politics and government; (v.) to the act of utilizing the power of veto

The veto power of the President is set out in the Constitution.

Suppliant (adj.) making a humble plea to one with power or authority; (n.) one who is suppliant

Their suppliant request was received by the director of activities, who acceded to their demands.

Eccentric (adj.) flamboyant, unusual, or defiant of convention

Though all the brothers were eccentric, Billy was by far the most odd.

Wheedle (v.) to attempt to persuade, often employing desperate measures such as acts of flattery

Though he was unqualified for the job, Eddie thought he might be able to wheedle his way into getting them to hire him.

Discursive (adj.) of or relating to discourse

The discursive texts on the issue of the authorship of Shakespeare's plays lacked any uniform alternative theory.

Disaffect (v.) to cause an individual to cease liking someone or something; to undo any affection

He did not know just how much the incident would disaffect him with regard to Thai food.

Gambol (v.) to run or frolic in a playful manner

The children proceeded to gambol about as their parents waited.

Plethora (n.) a large number and variety

Though he had a plethora of excuses, none of them were satisfactory in the eyes of Mrs. Mitchell.

Abjure (v.) to solemnly renounce a belief or claim, to make something go away

She had every intent to abjure her memories of that summer.

Picaresque (adj.) of or relating to a roguish but likeable protagonist, often in fiction

Hemingway's The Old Man and the Sea contains an example of the picaresque hero.

Ponderous (adj.) (1) excessively dull or tedious; (2) awkward and clumsy due to heaviness

The ponderous schoolwork annoyed Gabrielle.

Tumultuous (adj.) excitedly lacking order and confusing

The two had a tumultuous relationship for many weeks after the fight at the Bulldog.

Intestate (adj.) having not made a will upon death; (n.) one who dies intestate

Grandpa Hal died intestate.

Mundane (adj.) typical; boring

The mundane math problems in the textbook were never-ending.

Vaunt (v.) to brag or boast

Like every other car dealer, he would vaunt the capabilities of his own product and trash talk those of the competition.

Rhetoric (n.) persuasive or inciteful language, often which is lacking in substantive content

The speaker continued with his inflammatory rhetoric.

Arabesque (n.) a pattern of elaborately interwoven lines, often utilizing plant or animal outlines

Delilah loved the arabesque on the handbag, and had no choice but to purchase it.

Hallow (v.) to make or venerate as holy; consecrate

The church went on to hallow St. Martha, and her legend lived on.

Beneficent (adj.) that which his good, moral or helpful to society, especially in the context of charity and public service

The Jesuits are well known worldwide as a beneficent order.

Adorn (v.) to cause to be more appealing or attractive

The nation would adorn his uniform with medals for his war efforts.

Assiduous (adj.) done very carefully or persistently with great attention

Carrie's assiduous efforts paid off when she was offered the contract.

Acarpous (adj.) not producing any fruit

Though most of the apple trees were healthy, there were a few sickly and acarpous trees near the fenceline.

Improvidence (adj.) having little thought for the future when making decisions

She was intelligent but a poor planner, displaying remarkable improvidence.

Hirsute (adj.) hairy

The hirsute fellow's dreadlocks disgusted Sandy and her husband.

Arcane (adj.) secret or hidden; known only to a few

The recipe for Swedish cheesecake had become quite arcane over the years.

Feign (v.) to assert an untrue impression; to fake or pretend

Friar Lawrence gave her a potion designed to feign death, after the effects of which she would wake up.

Expurgate (v.) to cleanse or purge of something wrong or evil

Truman felt a day at church was needed to expurgate himself after what he had done.

Derivative (n.) that which stems from or is based on something else; that which is derived

She couldn't believe that such a nasty drug was a derivative of the opium poppy.

Eschew (v.) to consistently avoid, often on religious or moral grounds

Holly would consistently eschew drinking alcohol.

Euphemism (n.) a word or phrase, often large or lengthy, used in place of a more ordinary term to invoke a more positive connotation

"Loss prevention associate" was just a euphemism for "security guard."

Heretical (adj.) dissenting or departing from an accepted belief or standard

Huss was vilified for his allegedly heretical statements.

Precarious (adj.) uncertain or insecure; likely to fall or collapse

The mayor's office found themselves in a rather precarious position, with public support beneath them wavering.

Extravagance (n.) the state of being excessive or overdone, usually in the context of money

Donald Trump's extravagance is well known.

Anodyne (adj.) (1) pain-relieving; (2) unlikely to offend or upset

Though Mary Ann worried she'd upset the others, her comments were in fact rather anodyne.

Polyglot (adj.) having mastery of or capable of using multiple languages; (n.) a polyglot individual

Jimmy was such a polyglot that he even knew Swahili.

Caret (n.) the symbol “^” which denotes that something is to be inserted

Every caret on the document indicated a word that Sylvia needed to fill.

Indelible (adj.) incapable of being removed or erased

The exact name on the document could be erased, but the ink was indelible.

Egregious (adj.) noticeable or serious, usually used in a negative context

The most egregious part of it, in Laura's opinion, was the fact that he never even apologized.

Inimical (adj.) hostile, contrary or harmful

Her behavior was inimical to the best interests of the Republican Party, which subsequently distanced itself from her.

Dissent (n.) a contrary opinion or disagreement; (v.) to express dissent

While most of the directors agreed to the new policy, Tollefson did not, and he made sure his dissent was noted.

Stentorian (adj.) loud, forceful and powerful, typically with regard to a person’s voice

The playground aide's stentorian voice echoed for a wide radius, ordering the children to return inside.

Frugal (adj.) resistant to spending money, thrifty

Pierre's frugal lifestyle earned him the nickname "Penny Pierre."

Sanctimonious (adj.) embellishing the belief that one is morally superior to others

While both Luke and Kate were rather upstanding members of the community, only the former was sanctimonious about it.

Waver (v.) (1) to shake or tremble; (2) to give in or compromise

Though pressured, Phil never wavered in maintaining his innocence.

Poseur (n.) one who pretends, often regularly, to be something which that person is not

With her lengthy collection of French novels and cookbooks, Katy - an American - was something of a poseur.

Celerity (adj.) done with quickness or rapidity

The two of them fixed the '71 Chevelle with remarkable celerity.

Welter (v.) to move about in disarray (n.) a number of things in disarray

The welter filling her room was certainly a sight to see.

Importune (v.) to inquire, often persistently or with a strong desire

Shelly would constantly importune with regard to whether there were any job openings at the restaurant.

Burnish (v.) to smooth or shine something by rubbing it

He always made it a point to burnish the silver.

Dissertation (n.) a long discussion of a subject

Victoria's dissertation discussed the change in the law of the Republic of Venice as the result of the influx of commerce.

Temper (v.) to improve the durability of something through some process

An accomplished blacksmith, he went on to temper the metal with his hammer.

Indefatigable (adj.) incapable of becoming tired

Though others slowly became weary in the marathon, Gregory was indefatigable for its entire duration.

Coeval (adj.) having the same age

The coeval samples had strikingly different aspects, despite having the same age.

Somber (adj.) sad, depressing, mournful

The background music on the film had a somber tone.

Maverick (n.) a rebellious or unorthodox person

The senator repeatedly touted his reputation as a maverick.

Obsequious (adj.)excessively obedient or attentive

Harold was by far the most obsequious of any of the office employees.

Implicit (adj.) something which is a necessary or likely consequence or inference which is not directly stated; something which is implied

It was implicit in Kenny's dissertation that he was hostile to organized religion.

Abscond (v.) to leave or disappear in a quick and secretive manner

She suspected her husband's intentions were to abscond with the couples' child.

Profligate (adj.) recklessly extravagant, indulgent, or wasteful; (n.) one who is profligate

Miguel's profligate spending called a lot of attention to him.

Sophomoric (adj.) (1) of or relating to the second in a series, typically of years in school; (2) rude and insubordinate in a manner reminiscent of a juvenile

His sophomoric opinions were an annoyance to the others in philosophy class.

Provocation (n.) an act intended or having the effect of prompting or stimulating another’s course of action

Roger claimed that the attack was completely without provocation.

Vernacular (n.)a language or dialect specific to a region or group; (adj.) of or relating to a vernacular

A native England, she had a tough time adjusting to the Massachusetts vernacular.

Descant (v.) to talk tediously or comment at length

She proceeded to descant about the issue of clean water in the community.

Ambrosia (n.) (1) food of the Greek or Roman deities; (2) that which is very tasteful

Pad Thai was Mr. and Mrs. Reynolds' personal ambrosia.

Cynical (adj.) excessively negative or pessimistic, often with regard to a person’s outlook or worldview

While not cynical per se, Claudia often had a very negative attitude.

Exegesis (n.) an explanation or giving of reasons, often of a literary work

Eddie's compilation of war memoirs was introduced by an exegesis in the first few pages.

Forestall (v.) to prevent, exclude, or hamper

The harsh winter proved to forestall their efforts to fix up things around the shed.

Ineluctable (adj.) unavoidable; irresistible

Ethel found the smell of fresh apple pie ineluctable.

Untenable (adj.) incapable of being supported, especially with regard to an argument or contention

In his opinion, any argument for gun control was simply untenable.

Onerous (adj.) prone to cause trouble

Knowing Haley's onerous behavior, he figured it was only a matter of time before she called needing help out of some situation.

Auspice (n.) (1) a sign of what is to happen in the future, sometimes based on watching birds; (2) patronage, support or guidance

He completed his training under the auspices of Father Murphy.

Censure (n.) condemnation or negative opinion; (v.) to censure

The censure of A Clockwork Orange was widely known at the time it was published.

Captious (adj.) having the tendency to disagree, object, argue, or be difficult

Bill was always captious, whether or not he was talking about politics.

Dross (n.) (1) a scum that forms on the surface of molten iron; (2) something that is impure or worthless

Judy was sick of hearing about the sappy plots of the romance novels and such other literary dross that her mother would buy at the supermarket.

Wizened (adj.) shriveled or wrinkled as the result of the passage of time

The dog's wizened face reflected its age.

Acerbic (adj.) (1) sour or bitter in taste; (2)(in speech) straightforward, harsh, uncompromising

Barbara didn't understand how they could all drink such an acerbic, rancid beverage.

Elegy (n.) a sad poem or song, often written to mourn the dead

The flautist and cellist played a beautiful elegy at Clint's funeral.

Din (n.) a loud and often dissonant emanation of noise or noises

The din from the gymnasium could only be the result of a basketball game.

Resuscitate (v.) (1) to revive from unconsciousness; cause to breathe again; (2) to bring back to life or existence, often in the context of an idea

His book would attempt to resuscitate the some of the laissez-faire ideas of the late 1800s.

Tendentious (adj.) promulgating or intending to promote an idea or viewpoint, often which is controversial

Bennett's book on the Kennedy assassination was, as to be expected, tendentious.

Tangent (n.) divergence from a prior course or direction

Mrs. Makela subsequently went off on a tangent about her pet cat.

Expedient (adj.) effective in achieving some particular result, often which is practical but unethical; (n.) something which is expedient

Though an expedient solution, Mr. Kervin wondered if it burning all the garbage was really the best thing for the air.

Unflappable (adj.) having a state of calmness; not wavering, especially during some crisis

His spirit proved unflappable, even through the depths of the recession.

Adumbrate (v.) to outline, sketch or represent

The report went on to adumbrate the layout of the apartment where the incident took place.

Defer (v.) to abstain or pass on an action or decision and direct it elsewhere

Jamarcus elected to defer to his friends' decision with regard to what type of pizza to order that evening.

Rebus (n.) a puzzle, riddle, or other writing in which words are represented with pictures

The rebus sentence, while initially decipherable, was easy to figure out.

Emulate (v.) to copy or try to be more like something; to draw inspiration from something

He often tried to emulate many of his heroes from the World War II generation.

Treacherous (adj.) (1) culpable with regard to fraud, deceit or betrayal; (2) posing great risks

The first day of the mountain climb was by far the most treacherous.

Ossify (v.) (1) to turn into bone; (2) to stagnate or cease in terms of progress or development

The merger began to ossify after Lance quit his job and left no one to manage or coordinate it.


Sentence Completion Strategy

Sentence Completion Strategy edit

Determine what the word in the sentence is doing, and then define the words in the blanks to see which matches.

7. The development of colored dyes would ultimately _____ to the benefit of the middle class artisans who became _____ at using them and _____ their very utility.

(A) inure (D) adept (G) exploiting (B) tirade (E) instilled (H) traversing (C) engulf (F) aggrandized (I) corroborating

For example, the first blank above is describing what the dyes did with regard to the middle class. The correct answer is “inure”, which means to go towards something. The second blank is describing the artisans' disposition toward using the dyes. In this case, the correct answer is “adept”, meaning skill. Finally, the third word is describing what the artisans did with regard to the utility of the dyes. The correct word is “exploiting”, which means using or utilizing something.

As can be seen, sentence completion comes down to knowing what the words mean. The following pages contain a series of “quiz lists” to practice the vocabulary words from the previous pages.


Introduction to GRE Reading

There is no secret, trick, or formula to GRE reading passages other than to read them steadily and carefully, and to practice reading them.

Read word-by-word, sentence by sentence, understanding each concept and connecting it to the others.

Reading questions will typically ask for the meaning or purpose of a word or phrase, or of the passage itself. They may also ask to apply a concept discussed in the passage, or what must be true based on the passage. Sometimes questions will have three choices with more than one potential correct answer - this will be clearly indicated.

Economists frequently elaborate upon the imbalance that occurs when two previously isolated markets are suddenly connected, allowing a new flow of goods 5 or services from a market in which they were in greater abundance, to one in which they are less plentiful - to the disruption of current economic activities there. Perhaps nothing from history more explicitly 10 illustrates this concept than the trans- Saharan gold trade initiated by the Kingdom of Mali under Mansa Musa. Trade across the vast desert had occurred since ancient times, as 15 documented in chronicles of caravans in the Nile Valley. However, Musa's expansion of trade routes to the Mediterranean coastline and ultimately the Middle East and Europe were an 20 unprecedented achievement in global trade for their time, and their impact on previously insular economies cannot be understated. The inflow of gold from Africa's “Gold Coast” was particularly 25 tumultuous in economies such as that of Egypt, where gold was a standard unit of currency. The new trade expansion may have paved the way for further developments in commerce between 30 distant parts of the world which ultimately manifested in the voyages of Da Gamma and Columbus. Look for the “hard words” such as subjects and verbs in passages such as the one to the left.

Be ready to refer back to particular lines of the passage

The GRE frequently points to a concept by its particular line; thus, keep track of these lines.

Practice

Read the passage to the left and try answering the following questions.

1. Based on the passage, what is the relationship between the Kingdom of Mali and Mansa Musa, and the economic concept discussed in lines 1-7?

2. Describe the effect that gold trade had on places such as Egypt, based on the passage.

3. Why does the author of the passage mention Da Gamma and Columbus (line 32)?

Answers to Practice Questions

1. The author mentions Mali and Mansa Musa as an example of the concept stated in the first sentence. “Mali” is a place in Africa which was a kingdom many years ago. Mansa Musa was a king of Mali. The GRE will frequently mention things of this sort and not explain what they are, expecting the reader to infer or guess at its passage topics.

 The concept mentioned is that sometimes two economies are linked, and sometimes a good is more plentiful in one than the other.  This means that the economies may be disrupted.  The author explains that this is what happened when gold from Mali was traded across the Sahara to Egypt and other locations.

2. According to the author, the influx of gold from Mali and the “Gold Coast” upset the economy in Egypt particularly severely because gold was used as a unit of currency. With a new oversupply of gold, the economy was disrupted.

3. Da Gamma and Columbus were explorers. (Da Gamma was the first European to sail to India). The author mentions them because the author thinks it is possible that the trans-Saharan trade under Mansa Musa inspired future developments in trade, and may have led to merchants and rulers commissioning Da Gamma and Columbus and their voyages.


Main Points

Make sure to completely understand the scope and extent of the author's main points in the passage.

The GRE will have “trap answers” which try to “suggest things” to you. Often they will mention things that were in the passage, but not in the manner mentioned.

In the real world, passages typically set out their topics at the beginning. On the GRE, however, the real subject of the passage is often confusing to find, as evidenced here:

	  The armadillo is the only extant
 	member of order Cingulata surviving to
 	date, and exists throughout the Americas
 	in the form of various subspecies.  Little 
5	known is the fact that an ancient cousin
 	to the armadillo, the glyptodon, also
 	inhabited the armadillo's current range
 	until very recently, dying out at the end
 	of the most recent ice age.  Like its 
10	modern relative, glyptodon was a
 	placental mammal with a thick armored
 	shell, and was probably capable of
 	tucking into a ball or something similar
 	to protect itself.  However, unlike 
15	armadillos, which are typically about the
 	size of a small dog, glyptodons grew to
 	sizes of perhaps five meters in length,
 	making them one of the largest land
 	animals known to coexist with humans.
20	  There has thus been widespread 
	speculation as to whether the indigenous
 	peoples of the Americas hunted the
 	glyptodon, much of which has been met
 	with skepticism given the creature's size 
25	and presumable power.  However,
 	ancient humans are known to have
 	hunted other highly dangerous creatures
 	such as the woolly mammoth, which were
 	arguably even more dangerous with 
30	their great tusks, as well as the fact that
 	mammoths were social creatures
 	tending to roam in packs and protect one
 	another.

Practice

1. The author discusses the armadillo in lines 1-4 primarily in order to
(A) explain why the armadillos survived the last ice age while glyptodons did not
(B) cite an example of another animal which was hunted by early humans
(C) elaborate on characteristics unique to placental mammals
(D) give an example of the nearest modern relative of a creature it is about to discuss
(E) point to another instance of a social animal which roams in packs

2. Which one of the following best describes the author's attitude toward the notion that ancient humans might have hunted the glyptodon?
(A) skepticism given the creature's size and great power
(B) receptiveness given the fact that ancient humans hunted the even more powerful and social woolly mammoth
(C) dismissal on the grounds that the wooly mammoth would've been superior as a food supply
(D) uncertainty due to lack of evidence that the glyptodon roamed in packs
(E) acceptance based on the current documented evidence

Answers to Practice Questions

1. (D)

This passage is about the glyptodon, which is a giant armadillo-like creature, which died out in prehistoric times. The first paragraph describes the glyptodon, while the second discusses the theory that ancient humans may have eaten them. The discussion of armadillos serves to introduce the glyptodon by describing the nearest modern relative of the creature, as occurs in answer (D).

The other answers are typical reading passage trap answers. Choice (A) is incorrect because, while the ice ages are mentioned as well as the glyptodon's extinction at the end of them, this is not the topic the author seeks to explore in the passage. The author is merely mentioning that there was such a creature as the glyptodon and that early humans may have hunted them. Similarly, choice (B) is incorrect because, while the author is talking about hunting, the author never talks about humans hunting armadillos. Rather, in the second paragraph, the author is contemplating whether humans ever hunted the glyptodon.

Answer (C) is incorrect; although “placental mammals” are mentioned, the author never explains what this term means or what else is included in this category. Finally, (E) is incorrect - while the author discusses “social animals”, the author discusses this in the context of woolly mammoths and not the armadillo.

2. (B)

The author thinks it is plausible that ancient humans hunted the glyptodon, the main reason for believing this being that humans hunted powerful and dangerous creatures like the woolly mammoth.

Choice (A) is a trap answer - while the author mentions “skepticism”, this is not the author's skepticism, but rather, that of other individuals. Answer (C) is similarly incorrect because, while glyptodons are compared to woolly mammoths, this is not to illustrate differences in their availability as a food supply, but rather to say that if humans hunted one, they probably hunted the other. (D) is incorrect because there is no lack of evidence - the author doesn't discuss sufficiency of the evidence. Finally, (E) is wrong because it is too strong - the author doesn't “accept” the hunting of glyptodons but rather feels it is merely plausible.


Language Shifts

Language Shifts edit

GRE reading passages often contain "language shifts", where the topic of the passage meanders or deviates. The ability to clearly follow a reading passage is an important skilling in scoring well on the test.

Overview edit

Normally writers are taught to write with a clear thesis backed up by supporting sentences. For example:

The grey wolf population was mostly eradicated in the United States in the 19th and 20th centuries. Since wolves were a nuisance to settlers and their livestock, they were targeted by these groups and as a result were exterminated in most of the U.S.. Though its range once extended all the way to Mexico, today the wolf is only found in a few isolated northern regions of the United States.

However, the GRE will typically not do this. Rather, the GRE tends to have passages with meandering topics, probably because clear readable passages would be too easy for test takers. Compare the prior passage with this one:

The United States was initially populated by many agrarian settlers with livestock. These settlers dealt with many problems in the new nation, one of them being wildlife on the frontier. Efforts were made to exterminate a common nuisance to the settlers - the grey wolf - and as a result, this creature is mostly extinct, save a few isolated northern regions of its former range, which once stretched all the way to Mexico.

While both passages say essentially the same thing, the first one clearly states its thesis and follows up with sentences clearly reinforcing or elaborating upon this thesis. The second begins with a sentence about settlers that does not mention wolves at all, and it is only after two sentences that the grey wolf is even mentioned. Furthermore, the third sentence mentions the grey wolf in a hyphen clause voice (the grammatical subject of the sentence is "efforts"). Yet both passages nonetheless are about the grey wolf population in the United States.

These wolf passages exemplify the common "language shift" seen on the GRE - that a passage will often be about something very different from what its initial sentence would lead a reader to think.

Signal Words edit

Pay attention to words like "but" and "however." Often a language shift will be denoted by one of these signal words. Consider the following passage:

Traditional economics has focused on the importance of the law of supply and demand - the notion that the value of a commodity is inversely proportional to its availability. The principle of supply and demand has long been a cornerstone of economics, though even it has some limitations. For example, the increased availability of a good might spark increased consumer knowledge of a product that they otherwise did not know existed, and prove to generate further demand.

This passage essentially makes the statement that sometimes, increased supply can actually increase demand. For example, if Bart gets a new iPad and shows all his friends - all of whom didn't know that iPads existed - this might generate more iPad demand Notice how the passage "smuggles" this concept in, shifting language with the word "though." The first two sentences seem to be leading up to the idea that supply and demand is a universal, infallible rule of economics, and in fact, the passage is stating something to the contrary.

Practice edit

The famous “Code of Hammurabi” is one of the oldest legal codes discovered by archaeologists, its earliest known version inscribed into an ancient stone found in modern-day Iraq. The code is frequently referenced as an ancient publication of laws to govern the people of Mesopotamia, a signof the early civilization developing in the region. Naturally, one would think the laws were inscribed in stone for the public to view them and understand them. The prevalent academic view, however, is that the stone containing Hammurabi's code, however, probably was not intended as a dissemination of laws to the general public or a publication of the rules of civilization. Few people could read in the early days of Mesopotamia, and written codes would only have been useful to scribes; thus, a public display could do little to inform the public. Scholars rather espouse that the code was in fact much more likely to have been made as a tribute to the king and the current ruling dynasty of Babylon and Mesopotamia, given its written nature and presence in stonework.

1. According to the author, why was the Code of Hammurabi most likely chiseled into stone?

2. What reasons does the author give for believing that the Code of Hammurabi wasn't publicly displayed to inform the public of the laws of ancient Mesopotamia?

Comments edit

The GRE frequently uses very "soft language", that is to say, language which conveys a point in a very subtle manner. Compare the sentence:

Bridget shut the door.
with
The door was quickly pulled shut with a slam, and Bridget managed to catch a glimpse out the window just as she was closing it.

Both of these sentences convey the information that Bridget shut the door. However, in the second sentence, the three operative words necessary to derive this information, that is "Bridget" (the noun), "closing" (the transitive verb), and "door" (the direct object of the verb) are so very far flung across the sentence that it makes it more difficult to derive this inference.

The GRE commonly employs language like this, which is why it is important for test takers to pay attention to the so-called "hard words" (subjects, verbs, and object nouns).

Answers to Practice Questions edit

1. The author believes that the Code of Hammurabi was probably made as a public tribute to the king. The passage begins as a discussion of the code and its historical significance, and explains that “[n]aturally” (line 9) one would think it wade made to inform people of the laws. The code, “however”(line 13) was probably not intended in this fashion.

2. As stated in the previous answer, the author doesn't believe the stone containing Hammurabi's code of laws was intended to inform the public, but rather, as a tribute to the king. The author's rationale for this is that few people could read at the time of Hammurabi; thus, it would have done little good towards informing the public.

This is a good example of a passage with shifting language, which begins as a mere discussion of a topic but later “smuggles in” the author's opinions and viewpoints on the matter.


Extracting Content

Pay close attention to what facts can be deduced based on the information in the passage, and understand the meaning of each term and concept.

Reading passages on the GRE will typically introduce and explain concepts quickly, and go on to discuss them assuming you have a full understanding of what they are. Make sure to understand each item's meaning.

“Telegraph” is typically used to describe communication using electrical signals relayed through wires, such as in the system promulgated by Samuel Morse in 5 the early 1800s. In fact, the terms “telegraph” and “telegraphy” refer to any communication over long distance, such as through modern-day telephones or internet communications. Likewise, “telegraph” 10 refers to predecessors of electrical communication, including the once used semaphore line. A semaphore line is a system of relaying communications over long distances. Also 15 known as a “Chappe Telegraph”, the system is entirely mechanical and relies on no electricity or electronics. Rather, the relay is accomplished by a series of towers with flags or mechanical arms that could be 20 arranged in a particular way so as to convey a message, which would be passed on to the next tower which similarly arranged itself in this fashion. First widely used in Europe in the late 1700s, such 25 means of communication were instrumental in the coalition war against Napoleon in Iberia during the early 19th century and remained useful in Europe until replaced by electrical telegraphs. Though perhaps a 30 primitive means compared to modern technology, the achievements of semaphore lines were unprecedented for their time and contemporaneous technology. Practice

1. What is a semaphore line? How does it work and what does it do?

2. The author mentions the wars against Napoleon primarily to

(A) give an example of successful use of semaphore lines (B) demonstrate the difficulties of communicating over long distances in years past (C) show how semaphore lines could communicate in multiply languages (D) illustrate the problems with predecessors of the semaphore line (E) explain why semaphore lines would've been more widely used without political disharmony

3. According to the passage, what was the main cause of the decline of the semaphore line?

(A) the end of the coalition wars in Europe (B) the lack of widespread electricity to power the system (C) the introduction of a more efficient means of communication (D) the rise of telegraphy in the developed world (E) the introduction of the Chappe Telegraph 3.03 Extracting Content & Drawing Inferences

Answers to Practice Questions edit

1. A semaphore line is a series of towers with mechanical arms which convey messages over long distances by flagging the arms in a certain fashion. According to the passage, they were widely used in Europe before the introduction of the electrical telegraph.

2. (A)

At this point in the passage, the author is explaining how semaphore lines worked and how they were used. The coalition war in Iberia is given as an example of the successful use of the semaphore line. The remaining answers are typical GRE “trap” responses. Answer (B) is incorrect because, while this might be an underlying subject of the passage generally, the use of semaphore lines in the coalition is given as an example of the successful use of semaphore lines, not of the difficulty of using them. Choice (C) is similarly incorrect because languages and translations are not mentioned at all.

Answer (D) is incorrect because the predecessors of the semaphore line are not discussed in detail in the passage, and the passage does not make any comparisons between them and the semaphore line. Choice (E) is similarly outside the scope of the passage - while a war is mentioned, nothing in the passage states that this war had any effect on the use of semaphore lines. If anything, it provided an opportunity for them to be used.

3. (C)

This comes straight from lines 28-29. Often the correct answer to a reading question will be correct but stated in a vague form such as this, and stand amidst four incorrect “trap” answers. Choice (A) is incorrect because, while the coalition wars are mentioned as an instance of the use of the semaphore line, their end is not cited as the cause of semaphore lines' decline. Similarly, (B) is incorrect because semaphore lines do not require electricity - electrical telegraphs do. (D) is incorrect because “telegraphy” according to the author means any form of long-distance communication, and thus there was no “rise” of telegraphy after the semaphore - the semaphore was a telegraph. Finally, (E) is a trap answer - a “Chappe Telegraph” is just another name for a semaphore line according to the passage.

This passage is a good example of a strange word that it is important to understand and define subconsciously while reading a passage.


Content Purposes

On the GRE, some questions will ask “in which sentence does the author ...” and state a vague summary of the function of the sentence.

Practice identifying the roles of particular sentences.

Folklore in ancient civilizations often served to explain facts of the world as well as merely to entertain. The creation of these stories has persisted into modern

5 times - Americans once told stories of a great lumberjack, Paul Bunyan, who felled the trees of the great plains, and created the “Badlands” of South Dakota by tapping his axe during a nap. In Origins of the

10 Cyclops Myth, Matthew Williams argues that the ancient Greeks in fact did believe in or at least prepend the existence of one- eyed giants - cyclopses - in the Mediterranean. According to Williams,

15 Greeks likely encountered elephant skulls and skeletons in their voyages, and the idea of a single-eyed creature came from the presence of the larger aperture in the face of the skull.

Questions edit

  1. In which sentence does the author given an example of modern folklore? What is this example?
  2. In which sentence does the author present a reason why the Greeks may have believed in the actual existence of the cyclops? What is this reason?

Answers to Practice Questions edit

  1. This example occurs in the second sentence. The example is Paul Bunyan, who the author discusses to give an instance of contemporary folklore as well as its use to explain something - the great plains and the badlands.
  2. This occurs in the last sentence. After the author articulates a theory found in what is apparently a scholarly work, the author explains that Greeks believed that there were one-eyed giants because they found elephant skulls on their voyages which had large single holes (“apertures”) in them. They believed, according to this theory, that these were the skulls of one-eyed giants.


Words in Context

The GRE will ask questions about the meaning of words in context.

These will often be words with secondary meanings. Be sure to practice identifying the meaning of words in context.

Narrative of the Life of Frederick Douglass, an American Slave, published in 1845, was an autobiographical work which garnered a great deal of attention 5 and popularity during the abolitionist movement in the United States. However, since the abolition of slavery in the United States, the utility of the book has arguably commuted from that of promoting a 10 particular policy agenda to one of mere history and entertainment for its readers. In this light, several aspects of the book which served its original purpose are inhibitive to these new purposes. For 15 example, the book does not describe how Frederick Douglass escapes from slavery. At the time it was written, this was for obvious reasons - so as not to give away a secret. However, in contemporary 20 context, this arouses great curiosity in readers, who inevitably turn to Douglass' second autobiographical work, Life and Times of Frederick Douglass, published in 1881 after the realization of Douglass' 25 goals of the first book. Practice

1. As used in the passage, “commuted” (line 9) most nearly means

(A) declined (B) shifted (C) appeared (D) manifested (E) elucidated

2. As used in the passage, “inhibitive” (line 14) most nearly means

(A) conducive (B) pejorative (C) ambiguous (D) detrimental (E) counterintuitive

3. As used in the passage, “realization” (line 24) most nearly means

(A) learning (B) receipt (C) promotion (D) expostulation (E) accomplishment

3.05 Words in Context

 Answers to Practice Questions

1. (B)

This is an example of an atypical use of a word being tested on the GRE. “Commuted” means moved or changed. Usually in the 21st century it is used to refer to traveling for work purposes. In this context, however, it refers to a change.

“Declined” means having taken a downward turn or having decreased, while “appeared” means having come into view or presented itself. “Manifested” also means having presented itself or come to fruition, and “elucidated” means having been explained or made clear.

2. (D)

“Inhibitive” means tending to stop or slow something. The passage is explaining how many of the old features of Douglass' book, which was written to inspire the abolitionist movement, are now detrimental to the story; for instance, readers don't know how he escapes.

“Conducive” means tending to accomplish a particular result, while “pejorative” means derogatory or tending to evoke negative feelings about a subject. “Ambiguous” means prone to more than one meaning, and “counterintuitive” means tending to go against one's instinctive feelings on a matter.

3. (E)

This is another good example of a common secondary meaning of a word. As used here, “realization” means the fulfillment or accomplishment of a goal. “Realize” normally means acquiring knowledge of something; however, in this context it refers to accomplishment.

“Learning” means acquiring knowledge, while “receipt” refers to the act of taking something after being given or delivered the thing. “Promotion” means to place something in a better position than before, while “expostulation” refers to stating something to be true or arguing for this to be the case.


Buried Conclusions

The key points of the author are often hidden amidst thick text. Practice identifying them and picking them out.

Though commonly thought of as a theory on the origins of life, biological evolution in fact does not address this topic, but rather, the diversity of life. The concept of 5 the origins of life is called “abiogenesis” and was the question addressed by the famous Miller and Urey experiment. In 1952, the two namesakes of this experiment built a machine which applied 10 heat and electrical sparks to water and a plethora of gases to determine whether amino acids, a fundamental component of life on Earth, could form from these chemicals. The experiment produced over 15 twenty amino acids, but, while frequently cited as a milestone in the research of abiogenesis, the experiment is just as heavily disapproved by many for its inaccuracies. These criticisms are 20 heavily misplaced. Critics frequently point out that the gases used in the Miller and Urey experiment were probably quite different from those which made up the Earth's atmosphere 25 billions of years ago, and therefore, the experiment is inaccurate and its informative nature is limited. The point of the experiment, however, was never to attempt to simulate actual abiogenesis. 30 Miller and Urey were concerned with the question of whether or not amino acids could arise from gases, water, heat, and electricity, and in their research, they proved just what they set out to 35 determined. 1. What was the “Miller and Urey” experiment and what did it seek to accomplish?

2. How did the Miller and Urey experiment proceed? What was done in the experiment?

2. According to the passage, what is a common criticism of the Miller and Urey experiment, and how does the author feel with regard to this criticism?

3.06 Buried Conclusions

Answers to Practice Questions

1. The Miller and Urey experiment sought to see whether amino acids - a basic component of life - could be produced from naturally occurring gases through application of heat and electricity. The experiment was a success, and produced over twenty such amino acids.

2. The Miller and Urey experiment ran a group of different gases (“plethora”) through a machine which applied heat and electrical sparks to these gases as well as water.

2. The passage explains that the Miller and Urey experiment is commonly criticized because the gases used in it were probably quite different from the early atmosphere on Earth. However, the author claims that this is not a valid criticism. As the author explains, the purpose of the experiment wasn't to simulate how life arose on Earth. Rather, the purpose was to determine whether it was possible for amino acids to form naturally under certain circumstances. The experiment showed this, and thus was a success.


Arts and Literature Passages

3.07 Art & Literature Passages

The GRE contains many passages which are commentaries or analyses of arts or literature. Pay attention to theories about the subject discussed.

Liber de Coquina is one of the oldest known cookbooks, dating back to approximately the 13th century, but even at its age, the mysteries surrounding the 5 book are a curiosity. The book seems to have been written by two authors, one French and one Italian, but neither the authors' names nor the date of publication are clear. While this may seem expected 10 for a book of its age, Apicius' De re coquinaria from approximately 500 as well as Viandier by Guillaume Tirel each contained its authors name. The anonymity of the work has led some to 15 believe that the authors had reason to conceal their identity, perhaps because the authors were female. It was considered improper for women to appear in public fields during the Middle Ages; even at the 20 time of Shakespeare, female roles in plays were usually played by men. Yet culinary trades were known to be common amongst women in the Middle Ages. 1. What is Liber de Coquina and how does it compare to similar things of its kind?

2. What does the author think may be the case about the authors of Liber de Coquina?

3. What reasons does the author state for this conclusion?

3.07 Art & Literature Passages

 Answers to Practice Questions

1. Liber de Coquina is a very old cookbook and is distinct from other very old cookbooks in that it is written by two anonymous authors.

2. The author thinks that the authors of the book may have been female.

3. The author's reasons for potential female authorship of Liber de Coquina are both the nature of medieval society - where it was shameful for women to have public professions - as well as the fact that culinary duties were often the prerogative of women in many contexts during the era.


Social Science Passages

GRE passages are often about complicated social science topics.

Linguists are uncertain as to why languages change and modify the way they do - changes perhaps attributable to mere idiosyncratic factors. Whatever the case, 5 the diversity of languages occasioned by geographic dispersal is all too apparent, such as with the Latin-based languages of Italian, Spanish, French, and Portuguese - all of which trace their roots, at least in 10 part, to a uniform common ancestor. Curious, however, is the fact that Portuguese - geographically the furthest of any of the Romance languages from Rome - bears the closest resemblance to Latin, the language of the ancient Romans. The 15 Portugal example perhaps suggests that it is exchange and intermingling which change languages, not time or distance alone, and that Portugal, isolated in its corner of Europe, therefore saw the least of 20 such mingling. The example of Scandinavian languages is certainly consistent with this theory. Norwegian, Swedish, and Danish all trace their roots to the Old Norse spoken by the 25 Vikings. Yet it is Icelandic - the furthest tongue from the Scandinavian peninsula - which bears the strongest resemblance to Old Norse, further supporting the notion that intermingling is a primary cause of 30 linguistic change. 1. What is strangely true about both Icelandic and Portuguese, based on the passage?

2. According to the author of the passage, why might it be the case that Portuguese and Icelandic both bear these characteristics discussed?

3.08 Social Science Passages

Answers to Practice Questions edit

1. According to the passage, both Icelandic and Portuguese are the closest to their linguistic ancestors - Old Norse and Latin, respectively - despite being furthest from the source of these languages.

2. This passage discusses the idea that it may be the interspersal and intermingling of people and cultures which produces a change in languages, not distance and time alone. The author is stating that the reason that Portuguese is most similar to Latin may be the fact that Portugal is more isolated from the rest of the world, while France, Spain, and Italy were much more subject to intermingling with other cultures. The author thinks it may be this “intermingling” which produces linguistic change, as new words and linguistic devices are exchanged, adopted, and substituted into a language.


Hard Science Passages

Hard Science Passages edit

The GRE also contains passages with “hard science” concepts, often which are difficult to understand.

It was not until the early 20th century that the energy convective properties of matter were extensively studied, but during this era, physicists began to explore 5 the manner in which particles behaved when exposed to light, electricity, or other forms of energy. To their surprise, they found that matter tended to convect energy at fixed numeric intervals - a minimum 10 “unit” of measurement which was proposed and calculated by German physicist Max Planck. Planck, and ultimately the entire scientific community, came to except the “quantum theory” - 15 that energy is exchanged in certain basic units which correspond to basic particles of matter. 1. What does “quantum theory” mean?

2. What is the evidence for “quantum theory”?

Comments edit

Answers to Practice Questions edit

1. “Quantum theory” refers to the idea that energy is “quantized” or transmitted in numerical values. According to quantum theory, there is a basic “unit” or “package” in which energy is transmitted.

2. As explained in lines 4-9, scientists found that when energy was exposed to matter, it gave off (“convected”) the energy at particular intervals. This implied that there were numerical “packages” in which the energy was transmitted.


Miscellaneous Passages

Miscellaneous Passages edit

The GRE will also throw in more unusual topics on occasion.

	  By the late 1800s, the concept of a
 	“corporation” - a legal entity separate and
 	distinct from its stockholders - was firmly
 	entrenched in Anglo-American law.  
5	However, its use and creation had a
 	remarkably different context.  The first
 	corporations had been royal “charters” of
 	the British Crown, such as the British East
 	India Company or the Hudson's Bay 
10	Company, and were used to exploit
 	exclusive territorial and trading rights
 	secured through English global imperialism.
	This model, while highly effective for
 	managing global trade, was far less
15	applicable in the United States, which for
 	the brunt of the 1800s was focused on
 	pushing westward within its own
 	borders.  Accordingly, the “charter” - 	
	which typically would require a 
20	government enactment - was of little help.
  	Instead, various states, beginning with
 	New Jersey, began a new trend of the
 	“enabling” corporate statute, under which
 	any citizen could create a corporation to 
25	manage private business affairs.  By the
 	20th century, most states of the United
 	States as well as nations worldwide had
 	created some type of “enabling” statute,
 	allowing private individuals to form 
30	corporations to manage their mutual
 	assets.

1. How did the corporation originate?

2. How was the corporation as allowed by law in the United States, beginning in New Jersey, different from the original corporation used by the British?

3. What differences between Britain and the United States prompted this difference and change in the nature of a corporation?

Comments edit

Answers to Practice Questions edit

1. According to the passage, corporations originated as royal charters by the British Crown to manage economic affairs in various parts of the world. This is explained on lines 6-12, which give examples of the Hudson's Bay Company and the British East India Company.

2. New Jersey's law allowed corporations to be created voluntarily by individuals, as opposed to formed by an act of the government. It thus differed from traditional English law in this regard.

3. As explained in lines 13-20, the economic expansion of the United States was primarily domestic, and thus the British model was of little use to Americans, who sought to have business entities to regulate things internally. Accordingly, the corporation which evolved under American law was one which private individuals could form to manage their affairs, with the permission and protection of the state.


Reading Practice 1

Reading Practice 1 edit

“Tiktaalik” is one of the most famous ancient fossils ever discovered due to its status as a transitional species - a fish which also walked on land. It is perhaps 5 this great reverence of the creature which has led many to ignore its significance quite independent of its place in the evolutionary record. The fossil has also been greatly illuminative due to its status 10 as an ancient fish, and thus, its capacity as a fossil to speak to the physiology of such ancient fish and their differences from their contemporaries. Combined with other fossils, Tiktaalik forms a piece of 15 the puzzle in tracking the development of modern fishes from their pre-Cambrian ancestors.

2. As used in the passage, “reverence” (line 5) most nearly means

(A) indifference (B) exaltation (C) criticism (D) proliferation (E) extinguishment 1. According to the passage, “tiktaalik” is most commonly referenced or discussed because

(A) it dates back to the pre-Cambrian period (B) it has many significant physiological differences from other ancient fish (C) it is the oldest transitional fossil ever found (D) it is an example of an amphibious fish (E) it is not the only fossil from its era of a land-walking fish

3. According to the passage, the author believes which one of the following about tiktaalik?

(A) Prior to its discovery, there were no fossils linking ancient pre-Cambrian fish to modern fish. (B) Its basic physiology is not substantially different from modern fishes today. (C) It spent most of its life in the water despite its terrestrial abilities. (D) It is scientifically valuable due to its status as an ancient fish notwithstanding any capacity to walk on land. (E) It is inconsistent with many other fossils from the era, spurring the need for more research.

3.11 Reading Practice Passage 1

Answers to Practice Questions edit

1. (D)

This comes from the first sentence in lines 1-4. Tiktaalik is most famous because it was a fish that could walk on land.

While the pre-Cambrian period of choice (A) is mentioned, it is not mentioned as the era of tiktaalik. Moreover, there are not “many” physiological differences between tiktaalik and other ancient fish according to the passage - the passage only discusses their similarities. Choice (C) is incorrect because nothing claims that tiktaalik is the oldest transitional fossil. (All fossils are arguably transitional fossils) Choice (E) is similarly not supported because the passage at no point says tiktaalik is the only land-walking fish.

2. (B)

“Reverence” refers to holding something in high regard, which is also the meaning of “exaltation.”

“Indifference” is the lack of any strong opinion on a given matter, while “criticism” is the expression of negative feelings on a matter - the opposite of reverence. “Proliferation” refers to the spreading or multiplying of something, while “extinguishment” means the cessation or expiration of something.

3. (D)

This is arguably the main point of the passage. This passage is stating that while the fossil tiktaalik is primarily remembered for is amphibious characteristic, it nonetheless stands on its own as an example of a prehistoric fish.

Choice (A) is incorrect because, while the author claims tiktaalik links ancient fish to modern fish, the author doesn't claim that it is the only fossil that does this. Choice (B) is incorrect because nothing in the passage compares the physiology of tiktaalik to that of modern fish - tiktaalik is only compared to its contemporaries. Answer (C) is incorrect because nothing in the passage describes how much time tiktaalik spent in the water, and choice (E) is incorrect because there is no such “inconsistency” discussed between tiktaalik and other fossils of the era.


Reading Practice 2

Reading Practice 2 edit

Black Elk Speaks chronicles author John Neihardt's conversations with a shaman of the Oglala Sioux tribe in the early 20th century, and is most well known for its 5 subject's recollection of the Battle of Little Bighorn, or as the author calls it, “The Rubbing Out of Long Hair” in reference to the Sioux name for General Custer. At age 13, Black Elk had participated both in 10 this battle as well as the subsequent altercation at Wounded Knee. However, a focus on the book for these recounts would be misplaced, not only because they are told some forty years after the events occurred, 15 but also because the majority of the work is dedicated not to the Sioux Wars of the late 1800s, but to the spiritual and cultural traditions that Neihardt sought to capture, including the rituals in which the 20 author of the book participated firsthand. 1. The author of the passage holds which of the following views about the book Black Elk Speaks?

(A) It contains a reliable contemporary account of the incidents at Little Bighorn and Wounded Knee. (B) It is the most comprehensive chronicle of Sioux spiritual traditions of any work to date. (C) References to the book for its chronicles of the Sioux Wars miss a greater point of the book. (D) It is also known by the title “The Rubbing Out of Long Hair.” (E) Its author did not have any experience with Sioux culture.

2. Based on the passage, Neihardt's purpose in writing Black Elk Speaks was to do which of the following?

(A) interview a survivor of Little Bighorn and Wounded Knee (B) contrast the views of the Oglala Sioux to that of other Indian tribes (C) describe changes in Sioux life in the early 20th century from their traditional way of living (D) document aspects of traditional Sioux culture (E) capture the entire history of the Sioux Wars of the late 1800s

Comments edit

Answers to Practice Questions edit

1. (C)

This can be inferred from the phrase in lines 11-13, which state that a focus on the accounts of the Sioux conflicts is misplaced.

Choice (A) is incorrect because the author never attests to the reliability of the accounts of Little Bighorn in the book; indeed, the author suggests they may be somewhat compromised given the great stretch of time. Answer (B) is incorrect because the passage makes no claim that the book is the most reliable account. Choice (D) is wrong because “The Rubbing Out of Long Hair” refers to Little Bighorn itself, not the book. (E) is directly contradicted by the passage - the author of Black Elk Speaks interviewed a Sioux shaman.

2. (D)

This comes from lines 17-20. Neihardt wasn't predominantly concerned with interviewing survivors of the Sioux Wars as stated in answers (A) and (E) or contrasting the Sioux with others as in answer (B) - indeed, no other Indian tribes are mentioned. Likewise, nothing explicitly mentions the changes discussed in answer (C). Neihardt's purpose was to document Sioux culture.


Reading Practice 3

Reading Practice 3 edit

The Cathedral of St. Vincent de Paul presents a noteworthy example of a mixture between Moorish and Middle- Eastern architectural styles as well as the 5 traditional European Gothic style - a medley which naturally resulted from the history of the structure. Built in Tunis, the capital of Tunisia, in 1897, the cathedral harkens back to an era which saw a great 10 influx of French and Italians into Tunis, as the city was at the time a French protectorate - thus, its construction drew upon the skills and preferences of the locals, both French and Italian as well as 15 Tunisian. The result is a unique construction, featuring the domes and arches typical of the Islamic mosque, and greeted with palm trees at its front entrance, but adorned with crosses, both at 20 the façade and at its steeple. Though the Roman Catholic population has sharply declined in Tunis since Tunisian independence, the Archdiocese of Tunis still maintains the building and uses it as 25 the seat of its presence in the city.

1. According to the passage, which one of the following is true about the Cathedral of St. Vincent de Paul?

(A) When it was built, Tunis was a protectorate of France. (B) It was the first Roman Catholic cathedral constructed in Tunis. (C) It has features which are also associated with Islamic mosques.

2. Which of the following can be inferred about Tunis, based on the passage?

(A) Its Roman Catholic population has decreased since the Cathedral of St. Vincent de Paul was built. (B) It is the capital of Tunisia. (C) It has many instances of Gothic and other European architectural styles which harken back to its days as a French protectorate.

Comments edit

Answers to Practice Questions edit

1. (A), (C)

Choice (A) can be inferred from lines 10-11, which explain that Tunis was a French protectorate at the time. Answer (C) can be similarly inferred from lines 15-18, which describe the features of the cathedral.

Answer (B), however, cannot be inferred. While the passage states that there was a great influx of Roman Catholics to Tunis, it does not state that the cathedral was the first in the region.

2. (A), (B)

Choice (A) can be inferred from lines 20-23 and their discussion of Tunisian independence. Lines 7-8 establish choice (B) - that Tunis was the capital of Tunisia. The third choice, however, cannot be inferred. The passage doesn't establish that there are many instances of Gothic and other European architectural styles. Only one is discussed - the cathedral in question.


Reading Practice 4

Reading Practice 4 edit

While it is often unclear to readers of the novel Flatbush whether the protagonist’s disappointment is more directed at society or her own family, the better view is that the latter is an allegory for the former. The main character, who moves from Puerto Rico to Brooklyn with her parents at the age of three, progressively elucidates the life of crime, gangs, dereliction and decay that awaits her two brothers, and to a lesser extent herself, as they grow older in the city. However, while the novel clearly propounds better education and nurture as a solution, it is unclear whether the novel is primarily faulting the family unit or civilization as a whole. Indeed, the curious shift between the father’s alcoholism and the decay and despair on the streets of Brooklyn in the 1970s confounds and polarizes many readers as to what, if anything, is the reform message in the book. However, it seems that there is a quite natural solution that the family in the story is a metaphor for society itself, and the two themes are part and parcel.

1. Based on the passage, which of the following is true about the novel Flatbush?

Select all that apply.

(A) It has been interpreted as faulting both society and the family unit for social problems.

(B) It primarily blames external and non-familial sources for urban decay in Brooklyn in the 1970s.

(C) Its protagonist is a Puerto Rican native growing up in Brooklyn in the 20th century.

2. Based on the information in the passage, the author of Flatbush would likely agree with which one of the following statements?

Select all that apply.

(A) Better education is a viable solution to problems of crime and urban decay.

(B) Social problems cannot be solved only at the family level.

(C) Problems within familial units are not the sole cause of social problems.

3. As used in the passage, “polarizes” most nearly means

(A) bifurcates

(B) instigates

(C) adjudicates

(D) invigorates

(E) placates

Comments edit

GRE humanities passages can often be some of the worst in terms of verbose, unclear language. Remember to follow language shifts and understand the "hard word" in sentences - that is, subjects, verbs, and verb objects.

It can sometimes help to try and imagine and re-explain what the author is talking about. For example, this passage is about a novel. Try and imagine what it would mean for an author to say that the novel is describing "dereliction."

Answers to Practice Questions edit

1. (A), (C)

Choice (A) can be inferred from lines 20-22, which explains how the book “confounds” readers as to its social message. Answer (B), however, is incorrect. The book does not primarily blame external sources. Based on the passage, the book blames them both equally and uses one as an allegory for the other.

Answer (C) can be inferred - this is stated in lines 6-8. The mention of the 1970s in line 20 also indicates that the book takes place in the 20th century.

2. (A), (C)

Both (A) and (C) can be inferred from lines 12-14, however, choice (B) cannot, for the sole reason that choice (B) states primary. The book doesn't take the position that anything is a primary cause of anything.

3. (A)

“Bifurcates” means splits in half. As used in the passage, it is discussing how the book instills different opinions in its readers. “Instigates” means starts something, usually a conflict, while “adjudicates” means renders judgment on a particular matter, often in the context of a court. “Invigorates” refers to giving new life to something or encouraging it, while “placates” refers to satisfying or pacifying something.


Reading Practice 5

Reading Practice 5 edit

“Cave man” is a term commonly applied to neolithic peoples throughout the world. The term, however, is a misnomer. Contrary to popular belief, early stone 5 age peoples probably did not live in caves, but rather used them for worship and religious ceremonies. The idea of cave dwellers stems from the discovery of a variety of cave paintings, which were 10 initially thought to have been decorations by inhabitants of the caves. The complexity of these paintings, however, suggests that they were done by people who devoted their time exclusively to 15 making cave paintings, and while some argue this is indicative of prehistoric support for arts and culture, a more prevalent view amongst anthropologists is that these ancient peoples believed that 20 their artistic depictions of creatures on the walls of caves gave them special powers over the creatures, enabling the creatures to be hunted and ultimately eaten.

1. In which sentence does the author express the author's own opinion about the purpose of cave paintings by prehistoric humans?

(A) the first sentence (“'Cave man'...world.”) (B) the second sentence (“The term...misnomer.”) (C) the third sentence (“Contrary...ceremonies.”) (D) the fourth sentence (“The idea...caves.”) (E) the fifth sentence (“The complexity...eaten.”)

2. As used in the passage, the term “misnomer” (line 3) most nearly means

(A) euphemism (B) ambiguity (C) overgeneralization (D) inaccuracy (E) incompletion

3. Based on the information in the passage, why did ancient peoples most likely depict animals on the walls of caves?

(A) to celebrate successful hunts (B) to gain mystical powers over the animals which they hunted (C) to decorate the caves, which they used for ceremonial reasons (D) to demonstrate the level of skill of their artisans (E) to document the various types of animals which they hunted

3.15 Reading Practice Passage 5

Comments edit

Answers to Practice Questions edit

1. (E)

While the fifth sentence initially discusses a view that the author claims some people hold, this very long sentence ultimately goes on to explain what the author thinks - that prehistoric people thought that depicting the creatures they hunted helped achieve power over these creatures, enhancing their hunting skills.

2. (D)

A misnomer is a misleading or inaccurate name. In context, the passage is using it to explain why the term “cave man” is inaccurate since neolithic peoples probably did not live in caves.

“Euphemism” refers to a nicer way of saying something, while “ambiguity” means something which is unclear or open to different interpretations. “Overgeneralization” is something which too broad and not specific enough to its content. “Incompletion” refers to something not yet finished.

3. (B)

As explained in the passage and the first answer, the author is of the opinion that ancient people thought depicting animals gave them powers over the animals they hunted and thus enhanced their hunting skills. There is no mention in the passage of answer (A)'s successful hunts, and while (C) is mentioned as a viewpoint, it is not the author's viewpoint. This is also the case with (D) - while the author mentions some might think the paintings were mere decorations, this is not what the author believes. Choice (E) is similarly incorrect because nothing in the passage discusses the use of the paintings for documentation.


Introduction to GRE Arguments

Introduction to GRE Arguments edit

There are three key types of short passage questions on the GRE:

1. Weakening or strengthening arguments

2. Identifying structures of arguments

3. Explaining circumstances

Weakening or strengthening arguments

 Studies of individuals suffering from schizophrenia have revealed that such patients are typically missing more than half of the normal levels of a glycoprotein known as “reelin” in a region of the brain.  Because this protein is so commonly deficient in persons suffering from schizophrenia, it seems likely that the absence of the protein is a cause or at least a contributing factor to the disorder, and that its supplementation to the individual would be a viable cure for schizophrenia.

Which of the following would most weaken the argument that reelin deficiency contributes to schizophrenia?

(A) The presence of reelin in the brain can be altered by changes in diet and other behavioral modifications. (B) Reelin deficiency is linked to other disorders such as bipolar personality disorder, the causes of which are thought to be unrelated to schizophrenia. (C) Medicine prescribed to patients with mental illnesses often has the effect of lowering reelin levels. (D) Individuals with more severe cases of schizophrenia typically have lower concentrations of reelin in their brains that individuals with milder cases of schizophrenia. (E) Many individuals with schizophrenia have reelin levels which are perfectly normal for the average human.

An argument can be weakened by attacking its underlying logic - the link between the premise and the conclusion. This argument asserts that reelin deficiency must cause schizophrenia. The reason for this conclusion is because reelin is less common in the brains of individuals with schizophrenia. Answer (C) weakens the argument because it demonstrates that, alternatively, it may be the medicine commonly taken by schizophrenics which reduces the levels of reelin - making the shortage an effect rather than a cause.


Argument Basics

Argument Basics edit

Overview edit

Identifying structures of arguments edit

“Nilo-Saharan” is a proposed language family for a group of dialects concentrated predominantly in Sudan, as well as in many other isolated regions. The languages, however, are not a “family” - a group of tongues with a common linguistic ancestor - at all. The proposed evidence for a common ancestor of these languages is the overlapping of many common words and phrases. However, this does not establish their similar roots. For instance, while English and Russian both contain many words derived from Latin, neither is in any way a descendant of this language. Likewise, while the various empires and the historic Kingdom of Nubia may have proliferated many terms into common parlance, the presence of these common words does not establish a common “family” among these diverse languages.

The portions marked in bold play which of the following roles in the argument?

(A) The first is a premise on which the main conclusion of the argument is based; the second is a statement of the implications of this conclusion. (B) The first is the main conclusion of the argument; the second is the general principle which is applied to reach this conclusion. (C) The first is a premise of the argument; the second is the argument's main conclusion. (D) The first is a subconclusion of the argument; the second is an application of the principle of this subconclusion. (E) The first is the main conclusion of the argument; the second is an analogy used to explain the rationale for this conclusion.

As will be explained, a “conclusion” is the main point of an argument, while the “premise” is the stated reason which leads to it. Choice (E) correctly identifies the first bolded sentence as the main conclusion, and the second as an “analogy”, or an example using a similar situation. 4.00 Short Passages

Explaining circumstances edit

These questions give a circumstance and ask to explain it.

Researchers studied the work patterns of computer programmers and applet designers and compared their rates of performance on various instruments. With the workers' knowledge and consent, software was installed on their computers which monitored their rates of progress throughout the study. As expected, the programmers and designers worked more quickly and more efficiently when given high quality, top-of-the line computers on which to do their work. However, curiously, even when the programmers and designers were given low-quality, antiquated equipment by the researchers on which to work, their rate of performance still outpaced normal work rates at their positions using normal equipment.

Which one of the following most explains why the computer programmers' rates of productivity increased when they were given antiquated, low-quality equipment?

(A) Even the lowest quality equipment available in contemporary times is still superior to equipment once available to computer programmers. (B) The equipment used by the programmers utilized different operating systems and command prompts with which they were equally familiar. (C) Individuals who are aware that they are being studied tend to become more self-conscious and to work more quickly and efficiently. (D) Software programming has encountered few changes in basic programming language since the 1970s. (E) The low-quality, antiquated equipment was still able to run all of the necessary programs just as normal and high-quality equipment could.

In this case, the circumstance is that programmers tended to work better with antiquated equipment. Answer (C) helps explain this because, even though the programmers had inferior equipment, they worked faster because they were more self-conscious due to the fact they were being studied.

On questions like these, identify the circumstance, answer by answer, asking what explains it, and why.

4.01 Argument Basics edit

An argument is a claim that something must be true or false, or that must be done or not done, based on statements asserted.

There will be about four argument questions on each GRE verbal section, consisting of short passages such as this one:

“Claims that increased spending on education will increase literacy are unwarranted. The correlations between literacy and educational spending are in fact because literate peoples spend more on education, not because the latter is conducive to the existence of the former.”

In this passage, the author makes a claim - that spending money on education won't increase literacy. The author's rationale for saying this is that, while apparently there is a documented link between spending money on education and literacy, the author asserts that the spending money on education is because of a literate population, not the other way around.

Every logically sound argument consists of a rule, a fact, and a conclusion.

Consider the following argument.

“If LSU lost the football game, Meredith will be sad. Therefore, it appears that Meredith will be sad, since LSU lost the football game last night.”

Rule: If LSU lost the football game, Meredith will be sad. Fact: LSU lost the football game. Conclusion: Meredith will be sad.

A logical gap occurs in an argument when either the rule or the fact is missing, or their conjunction is incomplete.

“Meredith is a huge fan of LSU football. Therefore, it appears that Meredith will be sad, since LSU lost the football game last night.”

Rule: Meredith is a huge fan of LSU football. Fact: LSU lost the football game. Conclusion: Meredith will be sad.

This argument is not logically sound. The fact that Meredith is a huge LSU football fan does not necessarily mean that she will be sad if the team loses. While in the real world, this would not be an unreasonable conclusion, from a logical standpoint based on what it states, it does not necessarily follow. 4.01 Argument Basics

Practice edit

Identify the rule, fact, and conclusion in each of the following arguments, and determine whether they are logically sound.

1. All bears like honey. Andy is a bear. Therefore, Andy likes honey.

2. Republicans invariably like money. Wes is a Republican, so it must be the case that Wes likes money.

3. Eating seafood makes Allyson sick. Thus, it looks like Allyson will be sick, since Allyson ate a large plate of the Ocean Spaghetti at Seaside Restaurant.

4. Bankers always have superior morals to lawyers. It must be the case that Charlie's morals are far below Julie's, since Julie is a banker, and Charlie is a lawyer.

5. Every German likes sauerkraut. Margot loves sauerkraut. Therefore, Margot must be German.

6. All of Salvador Dali's paintings can be classified as “surrealism.” Since this painting is a Salvador Dali painting, this painting can be classified as surrealism.

7. The neighbors own a '67 Shelby Mustang. Anyone who owns a classic muscle car is cool. Therefore, the neighbors are cool.

8. Humpback whales are typically much smaller than gray whales. Therefore, the humpback whale sighted in the inlet was inevitably much smaller than the gray whale spotted in the same inlet.

Comments edit

Answers to Practice Questions edit

1.

Rule: All bears like honey. Fact: Andy is a bear. Conclusion: Andy likes honey.

This argument follows logically. The fact comports with the rule and triggers its necessary conclusion.

2.

Rule: All Republicans like money. Fact: Wes is a Republican. Conclusion: Wes likes money.

This argument logically follows. Again, the fact comports with the rule and triggers its necessary conclusion.

3.

Rule: Eating seafood makes Allyson sick. Fact: Allyson ate Ocean Spaghetti Conclusion: Allyson will be sick.

This argument does not follow logically. The fact does not establish that Allyson ate seafood. It states that she ate “Ocean Spaghetti”, whatever that is. While the name of the food she ate and the restaurant strongly suggest the presence of seafood, the fact doesn't conclusively establish this; thus, the argument does not logically follow.

4.

Rule: Bankers have superior morals to lawyers. Fact: Julie is a banker and Charlie is a lawyer. Conclusion: Charlie's morals are superior to Julie's.

This argument follows logically. The facts comport with the rule and establish what the rule sets out for things which comport with these facts.

4.01 Argument Basics

 Answers to Practice Questions

5.

Rule: All Germans like sauerkraut. Fact: Margot likes sauerkraut. Conclusion: Margot is German.

This argument does not follow logically. The rule is that all Germans like sauerkraut, not that anyone who likes sauerkraut must be German. (Margot could be Polish and like sauerkraut too) The argument is thus backwards.

6.

Rule: All Dali paintings can be classified as “surrealism.” Fact: This painting is a Dali painting. Conclusion: This painting can be classified as “surrealism.”

This argument follows logically. The facts are consistent with the requirements of the rule, and therefore trigger what the rule states about these facts.

7.

Rule: Anyone who owns a classic muscle car is cool. Fact: The neighbors own a '67 Shelby Mustang. Conclusion: The neighbors are cool.

This argument does not follow logically. The rule is that anyone who owns a classic muscle car is cool. The fact doesn't state that the neighbors own a classic muscle car. One must assume that a '67 Shelby Mustang is a classic muscle car - perhaps not an unreasonable assumption, but an assumption nonetheless.

8.

Rule: Humpback whales are typically much smaller than gray whales. Fact: These whales are a humpback and a gray whale, respectively. Conclusion: The first whale is smaller than the second.

This argument does not follow logically. The rule is that Humpback whales are typically much smaller, not that every humpback is smaller than every gray. Always read the “rules” of arguments carefully.


Conclusions and Premises

Conclusions & Premises edit

Overview edit

Identify the conclusion of each argument

The “conclusion” is the ultimate point which the author seeks to have the reader believe. Conclusions are typically buried within very verbose language. For instance:

Thomas Hobbes wrote that life in the state of nature was “solitary, poor, nasty, brutish, and short.” However, many other writers took a more positive view of the state of nature. For instance, well known writers Henry David Thoreau and Ralph Waldo Emerson glorified nature as pure and virtuous compared to the evils of society and civilization. It is clear from this that Hobbes’s view of nature was quite cynical.

The conclusion is the last sentence - that Hobbes's view was cynical.

Conclusions can be located by looking for signal words such as “thus”, “therefore”, “it follows”, “so”, and “conclude.”

The conclusion will be the statement that the other sentences gravitate towards, and thus will often be signaled by language of this sort.

Practice edit

Identify the conclusions in each of the following short passages.

1. The common carp frequently occurs in rivers and lakes, and is often caught by fisherman. Recently, a similar fish, the black carp, has spread into many of the same bodies of water from the fish farms where it is raised. Since the black carp is native to China, it lacks immunity to many local diseases, and since the black carp looks very similar to the common carp, fishermen who catch carp are currently at risk of contracting food poisoning from the fish.

2. Paying taxes to support government welfare programs tends to reduce citizens’ desire to provide assistance to others volitionally, since if people feel they are already contributing to the welfare of others through taxes which fund government welfare expenditures, they will feel that their obligation to society has been fulfilled. It follows that public welfare programs in fact have at least some detrimental impact on the public good.

Comments edit

Answers to Practice Questions edit

1. The conclusion is that fishermen are at risk for food poisoning from catching the fish. This argument is explaining that, because fishermen can't distinguish the fish from other carp, and because the fish isn't immune to local diseases, fishermen will be at risk.

2. The conclusion of the argument is that tax-funded social welfare reduces incentive to give. The reason for this conclusion is that if people are forced to pay taxes, they will feel they have done enough, and will be less inclined to give.


Weakening Arguments

Weakening Arguments edit

Overview edit

Identify the conclusion and reasons for the conclusion when asked to weaken or strengthen an argument, and select the answer that affects these.

The local agriculture commission conducted a survey of local irrigated soybean crops. While ordinarily, over-irrigated or under-irrigated crops have a lower crop yield, in the survey, over-irrigated soybean fields saw the excess water simply drain away from the field unless the field was such that this drainage was inhibited. It follows that farmers need not worry about over-irrigating their soybean crop unless there is currently no way for excess water to drain.

Which one of the following, most weakens the conclusion that farmers need not worry about over-irrigating?

(A) Agriculturalists do not currently understand all the risks associated with over-irrigation. (B) Over-irrigation is not as harmful to crops already without adequate drainage for the water. (C) Excessive irrigation can cause a field with adequate drainage to eventually have inadequate drainage. (D) All productive soybean crops need to be irrigated to some extent. (E) Any excess irrigation will ultimately drain away from a soybean field over a period of time.

The conclusion of the argument is that farmers whose fields have adequate drainage don't have to worry about over-irrigating soybean fields. The reason for this is that excess water always drains away. Choice (C) weakens this conclusion because it demonstrates that, even if the excess water drains away, this excess water might inhibit the ability of excess water to drain from the field in the future.

In weakening or strengthening arguments, the key concept is “relevance.” The correct response will make the conclusion more or less likely. 4.03 Weakening & Strengthening Arguments

Practice edit

1. Many people refrain from eating any animal products. But most people who do not eat meat will still consume dairy products, eggs, and other foods made from part of an animal as long as this does not involve hurting the animal. The production of dairy products, however, involves inhumane things such as capturing an animal and forcing labor upon it, as do eggs and invariably any other animal product. Therefore, most people who do not eat meat are still prolonging inhumane things to be inflicted on animals.

Which one of the following most strengthens the argument that most people who do not eat meat are still prolonging inhumane treatment of animals?

(A) People who consume things, the production of which involves inhumane treatment of animals, are invariably assuring the continued existence such inhumane behavior. (B) Inhumane treatment of animals would be substantially reduced if a large fraction of the population refrained from eating animal products like eggs. (C) The primary reason that people refrain from eating meat is to stop the inhumane treatment of animals. (D) The majority of people who eat meat also eat other animal products such as dairy and eggs. (E) There are few people who wish to abandon eating all products produced by animals entirely.

2. Mozart’s opera “The Magic Flute” tells the story of a hero who finds himself in a faraway land, one which in many productions is reminiscent of ancient Egypt. Indeed, at the beginning of Act II, the people in the faraway land sing a hymn to the Egyptian deities Osiris and Dionysus. Because the hymn sounds much like the German hymns of Mozart’s time, some musicologists argue that Mozart’s opera was poking fun at contemporary religion.

Which one of the following, if true, would most weaken the conclusion that Mozart was ridiculing contemporary religion?

(A) The monarchy and clergy under whom Mozart lived imposed severe punishments for ridiculing religion or the national church. (B) Many modern pieces of musical theater satirize contemporary cultural elements such as religion. (C) Many people who lived in Mozart’s time would not have found it funny to ridicule the church or religion. (D) Other operas written by Mozart contained satire of contemporary society, but not of religion. (E) “The Magic Flute” pokes fun at many controversial things, and many modern versions of it omit sections that seem sexist or racist.

Comments edit

Answers to Practice Questions edit

1. (A)

Choice (A) states a principle which demonstrates that people who consume things which impel a result are invariably furthering that result. When applied to these facts, this principle means that eating dairy products prolongs animal suffering.

2. (A)

The argument's conclusion is that Mozart was making fun of religion. The rationale for this is the similarities between a hymn in Mozart's work and contemporary religious hymns. Choice (A) weakens this because it gives a reason why poking fun at religion at the time of Mozart would have been implausible.


Explaining Facts

Explaining Facts edit

Overview edit

Identify the fact that the question asks you to explain and the reasons surrounding that fact.

From 1993 to 1997 the local government passed a series of enactments which were very friendly to tenants and landlords, giving tenants greater rights against landlords in court with regard to rent disputes and evictions. Surprisingly, however, landlords' average profits per residential tenant increased after the enactments were passed.

Which of the following most explains why profits per tenant increased after the enactments?

(A) The total number of residential rental units increased from 1993 to 1997. (B) Many landlords ceased renting to persons with low incomes or questionable rental histories after the enactments in question. (C) Income from commercial rentals increased after the 1993 enactments. (D) The government also repealed old enactments giving special rights to tenants as part of the enactments in question. (E) The majority of rental agreements do not lead to either rent disputes or evictions.

The correct answer is choice (B). The key is that profits per residential tenant increased. The explanation is that landlords simply ceased renting to lower-income tenants altogether. 4.04 Explaining Facts

Practice edit

A “depreciating asset” is one that is bound to continue losing value over time. Depreciating assets are never a good investment for the long-term because they are bound to be worth less than one paid for them. Given this reality, it is strange that Raritan Holdings in its acquisition of Casimir, Inc., wanted Casimir's severely depreciating real estate rental properties so badly.

1. Which one of the following, if true, most explains why Raritan Holdings. would want to make sure it acquired the real estate rental properties in the acquisition?

(A) Acquired depreciated assets can be used to reduce tax liability of Raritan Holdings. (B) Raritan Holdings acquired many appreciating assets in the deal as well. (C) The depreciating assets had already lost most of their value. (D) Some assets neither appreciate nor depreciate, but tend to hold their value. (E) The gross national product in the rental sector has increased in recent years.

About one fourth of the horses at Lorimer Valley Downs that were under 3 years of age needed to be visited by the veterinarian this year. Interestingly, those over 3 years of age were less likely to need to be visited by the veterinarian, though about a fifth of them still did need such a visit.

2. Which one of the following statements, if true, most helps to explain while older horses at Lorimer Valley Downs were less likely to need veterinary visits?

(A) Lorimer Valley Downs has its employees do basic health examinations for all its horses, regardless of their age. (B) Riders of older horses are typically more gentle with them than they are with younger horses. (C) The older a horse becomes, the more likely the horse will be retired from Lorimer Valley Downs due to a decrease in speed. (D) Most younger horses at Lorimer Valley Downs tend to be of significantly different breeds than the older horses. (E) The majority of illnesses that afflicted the younger horses could have been avoided with better regular checkups.

Comments edit

Answers to Practice Questions edit

1. (A)

Choice (A) explains the fact because if the assets can reduce tax liability, they will provide a benefit to their new owner, even if depreciating - and this tax benefit may in fact prove to be greater than any future depreciation.

2. (B)

Answer (B) explains the fact because if the riders are nicer to the older horses, there will tend to be fewer injuries amongst these older horses. While answer (C) is tempting, remember that the fact is amongst horses which are at Lorimer; thus, it doesn't matter if many older horses are retired; the older ones which are still there are still less likely to need medical attention.


Roles in Arguments

Roles in Arguments edit

Overview edit

Identify the role of each sentence in relation to the argument's conclusion.

A society's policy toward its justice system reflects the society's concept of morality. Nations must therefore have fundamentally different concepts of morality. Nations around the world have drastically different justice systems, which vary widely. It is thus impossible for the world to have a single unified set of rules or regulations. Groups with fundamental differences with regard to morality are necessarily precluded from having uniform rules between them.

The words marked in bold play which of the following roles in the argument?

(A) The first is a subconclusion which it uses to reach its main conclusion; the second is an additional premise which the argument uses to reach this main conclusion. (B) The first is the main conclusion of the argument; the second is an additional premise which the argument uses to reach this main conclusion. (C) The first is a premise which the argument uses to draw its main conclusion; the second is an inference which can be drawn on the basis of this conclusion. (D) The first is a subconclusion which it uses to reach its main conclusion; the second is the argument's main conclusion. (E) The first is a premise which the argument uses to draw a subconclusion; the second is a subconclusion which the argument uses to reach its main conclusion.

The answer above is (A). The first sentence in bold is a subconclusion - one used to reach another conclusion in the second to last sentence. The second is a principle, or premise, used to reach this main conclusion.

Practice edit

Groceries are generally more expensive in Alaska than in other places because most of the groceries must be transported to Alaska from faraway places. The long transportation incurs costs, which pass on to consumers at Alaskan grocery stores. This means that if individuals move to Alaska, they will probably pay more for groceries each month.

1. The argument above proceeds by

(A) suggesting an alternative course of action to avoid an undesirable result (B) establishing a general fact and then applying that principle to a given set of facts (C) using an analogous situation to illustrate the problems with pursuing a given end (D) showing that certain measures, if followed, would lead to unacceptable conclusions (E) appealing to authority to establish what it seeks to prove

All too often contemporary academics criticize the government and public policy, but make no effort on their own behalf to change it. Such academics have no right to make such criticism. After all, one who willingly lets a malfeasance be inflicted upon them is not within his or her right to lament the results of that malfeasance.

2. The statement in bold plays which one of the following roles in the argument?

(A) a factual basis for articulating that malfeasance is currently being inflicted by the government (B) an example of the type of criticism given by contemporary academics (C) a general principle which allows the conclusion of the argument to be drawn (D) an illustration of a key problem for which the passage proposes a solution (E) evidence for an opposing claim which is subsequently dismissed

Comments edit

Answers to Practice Questions edit

1. (B)

The “general fact” is that things cost more in Alaska. Applied to individuals moving, the principle serves to impel the conclusion that more money need be spent on groceries.

2. (C)

The second sentence is the conclusion of the argument, which states that academics have no right to criticize the government. The rationale for this is found in the third sentence - that academics do not make any effort to change things, and that absent such an effort, they have no right to complain.


The GRE Writing Section

The GRE Writing Section edit

The writing section is of rather small importance in admissions decisions compared to the other two; nonetheless, it is worthwhile for test takers to do the best they can on it.

As stated at the beginning, there are two writing sections. One is an analysis of an issue; the other is an analysis of an argument.

There are three things to remember about the writing section.

1. Write a lot. The single strongest objective predictor of high essay grades is length.

2. Sound intelligent.

✗Bad ✓Good
Things like social security are a good idea. It is good to have a social safety net for seniors. Social security helps keeps seniors out of poverty. The beneficial nature of social security is clear, given the public interests furthered by the safety net it provides, and the extent to which it keeps seniors out of poverty.

3. Don't ramble.

✗Bad ✓Good
Direct voting by the public on political issues would help in some cases. It would depend, however, on the complexity of the matter - some things would be incomprehensible to the lay person. This raises an interesting question - is the average person cut out to make leadership decisions? Historically democracies have usually exercised some level of autonomy. Of course, there are varying levels of autonomy in the United States. Nonetheless, the United States has managed to accede to public will on most issues historically. Direct voting by the public on political issues would help in some cases. The government can only remain stable and effective if it serves the people, and direct voting would bring about this end. However, the question arises as to whether some issues are too complex for the average voter. Many laws are quite complicated, and the lay person may not have the necessary expertise to properly evaluate changes to them.

Keep thoughts concise and coherent; and, generally, use common sense on the writing section. Again, however, the section is not particularly important. Do not let it take away from the other very important sections.